Energian siirtyminen lämpösäteilyn avulla

fysiikan.kertausta

Ilmastonmuutoksessa lämpösäteilyllä on suuri merkitys energian siirtäjänä.

Palstalla on usein ollut keskustelua siitä, miten energia siirtyy lämpösäteilyn välityksellä esimerkiksi kuumemman ja kylmemmän kappaleen välillä. Tässä keskustelussa on tarkoitus käsitellä asiaa sen mukaisesti, mitä nykyisen fysiikan mukaan aiheesta tiedetään eli käytännössä mennään oppikirjojen mukaan.

226

4256

    Vastaukset

    Anonyymi (Kirjaudu / Rekisteröidy)
    5000
    • fysiikan.kertausta

      Google - haku lauseella "Nettomääräinen lämmön siirtyminen pinnalta toiselle" tuottaa osuman wikipediaan lämpösäteilyä käsittelevälle sivulle. Sieltä löytyy otsikon "Säteilynä siirtyvä lämpömäärä" alta lyhyt selostus siitä, miten kappaleen ympäristön lämpötila vaikuttaa kappaleen ja sen ympäristön välillä siirtyvään lämpömäärään.

      Kaavoissa esiintyy kirjaimia, joiden yläpuolella on piste. Se on fysiikassa usein käytetty tapa merkitä suureen muutosnopeutta eli derivaattaa ajan suhteen. Esimerkiksi "Q piste" on lämpömäärän Q (yksikkönä Joule) muutos aikayksikössä eli dQ/dt eli siirtyvä lämpöteho, yksikkönä Joulea sekunnissa eli Watti. Tehohan on energian (tässä lämmön) muutos aikayksikköä kohti.

      • Tiedoksesi

        Fysiikalle on oma palstansa.


      • kertauksen.syyt
        Tiedoksesi kirjoitti:

        Fysiikalle on oma palstansa.

        Tämä nimenomainen asia nousee ilmastopalstalla esille uudelleen ja uudelleen keskusteltaessa kylmemmän ympäristön vaikutuksesta Maapallon pintalämpötiloihin. Se liittyy oleellisesti ilmastonmuutokseen siksi, että kylmemmän ympäristön lämpötilan muutosten väitetään olevan ei-relevantteja lämpimämmän pinnan menettämän energian määrän kannalta. Siitä syystä johtuen on tarvetta käsitellä asia yksityiskohtaisesti.

        Lähes mikä tahansa ilmastoon ja ilmakehään liittyvä ilmiö on pohjimmiltaan fysiikkaa. Silti niitä käsitellään täällä. En ole nähnyt protesteja keskusteltaessa esimerkiksi CO2 molekyylien kyvystä absorboita säteilyä vaikka siinä ollaan vahvasti kaasumolekyylien kvanttimekaniikan eli fysiikan puolella.


      • Anonyymi
        Tiedoksesi kirjoitti:

        Fysiikalle on oma palstansa.

        Fysiikka tulee esille monen keskustelun osana.


      • Anonyymi
        Anonyymi kirjoitti:

        Fysiikka tulee esille monen keskustelun osana.

        Tässä se fysiikka on aika oleellinen asia.


    • Ei.ole.tarpeellista

      Palstan aihe on ilmastonmuutos !
      Termodynamiikan ja säteilyvaikutusten osalta niiden vaikutusten toteaminen on riittävä.
      Näiden olmiöiden perustat ja tarkempi yksityiskohtainen erittely kuuluu fysiikan piiriin.
      On täysin kohtuutonta lähteä perkaamaan jokaisen yleisesti tunnetun ja todennetun ilmiön todistelua ja erittelyä aina erikseen.
      Aihe ei kulu ilmastonmuutosaiheen käsiteltäväksi, joten olen pyytänyt sen poistettavaksi.

      • harhan.pysäytys

        Ilmakehän (kylmempi kappale) ja Maapallon pinnan (lämpimämpi kappale) välinen energian vaihto lämpösäteilyn välityksellä on ilmastonmuutoksen ytimessä. Et kai vain pelkää, että asian käsitteleminen alusta loppuun asti kunnolla pilaa mahdollisuudet esittää siitä jatkossa harhaanjohtavia väitteitä?


      • joko.taas.tätä
        harhan.pysäytys kirjoitti:

        Ilmakehän (kylmempi kappale) ja Maapallon pinnan (lämpimämpi kappale) välinen energian vaihto lämpösäteilyn välityksellä on ilmastonmuutoksen ytimessä. Et kai vain pelkää, että asian käsitteleminen alusta loppuun asti kunnolla pilaa mahdollisuudet esittää siitä jatkossa harhaanjohtavia väitteitä?

        Maapallo ei todellakaan ole musta kappale.
        Et kai luule, että nuo jostain nettisivulta kopioidut lauseet ja teoreettiset laskelmat todistavat todellisuutta ilmakehän kokonaislämmönsiirrosta?


      • askel.kerrallaan
        joko.taas.tätä kirjoitti:

        Maapallo ei todellakaan ole musta kappale.
        Et kai luule, että nuo jostain nettisivulta kopioidut lauseet ja teoreettiset laskelmat todistavat todellisuutta ilmakehän kokonaislämmönsiirrosta?

        Tässä olikin tarkoitus kuvata yksinkertaisimmillaan mekanismi, jolla ulkoapäin lämpöä vastaanottavan kappaleen lämpötila riippuu sen ympäristön lämpötilasta kun kappaleesta poistuvan lämmön siirtomekanismina on lämpösäteily. Tuo tavoite on nyt saavutettu.

        Jatkossa on tällä palstalla turha yrittää väittää, että ulkoapäin lämmitetyn kappaleen ympäristön lämpötila ei vaikuttaisi kappaleen tasapainolämpötilaan silloin, kun tuo ympäristö on kappaletta kylmempi.

        Maapallon pinnan emissiivisyys on pitkäaaltoisen lämpösäteilyn kannalta katsottuna suurelta osin varsin lähellä ykköstä. Jää, vesi ja kasvillisuus omaavat pitkäaaltoiselle lämpösäteilylle emissiokertoimen, joka on tyypillisesti luokkaa 0.95 eli ovat hyvin lähellä mustaa kappaletta. Poikkeuksena ovat kuivat aavikkoalueet, joiden emissiivisyys on pikemminkin luokkaa 0.6.

        Virhettä ei tule Maapallon pinnan vaan ilmakehän mallintamisesta mustana kappaleena. Pilvipeite voi käyttäytyä paksuudestaan riippuen harmaan tai mustan kappaleen . Pilvettömän ilmakerroksen ja siinä olevien kasvihuonekaasujen mallintaminen ylipäätään "kappaleena" on huonosti perusteltua.

        Jotta mutkikkaampia malleja voisi käsitellä on kuitenkin ensin käsiteltävä niiden pohjana olevat yksinkertaiset mallit. Tyvestä puuhun siis.


      • näin.se.on
        askel.kerrallaan kirjoitti:

        Tässä olikin tarkoitus kuvata yksinkertaisimmillaan mekanismi, jolla ulkoapäin lämpöä vastaanottavan kappaleen lämpötila riippuu sen ympäristön lämpötilasta kun kappaleesta poistuvan lämmön siirtomekanismina on lämpösäteily. Tuo tavoite on nyt saavutettu.

        Jatkossa on tällä palstalla turha yrittää väittää, että ulkoapäin lämmitetyn kappaleen ympäristön lämpötila ei vaikuttaisi kappaleen tasapainolämpötilaan silloin, kun tuo ympäristö on kappaletta kylmempi.

        Maapallon pinnan emissiivisyys on pitkäaaltoisen lämpösäteilyn kannalta katsottuna suurelta osin varsin lähellä ykköstä. Jää, vesi ja kasvillisuus omaavat pitkäaaltoiselle lämpösäteilylle emissiokertoimen, joka on tyypillisesti luokkaa 0.95 eli ovat hyvin lähellä mustaa kappaletta. Poikkeuksena ovat kuivat aavikkoalueet, joiden emissiivisyys on pikemminkin luokkaa 0.6.

        Virhettä ei tule Maapallon pinnan vaan ilmakehän mallintamisesta mustana kappaleena. Pilvipeite voi käyttäytyä paksuudestaan riippuen harmaan tai mustan kappaleen . Pilvettömän ilmakerroksen ja siinä olevien kasvihuonekaasujen mallintaminen ylipäätään "kappaleena" on huonosti perusteltua.

        Jotta mutkikkaampia malleja voisi käsitellä on kuitenkin ensin käsiteltävä niiden pohjana olevat yksinkertaiset mallit. Tyvestä puuhun siis.

        Kyllä sitä virhettä tulee melkoisesti maapallon pinnankin mallintamisessa.
        70% on vettä ja sen huomioiminen eri kulmista ja lisäksi jäätiköitä on useita miljoonia neliökilometrejä. Ja lisäksi vuodenaikojen vaihtelut ja lumipeitteen osuus ja sen vaikutus, niin ei tuota yhdellä kaavalla pysty hoitamaan, joten yleensä nuo on ihan poskellaan.


      • ScienceofDoom-blogi
        näin.se.on kirjoitti:

        Kyllä sitä virhettä tulee melkoisesti maapallon pinnankin mallintamisessa.
        70% on vettä ja sen huomioiminen eri kulmista ja lisäksi jäätiköitä on useita miljoonia neliökilometrejä. Ja lisäksi vuodenaikojen vaihtelut ja lumipeitteen osuus ja sen vaikutus, niin ei tuota yhdellä kaavalla pysty hoitamaan, joten yleensä nuo on ihan poskellaan.

        Jään, lumen ja veden emissiivisyys pitkäaaltoiselle lämpösäteilylle on noin 0.95 eli juuri niiden alueiden kohdalla Maapallo käyttäytyy lämpösäteilyn kannalta parhaiten kuten musta kappale. Näiden ominaisuudet lämpösäteilylle löytyvät esimerkiksi lämpökuvauksessa käytetyistä taulukoista.

        Näkyvän valon kannalta tilanne on tietenkin toinen eli se on huomioitava tulevan energian jakautumisessa Maapallon pinnalle.

        Näkyvän valon tapaus veden pinnassa on suoraan laskettavissa Fresnelin kaavoista veden taitekertoimen avulla. Käy ilmi, että veden emissiivisyys näkyvälle valolle on yli 0.93, kun valo osuu veteen korkeintaan 60 asteen kulmassa kohtisuoraan verrattuna. Kun kulma loivenee 80 asteeseen niin emissiivisyys putoaa 0.65:een. Merien pinta on tuulen vuoksi pienten aaltojen peittämää, joten siinä tulee emissiivisyys keskiarvoistettua yli eri osumakulmien.

        Merien pinta käyttäytyy näkyvän valonkin kannalta varsin tarkkaan samoin kuin musta kappale. Merenpinnan emissiivisyydestä on SoD - blogissa varsin hyvä esitys kuvineen ja niiden selityksineen. Siinä on myös kerrottu selityksessä käytettyjen kaavojen ja mittaustulosten alkuperä eli lähdeviitteet on merkitty näkyviin. Löytyy hakusanoilla

        27 2010 stefan-boltzmann 8-14 29 Miriam Sidran


      • kauas.pilvet.karkaa
        ScienceofDoom-blogi kirjoitti:

        Jään, lumen ja veden emissiivisyys pitkäaaltoiselle lämpösäteilylle on noin 0.95 eli juuri niiden alueiden kohdalla Maapallo käyttäytyy lämpösäteilyn kannalta parhaiten kuten musta kappale. Näiden ominaisuudet lämpösäteilylle löytyvät esimerkiksi lämpökuvauksessa käytetyistä taulukoista.

        Näkyvän valon kannalta tilanne on tietenkin toinen eli se on huomioitava tulevan energian jakautumisessa Maapallon pinnalle.

        Näkyvän valon tapaus veden pinnassa on suoraan laskettavissa Fresnelin kaavoista veden taitekertoimen avulla. Käy ilmi, että veden emissiivisyys näkyvälle valolle on yli 0.93, kun valo osuu veteen korkeintaan 60 asteen kulmassa kohtisuoraan verrattuna. Kun kulma loivenee 80 asteeseen niin emissiivisyys putoaa 0.65:een. Merien pinta on tuulen vuoksi pienten aaltojen peittämää, joten siinä tulee emissiivisyys keskiarvoistettua yli eri osumakulmien.

        Merien pinta käyttäytyy näkyvän valonkin kannalta varsin tarkkaan samoin kuin musta kappale. Merenpinnan emissiivisyydestä on SoD - blogissa varsin hyvä esitys kuvineen ja niiden selityksineen. Siinä on myös kerrottu selityksessä käytettyjen kaavojen ja mittaustulosten alkuperä eli lähdeviitteet on merkitty näkyviin. Löytyy hakusanoilla

        27 2010 stefan-boltzmann 8-14 29 Miriam Sidran

        Ja aina tahtoo unohtua se pilvisyys.


      • pilvet.huomiotu.myös
        kauas.pilvet.karkaa kirjoitti:

        Ja aina tahtoo unohtua se pilvisyys.

        Ei se aina unohdu.


      • nettosiirronteho
        askel.kerrallaan kirjoitti:

        Tässä olikin tarkoitus kuvata yksinkertaisimmillaan mekanismi, jolla ulkoapäin lämpöä vastaanottavan kappaleen lämpötila riippuu sen ympäristön lämpötilasta kun kappaleesta poistuvan lämmön siirtomekanismina on lämpösäteily. Tuo tavoite on nyt saavutettu.

        Jatkossa on tällä palstalla turha yrittää väittää, että ulkoapäin lämmitetyn kappaleen ympäristön lämpötila ei vaikuttaisi kappaleen tasapainolämpötilaan silloin, kun tuo ympäristö on kappaletta kylmempi.

        Maapallon pinnan emissiivisyys on pitkäaaltoisen lämpösäteilyn kannalta katsottuna suurelta osin varsin lähellä ykköstä. Jää, vesi ja kasvillisuus omaavat pitkäaaltoiselle lämpösäteilylle emissiokertoimen, joka on tyypillisesti luokkaa 0.95 eli ovat hyvin lähellä mustaa kappaletta. Poikkeuksena ovat kuivat aavikkoalueet, joiden emissiivisyys on pikemminkin luokkaa 0.6.

        Virhettä ei tule Maapallon pinnan vaan ilmakehän mallintamisesta mustana kappaleena. Pilvipeite voi käyttäytyä paksuudestaan riippuen harmaan tai mustan kappaleen . Pilvettömän ilmakerroksen ja siinä olevien kasvihuonekaasujen mallintaminen ylipäätään "kappaleena" on huonosti perusteltua.

        Jotta mutkikkaampia malleja voisi käsitellä on kuitenkin ensin käsiteltävä niiden pohjana olevat yksinkertaiset mallit. Tyvestä puuhun siis.

        Taas joku yrittää väittää, että ympäristön lämpötila ei vaikuta lämmön nettosiirron tehoon. Tällä kertaa tuo esitettiin keskustelussa, jonka otsikko kuuluu "Tieteistarinat jatkuu !!!"


      • Anonyymi
        pilvet.huomiotu.myös kirjoitti:

        Ei se aina unohdu.

        Ei aina.


      • Anonyymi
        harhan.pysäytys kirjoitti:

        Ilmakehän (kylmempi kappale) ja Maapallon pinnan (lämpimämpi kappale) välinen energian vaihto lämpösäteilyn välityksellä on ilmastonmuutoksen ytimessä. Et kai vain pelkää, että asian käsitteleminen alusta loppuun asti kunnolla pilaa mahdollisuudet esittää siitä jatkossa harhaanjohtavia väitteitä?

        Maapallo ei todellakaan sovi fyysikoiden koepalloksi, on se sen verran monimutkainen kokonaisuus. Kun fyysikot tekevät kokeitaan niin koeolosuhteet pyritään rajaamaan tarkasti, jotta satunnaiset tekijät eivät vaikuta tutkimuksen kohteena oleviin muuttujiin. Ilmastoa ei voida tutkia yksinkertaistamalla, propagandaa siitä kyllä voidaan tehdä nimenomaan yksinkertaistamalla. Sää- ja ilmastotieteessä tutkitaan monimutkaista todellisuutta, missä satunnaiset tekijät ovat aina kuvassa mukana joten on pakko operoida todennäköisyyksillä. Parempia tuloksia saadaan keräämällä enemmän tietoa ja käsittelemällä sitä tehokkaammin. Tietokonemallit ovat tässä ilmastotutkijoiden tärkeä työkalu, mutta ne eivät ole sama asia kuin todellisuus vaan epätarkka, jopa sumea kuva todellisuudesta. Ei mallien perusteella pitäisi voida tehdä ehdottomia päätelmiä.

        Palstan fyysikot yrittävät fyysikoiden tapaan rajata maapallon yksinkertaistetuksi koekappaleeksi, jossa ilmasto määritellään energian vaihdoksi. jota käsitellään lämpöopin ja säteilyenergian lakien avulla. Ei toimi sellainen yksinkertaistus. Ilmastomallit sen sijaan voisivat toimia paremmin jos yksi olennainen tekijä olisi tarkkaan määritelty lainalaisuus. Se tekijä on ilmaston herkkyys kasvihuoneilmiölle, eli infrapunasäteilyn absorbtion hidastava vaikutus lämmön haihtumiseen maapallolta avaruuteen. Tämä olennainen tekijä saadaan nyt juuri niistä samoista ilmastomalleista, joissa se sitten on itse tärkeä tekijä. Ilmastomallinnus on sen vuoksi simulointia, jota pitää koko ajan vertailla todellisuuteen. Ilmastomallit toimivat jos niiden antamat ennusteet osuvat kohdalleen. En nyt tarkkaan tiedä, miksi ilmaston herkkyyttä kasvihuoneilmiölle ei ole fysiikan perusteella kyetty kvantifioimaan, näin vain kuitenkin on, sen kyllä ihan kaikki lähteet kertovat kun googlaa "climate sensitivity" -haulla.


      • Anonyymi
        Anonyymi kirjoitti:

        Maapallo ei todellakaan sovi fyysikoiden koepalloksi, on se sen verran monimutkainen kokonaisuus. Kun fyysikot tekevät kokeitaan niin koeolosuhteet pyritään rajaamaan tarkasti, jotta satunnaiset tekijät eivät vaikuta tutkimuksen kohteena oleviin muuttujiin. Ilmastoa ei voida tutkia yksinkertaistamalla, propagandaa siitä kyllä voidaan tehdä nimenomaan yksinkertaistamalla. Sää- ja ilmastotieteessä tutkitaan monimutkaista todellisuutta, missä satunnaiset tekijät ovat aina kuvassa mukana joten on pakko operoida todennäköisyyksillä. Parempia tuloksia saadaan keräämällä enemmän tietoa ja käsittelemällä sitä tehokkaammin. Tietokonemallit ovat tässä ilmastotutkijoiden tärkeä työkalu, mutta ne eivät ole sama asia kuin todellisuus vaan epätarkka, jopa sumea kuva todellisuudesta. Ei mallien perusteella pitäisi voida tehdä ehdottomia päätelmiä.

        Palstan fyysikot yrittävät fyysikoiden tapaan rajata maapallon yksinkertaistetuksi koekappaleeksi, jossa ilmasto määritellään energian vaihdoksi. jota käsitellään lämpöopin ja säteilyenergian lakien avulla. Ei toimi sellainen yksinkertaistus. Ilmastomallit sen sijaan voisivat toimia paremmin jos yksi olennainen tekijä olisi tarkkaan määritelty lainalaisuus. Se tekijä on ilmaston herkkyys kasvihuoneilmiölle, eli infrapunasäteilyn absorbtion hidastava vaikutus lämmön haihtumiseen maapallolta avaruuteen. Tämä olennainen tekijä saadaan nyt juuri niistä samoista ilmastomalleista, joissa se sitten on itse tärkeä tekijä. Ilmastomallinnus on sen vuoksi simulointia, jota pitää koko ajan vertailla todellisuuteen. Ilmastomallit toimivat jos niiden antamat ennusteet osuvat kohdalleen. En nyt tarkkaan tiedä, miksi ilmaston herkkyyttä kasvihuoneilmiölle ei ole fysiikan perusteella kyetty kvantifioimaan, näin vain kuitenkin on, sen kyllä ihan kaikki lähteet kertovat kun googlaa "climate sensitivity" -haulla.

        Asenteella "liian vaikeaa, ei kannata edes yrittää" ei olisi tehty tietokoneita eikä internettiä jonka ansiosta pääsit mielipiteesi kertomaan.


      • Anonyymi

        Millä muulla kuin fysiikalla ilmaston ilmiöitä pitäisi käsitellä. Olkiukkoja pökustemällä ?


    • lämmön.nettosiirron.teho

      Jotta lämmönsiirtoa lämpösäteilyn avulla voisi ymmärtää pitää heti aluksi erottaa toisistaan kaksi käsitettä, jotka ovat LÄMPÖSÄTEILYN TEHO ja LÄMMÖN NETTOMÄÄRÄISEN SIIRTYMISEN TEHO.

      Ilmastokeskusteluissa olen nähnyt yllä esitettyjen kahden käsitteen menevän usein sekaisin. Näin käy varsinkin silloin, kun keskustellaan kappaletta kylmemmän ympäristön lämpötilan muutosten vaikutuksesta lämmön siirtymiseen kappaleesta ulospäin. Lämpösäteilyn teho ei ole sama asia kuin lämmön nettomääräisen siirtymisen teho.

      LÄMPÖSÄTEILYN TEHO tarkoittaa sitä tehoa, jolla kappale lähettää lämpösäteilyä ympäristöönsä. Tuon tehon määrää Stefan-Boltzmannin laki. Jokainen kappale koko ajan sekä lähettää ulospäin että ottaa vastaan siihen osuvaa lämpösäteilyä. Kappaleen lähettämän lämpösäteilyn teho riippuu kappaleen pinta-alasta A, sen pinnan emissiviteetistä ε ja pinnan lämpötilan T neljännestä potenssista T^4.

      LÄMMÖN NETTOMÄÄRÄISEN SIIRTYMISEN TEHO tarkoittaa sitä lämpömäärää sekunnissa, mikä kuumemmasta kappaleesta liikkuu kylmempään kappaleeseen. Termodynamiikan lakien perusteella lämpö siirtyy itsekseen aina lämpimämmästä kappaleesta kylmemmän kappaleeseen. Jos siis kuuma kappale on kylmän kappaleen ympäröimänä niin lämmön nettosiirto tapahtuu aina kuumasta kylmään ympäristöön, ellei siihen vaikuteta ulkoisella työllä kuten jääkaapissa. Kun kappaleet vuorovaikuttavat vain lämpösäteilyn välityksellä niin lämmön nettomääräisen siirtymisen teho on kappaleen lähettämän ja vastaanottaman lämpösäteilyn tehojen erotus.

      Pitää ymmärtää, että nuo kaksi käsitettä tosiaan ovat toisistaan erillisiä, Jokainen kappale sekä vastaanottaa että lähettää lämpösäteilyä. Lähetetyn lämpösäteilyn teho riippuu kappaleen omasta lämpötilasta ja vastaanotetun lämpösäteilyn teho kappaleen ympäristön lämpötilasta. Ympäristön lämpötilan muutos muuttaa ympäristön lähettämän lämpösäteilyn tehoon. Siten se vaikuttaa kappaleeseen absorboituvan lämpösäteilyn tehoon. Ympäristön lämpötila ei muuta kappaleen lähettämän lämpösäteilyn tehoa mutta vaikuttaa lämmön nettomääräisen siirtymisen tehoon.

      Lämmön nettomääräisen siirtymisen teho on lähetetyn ja vastaanotetun lämpösäteilyn tehon erotus. Kun kappaleen ja sen ympäristön lämpötila on sama niin sekä ympäristö että kappale edelleenkin lähettävät ja vastaanottavat lämpösäteilyä. Jos kappale ja sen ympäristö ovat samassa lämpötilassa niin kappale sekä lähettää että vastaanottaa yhtä paljon tehoa lämpösäteilynä, jolloin lämmön nettomääräisen siirron teho on nolla kuten kuuluukin.

      • totuudenetsijä

        Muuten tuo sinun esityksesi tuntuu uskottavalta, mutta säteileekö sitä lämpöä muka edestakaisin silloin, kun kappaleen ja ympäristön lämpötila on sama. Muutenkin minun kokemukseni on kyllä se, että lämpö säteilee aina lämpimästä kylmään, ja sitä kokemusta minulla on todella paljon.

        Muuten tuo esitys sopi tälle palstalle aivan hyvin, koska kyllä ilmankin ominaisuuksia voidaan käsitellä kuin se olisi kappale. Se on vaan erittäin harva kappale, joka kaiken lisäksi on jatkuvassa liikkeessä.

        Sen liikkeen ja sen merkityksen ymmärtäminen onkin tärkeää, kun puhutaan ilmakehästä. Se on paljon tärkeämpi kuin lämmön säteily. Ilma absorboi auringosta tulevan energian, mutta ei pysty sitä säteilemään pois, ennenkuin ilma joutuu kylmän kanssa tekemisiin.

        Kyllä sen auringosta tulevan energian absorboi koko ilma, sanoo vanhat teoriat mitä tahansa. Ei niihin uskota sellaisilla aloilla, joilla käsitellään kaasuja. Ei kaikki silti niilläkään aloilla välttämättä tiedä totuutta, mutta korkeimmin kolulutetut kyllä tietää. Se on heidän velvollisuutensa.

        Ne jotka kyseenalaistaa ilmasto-opin on järkiään metallin insinöörejä. Huuhaatietäjät ei vaan millään suostu heitä uskomaan. Kyllä metallin diplomi-insinöörit todellakin tietää kaasuista sellaisia ominaisuuksia, joita ilmastotiede ei tiedä, kun kieltäytyy ottamasta tietoa vastaan.

        Ilmastotiede on kummallinen tieteenala. Se on eristäytynyt omaan nurkkaansa eikä suostu soveltamaan muiden alojen tutkimustuloksia ja havaintoja. Onpahan niitä nyt muitakin sellaisia tieteeksi nimitettyjä aloja, jotka torjuu poikkitieteellisyyden vaikka poikkitieteellisyys on kaikista hedelmällisin asia maailmassa.

        Esimerkiksi valtaosa lääkletieteen keksinnöistä olisi jäänyt syntymättä ilman poikkitieteellisyyttä. Lääketiede sitä poikkitieteellisyyttä on kaikista eniten hyödyntänyt. Lääketiede on hyödyntänyt lähes kaikkia muita tieteitä, kun tutkimusmenetelmiä on kehitetty.

        Miksi ihmeessä ilmastotiede ei tee samoin. Ilmastoltiede kyllä väittää hyödyntävänsä fysiikkaa, mutta kun tieto on ristiriidasa muiden fysiikka hyödyntävien alojen tiedon kanssa, niin ei väite voi pitää paikkaansa.

        Niillä muilla aloilla se fysiikka toimii täsmälleen niin kuin väitetään. Ilmastotieteellä taas on pelkästään epämääräisiä ennusteita,jotka kaikki on menneet pieleen. Mielestäni se jo todistaa, että ilmastotiede väärinkäyttää fysiikkaa. tai sen väittämät ei lainkaan ole fysiikkaa.

        Kaikki minkä fysiikka on loppuun asti selvittänyt on hyvin täsmällistä. Sen lisäksi on sitten pilvin pimein kaikenlaisia teorioita, joista ei kukaan voi varmuudella sanoa, onko ne totta vai ei. Fysiikan tutkimus on nykyään edennyt siihen rajapintan. ja jatkuvasti sitä rajaa pyritään siirtämään.

        Aineiden kyky absorboida lämpöä on kuitenkin sellainen asia, jonka fyysikot on hyvin tarkkaan selvittäneet jo kauan sitten. Nykyään se on teollisuuden käyttämää faktaa. Lukemat on kilojouleina painokiloa kohti. Onhan kaasuiilakin massa, joka voidaan punnita.

        Tiedot on vähintään kahden desimaalin tarkkuudella teollisuuden käyttämissä taulukoissa. Sellaista on todellinen tieteellinen tieto. Se on harvoin kokonaisluku. Jos on kokonaislukujen haarukka se ei ole tieteellistä faktaa. Se on jonku heittämä, arvio, miten asia voisi olla.

        Kaikki ilmastotieteen tieto on 1800-luvulla sovittuja arvioita, jotka todellinen tiede on todistanut virheellisiksi jo kauan sitten. Ei asia ole sillä parantunut, että päälle on peitteeksi kasattu yhä lisää todistamattomia teorioita. Teollisuudessa tiedetään täsmällinen totuus.

        Ei eollisuudessa voisi mikään onnistua, jos siellä luotettaisiin epämääräiseen tietoon. Minun pomoni kyllä luotti epämääräiseen väittämään raaka-aineen ominaisuuksista, ja kyllä se tuotti hirveästi ongelmia.

        Automaattikoneella kun on erittäin tärkeää, että raaka-aine on kunnollista ja vakiolaatua. Nopeudet sillä paskaraudalla joutui pudottamaan jopa kolmasosaan laatuteräkseen verrattuna. Lisäksi oli runsain mitoin muita ongelmia, joita ei ollut laatuteräksellä'.

        Tuli firmalle todella kalliikiiksi halvan raka-aineen käyttäminen. Työhön meni 2-3 kertainen aika. Kaikki kunnolliset teräslaadut on kehitetty länsimaissa 1900-luvulla. Kiinassa tai Intiassa sitä 1900-luvun tietoa ei käytetä.

        Ilmastotieteen touhu on samanlaista menneessä rämpimistä. 1800-luvun tiedolla yritetään saada tulosta aikaan, mutta eihän se onnistu. Maailma kuitenkin uskoo, että ilmastotiedekin elää 2000 lukua niikuin muutkin tieteet. Sen takia ilmastotieteeseen uskotaan.

        Vaikka edes säätä ei pystytä ennustamaan kuin viikon päähän, niin uskotellaan, että ilmastoa,. joka on tuhat kertaa monimutkasempi juttu, pystytään muka ennustamaan sadan vuoden päähän. Ei muuten onnistu sanon minä ja moni moni muukin.


    • lämmön.nettosiirron.teho

      Tässä yksinkertainen esimerkki lämmön nettomääräisen siirron ja lämpösäteilyn tehon oleellisesta erosta.

      Oletetaan yksinkertainen tilanne, jossa kappaleet PALLO ja KUORI ovat mustia eli molemmilla on emissiviteetti ε=1. Oletetaan lisäksi että kuumempi kappale PALLO, pinta-ala A ja lämpötila T1 on kokonaan kylmemmän kappaleen KUORI, lämpötila T2 ympäröimä. Katsotaan tilannetta kuumemman kappaleen PALLO kannalta.

      -Kuumempi kappale PALLO lähettää ympäristöönsä lämpöä lämpösäteilynä teholla P_ulos.
      -Kuumempi kappale PALLO vastaanottaa ympäristöstään KUORI-kappaleen lähettämää lämpöä lämpösäteilynä teholla P_sisään.
      -Lämmön nettosiirto on kuumemmasta PALLO kappaleesta kylmempään KUORI kappaleeseen päin ja sen suuruus on P_ulos-P_sisään.

      Stefan-Boltzmannin lain mukaan tässä tilanteessa (T^4 = T*T*T*T eli neljäs potenssi)

      P_ulos= vakio*A*(T1)^4
      P_sisään= vakio*A*(T2)^4

      Lämmön nettosiirron teho kuumemmasta kappaleesta PALLO kylmempään kappaleeseen KUORI P_netto on siis kirjoitettavissa tähän muotoon:

      P_netto = P_ulos - P_sisään = vakio*A*( (T1)^4 - (T2)^4 )

      Tuo on kaikille lämpösäteilylaskuja tehneille varsin tuttu kaava. Kappale menettää ympäristöönsä lämpöä (lämmön nettosiirto) teholla, jonka suuruus on vakio kertaa kappaleen näkyvä ulkopinta-ala kertaa kappaleen lämpötilan neljännen potenssin ja ympäristön lämpötilan neljännen potenssin erotus. Vakiossa on mukana emissiivisyys ja Stefan-Boltzmannin vakio.

      Kun kaksi eri lämpötilassa olevaa kappaletta vuorovaikuttaa keskenään lämpösäteilyn välityksellä niin lämmön nettosiirto on aina kuumemmasta kappaleesta kylmempään. Kuten jo kerrottu kylmemmän kappaleen lämpötila ei vaikuta kuumemman kappaleen lähettämän lämpösäteilyn tehoon mutta se vaikuttaa kuumemman kappaleen kylmemmästä kappaleesta vastaanottamaan lämpösäteilyn tehoon.

      Kuumempaa kappaletta ympäröivän kylmemmän kappaleen lämpötila siis vaikuttaa siihen, mikä on lämmön nettosiirron teho kuumemmasta kappaleesta kylmempään kappaleeseen. Jos kuumaa kappaletta ympäröivän kylmemmän kappaleen lämpötila nousee niin lämmön nettosiirron teho kuumemmasta kappaleesta kylmempään kappaleeseen vastaavasti pienenee. Kun ympäristön lämpötila saavuttaa kappaleen lämpötilan niin lämmön nettosiirron teho menee yllä olevan kaavan perusteella nollaksi.

      Vaikka ympäristön ja kappaleen lämpötilat olisivat yhtäsuuret ei lämpösäteilyn teho yhtäkkiä häviäisi mihinkään. Tasalämpöisen (20ºC eli 293K) laatikon sisällä tilan kaikki sisäpinnat koko ajan lähettävät laatikon sisätilaan lämpösäteilyä luokkaa satojen wattien teholla per neliömetri. Kun kyseiset pinnat samanaikaisesti myös vastaanottavat lämpösäteilyä samalla teholla niin lämmön nettosiirto pinnoista laatikkoon pysyy nollana eli seinät eivät jäähdy eivätkä lämpene.

      Maapallon ja sitä ympäröivän ilmakehän tarkastelu on tietenkin paljon monimutkaisempi tilanne mutta tuossa yllä on siis alkeiden perusteet muutamien lämpösäteilyyn liittyvien ilmiöiden ymmärtämiselle keskusteluissa ilmastonmuutoksesta.

      • totuudenetsijä

        Mitä sinä tuolla hämärällä monmutkasella selostuksella yrität tpdistaa. Kun minä tuoolaista luen minulla herää epäilys, että mustaa yritetään maalata valkoiseksi. Yritätkö sinä tuolla todistaa jotakin ilmastonmuutoksesta. Ilmasto on kyllä paljon simppelimpi ja yksnkertaisempi juttu.

        Yritätätkö sinä tuolla todistaa, että kylmä maanpinta pystyy lämmittämään sen yllä olevaa ilmaa joka on lämpimämpi kuin maan pinta. Täällä on esitetty vaikka mitä hämäriä teorioita, joilla on yritetty selittää sellaista, mitä ei oikeasti ole olemassa.

        Pitäisiköhän minun lähettää linkki tekniikan tohtorille joka on lampöopin erityiasiantuntija. Hän pystyisi kyllä varmaan sanomaan, onko tuossa yhtään mitään järkeä. Minun käytännön kokemukseni mukaan lämpö siirtyy aina vain yhteen suuntaan lämpimästä kylmään.

        Veikkaisin,, että tohtorilla on aivan sama käytännön kokemus. Hän korosti käytännön kokemuksen merkitystä. Se vasta todistaa miten asiat oikeasti on. Minä olen useastikin jäähdyttänyt kuuman kappaleen laittamalla sen kylmän kappaleen päälle.

        Lämpö johtuu siitä kylmään kappaleeseen ja absorbopituu siitä sitten ilmaan. Käytännön lämpöoppi on minulle hyvin tuttu juttu, mutta tuosta sinun jutusta en pusty sanomaan onko siinä edes lämpöopista kysymys.

        En viitsi alkaa paremmin perehtymään asiaan, koska mielestäni tuolla ei ole mitään merkitystä ilmastossa. Ilmakehän toiminta on todellisuudessa aika yksinkertainen juttu..Ei siinä tuollaisia kaavoja tarvita.

        Tänne on monta kertaa linkitettu kuva, jonka väitetään todistavan kuinka mikäkin absorboi lämpöä. Minun mielestäni siinä kuvassa on se, että kuinka paljon auringosta mitäkin aaltopituutta tulee


      • lämmön.nettosiirron.teho

        Ilmastonmuutoskeskusteluissa tilanne on hieman toisenlainen kuin tuossa ylempänä esitetty. Kuuman kappaleen lämpötila ei olekaan mikään ENNALTA ASETETTU vakioarvo vaan se määräytyy kuumaan kappaleeseen tuodun energian ja kuuman kappaleen ympäristöönsä menettämän energian tasapainotilanteesta. Kylmemmän ympäristön lämpötilan muutos voi siis vaikuttaa kuuman kappaleen lämpötilaan.

        Jatketaan yksinkertaista esimerkkiä tuomalla siihen tuo uusi piirre. Oletetaan, että meillä on lämpötilassa T1 oleva kuuma kappale PALLO, jota ympäröi kylmempi kappale KUORI, lämpötilassa T2. Kappaleeseen PALLO tuodaan koko ajan lisää lämpöä ulkoapäin teholla P_lämmitys ja se menettää koko ajan lämpöä lämpösäteilyn kautta lämmön nettosiirron teholla P_netto. Ollaan tasapainotilassa eli T1 ei enää muutu.

        Jos PALLO on termisessä tasapainotilassa eli sen lämpötila ei nouse eikä laske niin silloin PALLOon tuleva lämpöteho P_lämmitys on oltava sama kuin pallosta poistuva lämmön nettosiirron teho P_netto. Kun lämpösäteilystä edellä käsitellyn esimerkin perusteella tiedetään mikä P_netto on voidaan kirjoittaa tasapainoehto näin:

        P_lämmitys = vakio*A*((T1)^4 - (T2)^4))
        missä
        P_lämmitys = palloon ulkoa tulevan lämmitystehon suuruus
        T1 = pallon lämpötila,
        T2 = ympäristön lämpötila
        A = pallon pinta-ala
        vakio = vakiotermi, jossa on mukana emissiivisyydet ja Stefan-Boltzmannin vakio.

        Mitä tapahtuu kappaleen PALLO lämpötilalle T1, jos sen ympäristön lämpötila T2 muuttuu? Ratkaisemalla tasapainoehdosta saadaan lämpötilalle T1 ratkaisu:

        T1 = (P_lämmitys/(vakio*A) (T2)^4 )^0.25

        Tuosta nähdään helposti, että PALLOa ympäröivän KUORIn lämpötilan T2 nostaminen eli PALLOa kohti sen ympäristöstä lähtevän lämpösäteilyn tehon nousu aiheuttaa PALLOn saavuttaman tasapainolämpötilan nousun, vaikka PALLOon ulkopäin tuleva lämmitysteho P_lämmitys pysyykin koko ajan vakiona.

        Edellä esitetty oli siis yliopisto- tai ammattikorkeakoulutason fysiikan peruskurssien asiaa.


    • uitunvähän

      Joo ja hiilidioksidia on ilmasta 0,04%

    • Naurettavin denialistiväite on, että "hiilidioksidia on niin vähän".

      Hiilidioksidi on lisääntynyt ilmakehässä 43 prosenttia viimeisten 60:n vuoden aikana. Se on paljon, kun kyseessä on erittäin tehokas kasvihuonekaasu, joka absorboi sellaisilla aallonpituuksilla, joilla muut kaasut eivät absorboi.

      • Kerro-lisää

        Kerrot samaa mantraa toistuvasti, josta ilmeisesti et tiedä mitään.

        Kerro se tai ne aaltoalueet, joita hiilidioksidi yksin absorboi, jos niitä on, tai lopeta väärän tiedon jakaminen.


      • jojojojojojojojoo

        Edelleenkin liian vähän vaikuttaakseen enempää kuin joitakin asteen C kymmenyksiä.


      • näin.se.on
        jojojojojojojojoo kirjoitti:

        Edelleenkin liian vähän vaikuttaakseen enempää kuin joitakin asteen C kymmenyksiä.

        Ja sekin on jäähdyttävä vaikutus, kuten muillakin kasvihuonekaasuilla.


      • Anonyymi

        Sitä kyllä normaalisti pidetään harhanajohtavana käyttää prosentteja silloin, kun on kyse kokonaisuuteen suhteessa pienistä luvuista.

        Otetaampa esimerkki, sanotaan vaikka, että jossakin maassa X on 50 000 000 asukasta. Ensimmäisenä vuonna yksi ihminen kuolee käärmeenpuremaan. Seuraavana vuonna kolme.

        Nyt tästä saadaan otsikko, että "Käärmeenpuremaan kuolleiden määrä kasvoi 200%".

        Tämä kertoo ongelman luonteesta paljon vähemmän kuin se, että kerrotaan luvut suoraan.

        Tämä näin sivuhuomautuksena, mitenkään ottamatta kantaan siihen onko 400 ppm paljon vai vähän.


      • Anonyymi
        Anonyymi kirjoitti:

        Sitä kyllä normaalisti pidetään harhanajohtavana käyttää prosentteja silloin, kun on kyse kokonaisuuteen suhteessa pienistä luvuista.

        Otetaampa esimerkki, sanotaan vaikka, että jossakin maassa X on 50 000 000 asukasta. Ensimmäisenä vuonna yksi ihminen kuolee käärmeenpuremaan. Seuraavana vuonna kolme.

        Nyt tästä saadaan otsikko, että "Käärmeenpuremaan kuolleiden määrä kasvoi 200%".

        Tämä kertoo ongelman luonteesta paljon vähemmän kuin se, että kerrotaan luvut suoraan.

        Tämä näin sivuhuomautuksena, mitenkään ottamatta kantaan siihen onko 400 ppm paljon vai vähän.

        Ilmakehän jonkun kaasun määrän muutoksilla on merkitystä silloin, kun muutokset ovat vaikutuksiltaan havaittavissa. Sillä ei sinänsä ole väliä onko silloin kaasun absoluuttinen määrä prosentteja vai miljoonasosia. Hiilidioksidin osalta tuo vaikutukset näkyvät hyvin 300ppm paikkeilla ja muutoksen vaikutukset ovat selvästi mitattavissa.

        Tämä keskustelu argumentein ei-se-voi-olla-merkityksellinen-kun-sitä-on-prosentteina-niin-vähän on jo käyty moneen kertaan ja siihen vastattukin moneen kertaan.


      • Anonyymi

        Hiilidioksidia on maapallon ilmakehässä niin vähän, että sen merkitys ilmastoon on jotakuinkin olematon. Vesihöyryä on ilmakehässä merkittävästi enemmän kuin hiilidioksidia, mutta sen määrä vaihtelee suuresti.

        Hiilidioksidi on varsin heikko kasvihuonekaasu. Kaasujen kykyä lämmittää ilmastoa mitataan GWP-arvolla (global warming potential). Hiilidioksidin GWP on 1 kaikille aikahorisonteille. Esimerkkejä muista kaasuista:
        Metaani: GWP/20v= 82 GWP/100v= 28
        Typpioksidi: GWP/20v=264 GWP/100v=265
        CFC-12 GWP/20v=10800 GWP/100v=10200 (ponnekaasu)
        Rikkiheksafluoridi: GWP/20v =17500 GWP/100v =23500
        Eli jos rikkiheksafluoridia olisi ilmakehässä sama määrä kuin hiilidioksidia niin sadassa vuodessa se lämmittäisi ilmastoa 23500 kertaa tehokkaammin kuin hiilidioksidi (!!). Onneksi rikkiheksafluoridi on keinotekoinen kaasu, jota ei luonnossa esiinny.


      • Hiilidioksidia on maapallon ilmakehässä naurettavan vähän. Siitä sanotaan tiedepiireissä, että se on "trace gas", suomeksi voisi sanoa vaikka että hitukaasu, sitä on häviävän pieni määrä. Meidän kaikkien onneksi sitä on ilmakehässä sentään sen verran, että aikojen kuluessa erittäin tehokkaiksi yhteyttäjiksi kehittyneet kasvit ja bakteerit tulevat tuolla naurettavan pienellä määrällä toimeen. Se on mahdollista koska hiilidioksidi kiertää elollisessa luonnossa ja ilmakehässä, se on tärkeä elämää ylläpitävä kaasu.

        Hiilidioksidi on todella heikko kasvihuonekaasu, sen heikompaa en ainakaan tiedä. Jos kaasu on hiilidioksidia heikompi ip:n absorboija niin se ei absorboi infrapunaa lainkaan eikä ole kasvihuonekaasu. Hiilidioksidia monin verroin tärkeämpi kasvihuonekaasu maapallolla on vesihöyry. Aallonpituudet, joilla CO2 ja H2O absorboivat menevät osin päällekkäin.


      • Anonyymi

        Hiilidioksidi on kaikkein heikoin ip-sätäilyn absorboija niistä kaasuista, jotka ylipäänsä luokitellaan kasvihuonekaasuiksi.


      • Anonyymi
        Anonyymi kirjoitti:

        Hiilidioksidi on kaikkein heikoin ip-sätäilyn absorboija niistä kaasuista, jotka ylipäänsä luokitellaan kasvihuonekaasuiksi.

        Sadalle vuodelle laskettu vesihöyryn GWP on luokkaa 0.001 ... 0.0005 kun hiilidioksidille se on 1 ja esimerkiksi metaanille 25. GPW = g­l­o­b­a­l­ w­a­r­m­i­n­g p­o­t­e­n­t­i­a­l

        https://iopscience.iop.org/article/10.1088/1748-9326/aae018


    • Aallonpituusalueella 13-16 mikrometriä absorboi lämpöä ainoastaan hiilidioksidi.

      Aallonpituusalue 16-18 mikrometriä on jonkin verran päällekkäin vesihöyryn kanssa, mutta silläkään alueella vesihöyry ei absorboi, jos sitä ei ilmassa ole.

      • Tarkista-tietosi

        Ilmoittamasi aaltoaluetta hiilidioksidi kylläkin absorboi, kuten vesihöyrykin.

        Valotan sen verran että harmaassa yläpalkissa olevat valkeat alueet on aallonpituuksia, jotka pääsevät läpi, eli ei absorboidu ilmakehään.
        AGW.n mukaan hiilidioksidin pitäisi absorboida juuri niitä taajuuksia.
        Alempana on eri ilmakehän kaasujen absorptioalueet, ja kuten huomaat co2 ei satu niille alueille ja lisäksi ilmoittamasi aaltoalueet absorboituvat 100 % sesti, joten co2.n lisäys ei sitä voi enempää absorboida.

        https://chriscolose.files.wordpress.com/2010/02/atmospheric_transmission.png

        Huomaa lisäksi, kuinka suuri vaikutus on vesihöyryllä, laaja aaltoalue ja 100 % absorbointi, koska määrä on aivan eri kertaluokkaa kuin hiilidioksidilla.


      • Aavikko-tietää
        Tarkista-tietosi kirjoitti:

        Ilmoittamasi aaltoaluetta hiilidioksidi kylläkin absorboi, kuten vesihöyrykin.

        Valotan sen verran että harmaassa yläpalkissa olevat valkeat alueet on aallonpituuksia, jotka pääsevät läpi, eli ei absorboidu ilmakehään.
        AGW.n mukaan hiilidioksidin pitäisi absorboida juuri niitä taajuuksia.
        Alempana on eri ilmakehän kaasujen absorptioalueet, ja kuten huomaat co2 ei satu niille alueille ja lisäksi ilmoittamasi aaltoalueet absorboituvat 100 % sesti, joten co2.n lisäys ei sitä voi enempää absorboida.

        https://chriscolose.files.wordpress.com/2010/02/atmospheric_transmission.png

        Huomaa lisäksi, kuinka suuri vaikutus on vesihöyryllä, laaja aaltoalue ja 100 % absorbointi, koska määrä on aivan eri kertaluokkaa kuin hiilidioksidilla.

        Vesihöyry on todellakin tehokkain kasvihuonekaasu, mutta se myös estää auringon säteilyä tulemasta maahan. Se lähinnä tasaa päivä ja yölämpötilojen eroja, kuten kaikki tietää.

        Jos aavikolla yöpyy niin mitä merkitsee se epäilty 0,2-0,4 neljän C asteen hiilen avulla lämpeneminen käytännössä?


      • kokokokokokoko
        Aavikko-tietää kirjoitti:

        Vesihöyry on todellakin tehokkain kasvihuonekaasu, mutta se myös estää auringon säteilyä tulemasta maahan. Se lähinnä tasaa päivä ja yölämpötilojen eroja, kuten kaikki tietää.

        Jos aavikolla yöpyy niin mitä merkitsee se epäilty 0,2-0,4 neljän C asteen hiilen avulla lämpeneminen käytännössä?

        Se menee mittaustarkkuuden piikkiin ja edelleenkin on helvetin kylmä


      • Anonyymi
        Tarkista-tietosi kirjoitti:

        Ilmoittamasi aaltoaluetta hiilidioksidi kylläkin absorboi, kuten vesihöyrykin.

        Valotan sen verran että harmaassa yläpalkissa olevat valkeat alueet on aallonpituuksia, jotka pääsevät läpi, eli ei absorboidu ilmakehään.
        AGW.n mukaan hiilidioksidin pitäisi absorboida juuri niitä taajuuksia.
        Alempana on eri ilmakehän kaasujen absorptioalueet, ja kuten huomaat co2 ei satu niille alueille ja lisäksi ilmoittamasi aaltoalueet absorboituvat 100 % sesti, joten co2.n lisäys ei sitä voi enempää absorboida.

        https://chriscolose.files.wordpress.com/2010/02/atmospheric_transmission.png

        Huomaa lisäksi, kuinka suuri vaikutus on vesihöyryllä, laaja aaltoalue ja 100 % absorbointi, koska määrä on aivan eri kertaluokkaa kuin hiilidioksidilla.

        Tässä se sitten tuli taasen järkevä selitys asialle jota AGW ei ota huomioon.
        Kiitos siitä.


    • Yleensä vesihöyryn absorptiokykyä liioitellaan, koska sen absorptiospektri on laaja. Kuitenkin vesihöyryn määrä ilmakehässsä vaihtelee suuresti. ( Lämpeneminen lisää vesihöyryn määrää, mikä on otettu huomioon tiedemiesten laskelmissa.)

      • jäätävääkeliä

        Jauhat taas pastaa jönssi. Vesihöyryä on ilmassa noin kymmen kertaa enemmän kuin hiilidioksidia.


    • Ilman hiilidioksidia vesihöyryä ei olisi ilmassa ollenkaan. Kaikki vesi olisi jäässä.

      • jäätävääkeliä

        Jos ei olisi vettä ja vesihöyryä niin ei olisi hiilidioksidia eikä jönssiäkään kirjoittamassa noita aivopieruja.


      • Ilmastofriikin-muutos

        Jönssin viimeaikaisissa kirjoituksissa on selvä muutos !
        Poissa on aikaisempi uho ja ylimielinen itsevarmuus, lisättynä aina samoilla IPCC - lätkämaila linkeillä ja muita halveksuvalla tekstillä.
        Nyt on 'ääni hiljentynyt', viestit ovat kuin pikkupojan ujoja yrityksiä saada edes hieman myötätuntoa ajatuksilleen, viimeisen sanan saaminenkin on jäänyt pois esiintymistavoitteista ja väitteitä ei enää edes yritetä vakuuttaa toivottomilla selityksillä ja denialisti-heitoilla.

        Kaikki tuo edellä mainittu saattaa tarkoittaa, että ajan kanssa vahva usko on alkanut horjua, kun on lukenut faktoja, nähnyt ennusteiden epäonnistumiset ja toivottavasti alkanut itsekin miettiä, sokean uskonsa todellisuutta ja järjettömyyttä.

        Tämä muutos on ilahduttavaa, sillä sanotaanhan jossain 'teoksessa' että kannattaa yrittää, jos yksikin ? (mikä se nyt olikaan) saadaan pelastetuksi.


      • co2.ei.lämmitä

        Ei muutosta havaittavissa. Sekaisin tuo jönkööping on kuin ennenkin.
        Olen kysynyt monesti, että millä perusteella hiilidioksidin puuttuminen jäädyttäisi kaikki vedet, mutta en ole saanut vastausta.
        Mutta se on totta, että ilman kasvihuonekaasuja täällä olisi paljon lämpöisempää.


      • huolipois
        Ilmastofriikin-muutos kirjoitti:

        Jönssin viimeaikaisissa kirjoituksissa on selvä muutos !
        Poissa on aikaisempi uho ja ylimielinen itsevarmuus, lisättynä aina samoilla IPCC - lätkämaila linkeillä ja muita halveksuvalla tekstillä.
        Nyt on 'ääni hiljentynyt', viestit ovat kuin pikkupojan ujoja yrityksiä saada edes hieman myötätuntoa ajatuksilleen, viimeisen sanan saaminenkin on jäänyt pois esiintymistavoitteista ja väitteitä ei enää edes yritetä vakuuttaa toivottomilla selityksillä ja denialisti-heitoilla.

        Kaikki tuo edellä mainittu saattaa tarkoittaa, että ajan kanssa vahva usko on alkanut horjua, kun on lukenut faktoja, nähnyt ennusteiden epäonnistumiset ja toivottavasti alkanut itsekin miettiä, sokean uskonsa todellisuutta ja järjettömyyttä.

        Tämä muutos on ilahduttavaa, sillä sanotaanhan jossain 'teoksessa' että kannattaa yrittää, jos yksikin ? (mikä se nyt olikaan) saadaan pelastetuksi.

        Ei jönssille vieläkään kuitenkaan mene jakeluus minkä takia ennusteet on menneet pieleen. Siihen on itsestää selvä selitys, jonka tietää metallin insinöörit ja kaasuteollisuuden insinöörit. Eiköhän se selitys ole öljyteollisuudenkin tiedossa, kun sekin nesteyttää kaasuja.

        Tärkein syy, miksi ennusteet menee pieleen, on se, että ne ilmastotieteen väittämät ilmakehän kaasuista ei pidä paikkaansa Ns kasvikhuonekaasut ei ole vahvoja kasvihuonekaasuja ja niitä on ilmakehässä erittäin vähän.

        Vesihöyry on kohtalaisen vahva, mutta sitäkin on ilmakehässä aika vähän ja sen pitoisuus vaihtelee. Mikä siis aiheuttaa kasvihuoneilmiön, kun ns. kasvihuonekaasut on heikkoja ja vesihöyrylläkään ei ole paljon merkitystä. Selitys löytyy kansainvälisestä SI standardista, jota teollisuus käyttää.

        Sieltä paljastuu, kuinka paljon happi ja typpi absorboi sitä IR säteilyä suhteessa muihin kaasuihin ja niitähän on ilmakehässä ylivoimaisesti eniten. Olen näitä asioita tuonut esille, jotta ymmärrettäisiin, miksi ennusteet menee pieleen.

        Ennusteet perustuu ns. kasvihuonekaasujen väitettyihin ominaisuuksiin ja väitettyihin osuuksiin kasvihuoneilmiön aiheuttajana. Kun niiden ja vesihöyryn osuus SI standardin mukaan onkin alle prosentti kasvihuoneilmiön aiheuttajana, niin tottakait ennusteet menee pieleen.

        Kyllä insinöörit jotka suunnittelee monimutkaisia kokonaisuuksia pystyy ymmärtämään myös ilmakehän toimintaa siinä kuin ilmastotiedekin. Se peruskaava, jonka mukaan ilmakehä toimii on loppujen jopuksi aika yksinkertainen, kun karsii tarpeettomat rönsyt pois.

        Sen ymmärtämiseen tarvitaan välttämättä SI järjestelmän tiedon ymmärtämistä ja käyttämistä. Se kääntää kaiken päälaelleen. Ilmastotieteen alkuperäinen oletus on se että ilmakehä sitoo auringosta tulevaa energiaa maan päälle.

        Se ei kuitenkaan pidä paikkaansa, mutta sen ymmärryksen mukaan eletään edelleen ja yritetään mallintaa ilmastoa ja ennujstetaan ilmastonmuutosta. Topdellisuudessa ilmakehä sitoo auringon energiaa itseensä. Sehän taas on aivan eri asia kuin ilmasotieteen oletus.

        Ilmakehä sitoo auringon energiaa itseensä, ja poistaa sitä maasta, johon sitä tulee joka päivä valtavasti lisää. Ilmastotieteen olettama oli siis täysin väärä. Siihen kuitenkin edeelleen perustuu ne ennusteet.-

        Kun otetaan huomioon se, että ilmakehä nimenomaan poistaa auringosta tulevaa energiaa, kumoutuu teoria ihmisen CO2 päästöillä aiheuttamasta ilmastonmuutoksesta.

        Tässä todellisessa mallissa ei ole mitään merkitystä ilmakehän kaasukoostumuksella. Tämä poistaa myöskin tarpeen tehdä pitkän aikavälin ennusteita. Ne on tarpeettomia, koska niillä ei koskaan voi ennustaa mitään todellista. Ainoastaan sääennusteet on tarpeen.

        Koko ilmaston tulevaisuutta on mahdotonta ennustaa. Tämä johtuu siitä että kasvihuoneilmiöstä 99% aiheuttaa happi ja typpi. Sitä ilmastotieteen olettamaa muutosta ilmakehän absorbointikyvyn muutoksesta ei ole olemassa.

        Ns. kasvihuonekaasujen lisääntyminen muuttaa kyllä ilmakehän koknaiskapasiteettia, mutta muutos on erittäin pieni. Ei sillä ole merkitystä. Auringon toiminna kiihtyminen 1900-luvun jäjlipuoliskolla loi harhakuvan, että ennusteet ja niiden perusteet pitää paikkansa.

        Kun vuosituhat vaihtui, alkoi näyttää yhä ilmeisemmältä, että jotain on vialla. Toistaiseksi sen on huomanneet vain kriitikot. Muu maailma ei sitä suostu tunnustamaan. Jäisihän hyvät kauhujutut käyttämättä. Jotku on sairaalloisen mieltyneitä niihin.

        En minä tilannetta seurannut reaaliajassa. Vasta tänä vuonna olen tutustunut mitattuihin lämpötiloihin ja ennusteisiin. Tuli tarve kun viime vuonna huomasin kuinka hirveän harhaanjohtavaa on uutisointi.

        Päättelin, että ennenpitkää se tuottaa ihmisille sairauksia. Olen melkoinen psykosomatiikan asiantutija ollut jo vuosikymmeniä. Kun ihmisiltä riistetään turvallisuudentunne, yksi sun toinen alkaaa oireilola myös fyysisesti. Sairaudet, joita se tuottaa, on hyvin moninaiset, ja niiden taustoja voi olla jopa mahdotonta selvittää.


      • SivullinenWaan
        huolipois kirjoitti:

        Ei jönssille vieläkään kuitenkaan mene jakeluus minkä takia ennusteet on menneet pieleen. Siihen on itsestää selvä selitys, jonka tietää metallin insinöörit ja kaasuteollisuuden insinöörit. Eiköhän se selitys ole öljyteollisuudenkin tiedossa, kun sekin nesteyttää kaasuja.

        Tärkein syy, miksi ennusteet menee pieleen, on se, että ne ilmastotieteen väittämät ilmakehän kaasuista ei pidä paikkaansa Ns kasvikhuonekaasut ei ole vahvoja kasvihuonekaasuja ja niitä on ilmakehässä erittäin vähän.

        Vesihöyry on kohtalaisen vahva, mutta sitäkin on ilmakehässä aika vähän ja sen pitoisuus vaihtelee. Mikä siis aiheuttaa kasvihuoneilmiön, kun ns. kasvihuonekaasut on heikkoja ja vesihöyrylläkään ei ole paljon merkitystä. Selitys löytyy kansainvälisestä SI standardista, jota teollisuus käyttää.

        Sieltä paljastuu, kuinka paljon happi ja typpi absorboi sitä IR säteilyä suhteessa muihin kaasuihin ja niitähän on ilmakehässä ylivoimaisesti eniten. Olen näitä asioita tuonut esille, jotta ymmärrettäisiin, miksi ennusteet menee pieleen.

        Ennusteet perustuu ns. kasvihuonekaasujen väitettyihin ominaisuuksiin ja väitettyihin osuuksiin kasvihuoneilmiön aiheuttajana. Kun niiden ja vesihöyryn osuus SI standardin mukaan onkin alle prosentti kasvihuoneilmiön aiheuttajana, niin tottakait ennusteet menee pieleen.

        Kyllä insinöörit jotka suunnittelee monimutkaisia kokonaisuuksia pystyy ymmärtämään myös ilmakehän toimintaa siinä kuin ilmastotiedekin. Se peruskaava, jonka mukaan ilmakehä toimii on loppujen jopuksi aika yksinkertainen, kun karsii tarpeettomat rönsyt pois.

        Sen ymmärtämiseen tarvitaan välttämättä SI järjestelmän tiedon ymmärtämistä ja käyttämistä. Se kääntää kaiken päälaelleen. Ilmastotieteen alkuperäinen oletus on se että ilmakehä sitoo auringosta tulevaa energiaa maan päälle.

        Se ei kuitenkaan pidä paikkaansa, mutta sen ymmärryksen mukaan eletään edelleen ja yritetään mallintaa ilmastoa ja ennujstetaan ilmastonmuutosta. Topdellisuudessa ilmakehä sitoo auringon energiaa itseensä. Sehän taas on aivan eri asia kuin ilmasotieteen oletus.

        Ilmakehä sitoo auringon energiaa itseensä, ja poistaa sitä maasta, johon sitä tulee joka päivä valtavasti lisää. Ilmastotieteen olettama oli siis täysin väärä. Siihen kuitenkin edeelleen perustuu ne ennusteet.-

        Kun otetaan huomioon se, että ilmakehä nimenomaan poistaa auringosta tulevaa energiaa, kumoutuu teoria ihmisen CO2 päästöillä aiheuttamasta ilmastonmuutoksesta.

        Tässä todellisessa mallissa ei ole mitään merkitystä ilmakehän kaasukoostumuksella. Tämä poistaa myöskin tarpeen tehdä pitkän aikavälin ennusteita. Ne on tarpeettomia, koska niillä ei koskaan voi ennustaa mitään todellista. Ainoastaan sääennusteet on tarpeen.

        Koko ilmaston tulevaisuutta on mahdotonta ennustaa. Tämä johtuu siitä että kasvihuoneilmiöstä 99% aiheuttaa happi ja typpi. Sitä ilmastotieteen olettamaa muutosta ilmakehän absorbointikyvyn muutoksesta ei ole olemassa.

        Ns. kasvihuonekaasujen lisääntyminen muuttaa kyllä ilmakehän koknaiskapasiteettia, mutta muutos on erittäin pieni. Ei sillä ole merkitystä. Auringon toiminna kiihtyminen 1900-luvun jäjlipuoliskolla loi harhakuvan, että ennusteet ja niiden perusteet pitää paikkansa.

        Kun vuosituhat vaihtui, alkoi näyttää yhä ilmeisemmältä, että jotain on vialla. Toistaiseksi sen on huomanneet vain kriitikot. Muu maailma ei sitä suostu tunnustamaan. Jäisihän hyvät kauhujutut käyttämättä. Jotku on sairaalloisen mieltyneitä niihin.

        En minä tilannetta seurannut reaaliajassa. Vasta tänä vuonna olen tutustunut mitattuihin lämpötiloihin ja ennusteisiin. Tuli tarve kun viime vuonna huomasin kuinka hirveän harhaanjohtavaa on uutisointi.

        Päättelin, että ennenpitkää se tuottaa ihmisille sairauksia. Olen melkoinen psykosomatiikan asiantutija ollut jo vuosikymmeniä. Kun ihmisiltä riistetään turvallisuudentunne, yksi sun toinen alkaaa oireilola myös fyysisesti. Sairaudet, joita se tuottaa, on hyvin moninaiset, ja niiden taustoja voi olla jopa mahdotonta selvittää.

        Antaisin vihjeen että älä kirjoita novellia, jos vastaat johonkin kommenttiin, vain harvat viitsivät lukea ylipitkiä viestejä ja näin se ei tavoita kohdetta.

        Nasevampi kompakti vastaus riittää, suurin osa kykenee jonkin verran itsekin ajattelemaan ja liian yksityiskohtainen selostus tuntuu rasittavalta.


      • täyteroskaa
        SivullinenWaan kirjoitti:

        Antaisin vihjeen että älä kirjoita novellia, jos vastaat johonkin kommenttiin, vain harvat viitsivät lukea ylipitkiä viestejä ja näin se ei tavoita kohdetta.

        Nasevampi kompakti vastaus riittää, suurin osa kykenee jonkin verran itsekin ajattelemaan ja liian yksityiskohtainen selostus tuntuu rasittavalta.

        Sama kommentti. Kaasuinsinöörillä on joskus viestinsä ekassa kappaleessa yksi tai kaksi lausetta vastauksen aiheeseen liittyvää ja sen perässä sitten elämänkertaromaani, joka on sisällöltään yleensä sama kuin edellisissäkin viesteissä. Vähintään 95% viestiensä sisällöstä on siis pelkkää täytetekstiä.


      • SivullinenWaan
        täyteroskaa kirjoitti:

        Sama kommentti. Kaasuinsinöörillä on joskus viestinsä ekassa kappaleessa yksi tai kaksi lausetta vastauksen aiheeseen liittyvää ja sen perässä sitten elämänkertaromaani, joka on sisällöltään yleensä sama kuin edellisissäkin viesteissä. Vähintään 95% viestiensä sisällöstä on siis pelkkää täytetekstiä.

        Kiitos tuesta, mutta tekstistäsi puuttui lainausmerkit ammatin ympäriltä.
        Kyseinen kirjoittaja ei varmuudella ole insinööri.


      • huolipois
        täyteroskaa kirjoitti:

        Sama kommentti. Kaasuinsinöörillä on joskus viestinsä ekassa kappaleessa yksi tai kaksi lausetta vastauksen aiheeseen liittyvää ja sen perässä sitten elämänkertaromaani, joka on sisällöltään yleensä sama kuin edellisissäkin viesteissä. Vähintään 95% viestiensä sisällöstä on siis pelkkää täytetekstiä.

        Enpä minä kirjoitakaan teille joille lukeminen on tuskallista. Minä kirjoitan niille, joille kelpaa lukemiseksi vaikka romaani. Itse olen lukenut todella paljon, joten teksitiäkin tulee sen mukaisesti monista aiheista. Ilmeisesti et ymmärtänyt jutun ydintä

        Jutun ydin oli se, että on olemassa täysin pitävät todisteet siitä, että ilmastotiede on väärässä. Sen takia ennusteet on menneet pieleen, ja tulee menemään tulevaisuudessakin. Jatkuvalla kauhukuvien uutisoinnilla aiheutetaan vaan ihmisille sairauksia.

        Mitään hyötyä siitä ei ole, koska CO2 pitoisuus tulee joka tapauksessa nousemaan. Siitä pitää huolen kehittyvät maat. Meitä ahdistetaan aivan turhaan. Suomen CO2 päästöillä ei ole mitään merkitystä.

        Jos tänne ylläpitäjän mielestä joltakin tulee liikaa tekstiä, niin kyllä ne siihen puuttuu. Muiden mielipiteillä ei ole merkitystä. Pitkänkin keskustelun tsekkaa läpi minuutissa. Minä pyrin joka tapauksessa vaikuttamaan tähän yhteiskuntaan.

        Olen saanut aikaan kirjeitse jopa maailmanlaajuisia muutoksia. Olen vaikuttanut vuosikymmeniä. Tänä vuonna on nyt vuorossa suomi 24 sivusto ja tiedotusvälineet ja yliopistot. Kyllä se ennenpitää avautuu jostain kautta.

        On jo avautunutkin. Tännehän oli yksi yliopistoihminen kirjoittanut yliopiston kannan. Aika pitkä oli sekin juttu, Ne jotka todella osaa ja tietää kirjoittaa paljon. Kaasuinsinööri minä nyt en ole, mutta luotan mieluummin kaasuinsinööriin kuin ilmastotieteeseen.

        Samat lainalaisuudet pätee sekä laboratoriossa että ilmastossa. Uskon, että enemistö loppujen lopuksi mieluummin luottaa laboratorioon. Todisteita ja todistajia on todella paljon. Ihmiset täytyy vaan herättää pohtimaan, mitä ne todisteet todistaa. Siitä se miknullakin lähti liikkeelle.


      • täyteroskaa
        SivullinenWaan kirjoitti:

        Kiitos tuesta, mutta tekstistäsi puuttui lainausmerkit ammatin ympäriltä.
        Kyseinen kirjoittaja ei varmuudella ole insinööri.

        Tällä palstalla kaikki väitteet omasta osaamisesta ja koulutustaustasta ovat oletusarvoisesti epäuskottavia kun ei esiinnytä omilla nimillä. Siksi kaasuinsinööri ja "kaasuinsinööri" ovat minun mielestäni näissä keskusteluissa saman arvoisia nimilappuja.

        Annoit sille juuri hyvän perusteen vastata keskusteluun aiheeseen liittymättömällä elämänkertaviestillä... Itse ohitan nuo viestit lukien vain niiden ekan kappaleen.


      • huolipois
        täyteroskaa kirjoitti:

        Tällä palstalla kaikki väitteet omasta osaamisesta ja koulutustaustasta ovat oletusarvoisesti epäuskottavia kun ei esiinnytä omilla nimillä. Siksi kaasuinsinööri ja "kaasuinsinööri" ovat minun mielestäni näissä keskusteluissa saman arvoisia nimilappuja.

        Annoit sille juuri hyvän perusteen vastata keskusteluun aiheeseen liittymättömällä elämänkertaviestillä... Itse ohitan nuo viestit lukien vain niiden ekan kappaleen.

        Senkun ohitat. Jotku muut kuitenkin haluaa oikeasti faktatietoa. Ilmesesti sinulle kelpaa vain alarmistitieto. Tulevaisuus sitten näyttää, kuinka väärässä olit. Minä olen halunnut estää niitä suuria virheitä,joihin jolhtaa se, että uskotaan turhaan pelotteluun.

        Kun olen seurannut puolueiden kannatusmittauksia, niin aika moni on vaihtanut puolta viime kevään jälkeen. Uskoisin, että siihen on enemmänkin vaikuttanut palstalla saamani vastakaiku. Sivistyneet ihmiset on ymmärtäneet, ettei kannata kannattaa niitä, jotka ei osaa muuta kuin vittuilla ja pelotella.

        Fiksut ihmiset lukee aika paljon jo tavasta kirjolittaa, Yksi sellainen halusi 1990-luvulla välttämättä kirjoittamani tekstin esimerkiksi muille. Minä ihmettelin että miksi. En ollut omasta mielestäni tehnyt mitään erikoista. Kysymys oli vain yhdestä A4 sivusta.

        Se opettaja vastasi. ,että täytyy lukea todella paljon, pystyäkseen kirjoittamaan tuolla tavalla. Katsokaa kirjailijoiden kirjahyllyjä. Ne ei todellakaan olöe mikään koristehuonekalu. Minullakin on aika iso kirjasto. Toivottavasti SPR ottaa sen vastaan sitten kun minulta lähtee henki.


      • taas-aavikko
        co2.ei.lämmitä kirjoitti:

        Ei muutosta havaittavissa. Sekaisin tuo jönkööping on kuin ennenkin.
        Olen kysynyt monesti, että millä perusteella hiilidioksidin puuttuminen jäädyttäisi kaikki vedet, mutta en ole saanut vastausta.
        Mutta se on totta, että ilman kasvihuonekaasuja täällä olisi paljon lämpöisempää.

        Joo ilmasto olisi kuin kuussa: päivät erittäin kuumia ja yöt kylmiä. Vähän niin kuin aavikollakin ilman vesihöyryä.


      • kokokokokoko

        Ilman hiilidioksidia ei olisi elämää ja nyt kasvillisuus kiittää hieman lisääntyvästä hiilen määrästä ilmassa. Maailma vihertyy


      • eikiusatajönssiä
        Ilmastofriikin-muutos kirjoitti:

        Jönssin viimeaikaisissa kirjoituksissa on selvä muutos !
        Poissa on aikaisempi uho ja ylimielinen itsevarmuus, lisättynä aina samoilla IPCC - lätkämaila linkeillä ja muita halveksuvalla tekstillä.
        Nyt on 'ääni hiljentynyt', viestit ovat kuin pikkupojan ujoja yrityksiä saada edes hieman myötätuntoa ajatuksilleen, viimeisen sanan saaminenkin on jäänyt pois esiintymistavoitteista ja väitteitä ei enää edes yritetä vakuuttaa toivottomilla selityksillä ja denialisti-heitoilla.

        Kaikki tuo edellä mainittu saattaa tarkoittaa, että ajan kanssa vahva usko on alkanut horjua, kun on lukenut faktoja, nähnyt ennusteiden epäonnistumiset ja toivottavasti alkanut itsekin miettiä, sokean uskonsa todellisuutta ja järjettömyyttä.

        Tämä muutos on ilahduttavaa, sillä sanotaanhan jossain 'teoksessa' että kannattaa yrittää, jos yksikin ? (mikä se nyt olikaan) saadaan pelastetuksi.

        Jönssiä ei voi enää pelastaa vaan hän uskoisi vaikka tulisi nopea jääkausi.
        Sellaista on aivopestyn elämä. Sellaisia oli 70-luvulla stalinistit.


      • huolipois
        kokokokokoko kirjoitti:

        Ilman hiilidioksidia ei olisi elämää ja nyt kasvillisuus kiittää hieman lisääntyvästä hiilen määrästä ilmassa. Maailma vihertyy

        Asia on juuri noin. On sairasta, että maailma haluaa kieltää hiilidioksidin todistetusti positiivisen vaikutuksaen luonnossa.- On sairasta uskoa vain kauhukuviin, joiden toteutumisesta ei ole mitään muita todisteita kuin tietokonemallinnus, joka on hyvin tulkinnanavarainen.

        Kuvitteleeko ihmiset todella, että kauhukuviin uskominen tuo turvallisuutta. Ihminen on luonnostaan turvallisuushakuinen. Useimmat ihmiset on sitä. Miksi heitä ei oteta ollenkaan huomioon.

        Itse olen kyllä ottanut hyvinkin suuria riskejä aikanaan hälyytysajoneuvon ratissa, mutta tiedän että yleisesti ottaen ihmiset on turvallisuushakuisia. Heidätkin pitäisi ottaa hyomioon eikä pelotella turhilla uhkakuvilla.

        Minä olin se kova jätkä, joka ajoi keskikaistaa kaksikaistaisella tiellä. Välillä pujoteltiin niinkuin mutkamäessä.


      • älä-ruoki-trolleja

        Kun hieman mietit niin keksit useitakin syitä sille, että joku kirjoittaisi tuolla tavalla. Osa syistä on vapaaehtoisia ja harkittuja valintoja, osa taas sellaisia joihin kirjoittaja itse ei voi vaikuttaa vaikka haluaisikin.

        Mikään niistä mahdollisista syistä ei ole sellainen, että jatkuvasti vastoin toistuvasti saamiaan ohjeita ohi aiheen kirjoittavalle kannattaisi vastailla.


      • huolipois

        Jos sinä nyt oikeasti olet sairaankuljettajam, niin en minä sinulle kirjoittanutkaan. Minä kirjoitin niille, jotka täällä ,hevostelee, vaikkei yhtään mistään mitään tiedä. Sairaankuljettajan ammatissa minä olin 1980-1990 luvuilla.

        Silloin vasta aloiteltiin jatkokoulutusta. Minä kävin manuaali defibrillointi ja intubointi kurssin. Kunnioitan todella niitä sairaankuljettajia, jotka nykyään sitä työtä tekeen. Heille on opetettu paljon sellaista, mitä minulle ei koskaan opetettu.

        Kuitenkin minä opetin vaimolle, kuinka saa parhaiten onnistumaan tippakanyylin laittaminen, eikä hän uransa aikana siinä koskaan epäonnistunut. Minä opin sen lääkäriltä, joka oli ollut kehitysaputyössä. Häneltä se onnistui niin hyvin, että katsoin tarkkaan, miten hän sen teki.

        Kyllähän minä tiedän, että ikävään suomalaiseen kansanluonteeseen kuuluu, ettei saa kehua itseään. Minä teen sitä ihan piruuttani. Jos asuisin Amerikassa, voisin panna kaikki todistukseni kehystettynä seinälle, ja se nähtäisiin vain positiivisena.

        Amerikassa pelastajat on sankareita. Suomessa he joutuu nöyristelemään. Pidän kyllä enemmän Amerikkalaisesta mentaliteetista. Siellä saa olla oma itsemsä.


      • SivullinenWaan
        huolipois kirjoitti:

        Jos sinä nyt oikeasti olet sairaankuljettajam, niin en minä sinulle kirjoittanutkaan. Minä kirjoitin niille, jotka täällä ,hevostelee, vaikkei yhtään mistään mitään tiedä. Sairaankuljettajan ammatissa minä olin 1980-1990 luvuilla.

        Silloin vasta aloiteltiin jatkokoulutusta. Minä kävin manuaali defibrillointi ja intubointi kurssin. Kunnioitan todella niitä sairaankuljettajia, jotka nykyään sitä työtä tekeen. Heille on opetettu paljon sellaista, mitä minulle ei koskaan opetettu.

        Kuitenkin minä opetin vaimolle, kuinka saa parhaiten onnistumaan tippakanyylin laittaminen, eikä hän uransa aikana siinä koskaan epäonnistunut. Minä opin sen lääkäriltä, joka oli ollut kehitysaputyössä. Häneltä se onnistui niin hyvin, että katsoin tarkkaan, miten hän sen teki.

        Kyllähän minä tiedän, että ikävään suomalaiseen kansanluonteeseen kuuluu, ettei saa kehua itseään. Minä teen sitä ihan piruuttani. Jos asuisin Amerikassa, voisin panna kaikki todistukseni kehystettynä seinälle, ja se nähtäisiin vain positiivisena.

        Amerikassa pelastajat on sankareita. Suomessa he joutuu nöyristelemään. Pidän kyllä enemmän Amerikkalaisesta mentaliteetista. Siellä saa olla oma itsemsä.

        Voi-Ystävä-Hyvä

        Erinomaisuudestasi, tai uskostasi siihen, huolimatta, oman itsensä esille tuominen ja korostaminen on kaikissa yhteyksissä vastenmielisin piirre, joka kertoo lähinnä leuhkaajan vajaasta itsetunnosta, arvostuksen kerjuusta tai jonkin sateisesta mielen sairaudesta.

        Asiantuntemuksestasi on heti sanottava, että et ilmeisesti edes ymmärrä, mistä ilmiöstä ilmastonmuutoksen ja hiilidioksidin yhteydessä on kyse.
        Puhut jatkuvasti ilmakehän kaasujen pitoisuuksista ja lämpökapasiteettisuhteista / sisällöstä,niiden kyvystä sitoa ja säätää lämpötilaa ja jopa että SI-järjestelmässä olisi joitain määritelmiä näistä ja jotain sekavaa "kaasuinsinööreistä" ? ?

        Hiilidioksidin vaikutusta perustellaan väitteellä että se absorboi maasta lähtevää säteilyä, estäen sen pääsyä suoraan avaruuteen.
        Tämä säteily on aallonpituudeltaan yli 1µm, ja sitä ei absorboi happi eikä typpi, tässäkin yhteydessä on linkki kaaviosta, josta asia ilmene.

        Lisävinkkinä vielä että osallistujia tuskin kiinnostaa ne asiat, joihin ensisijaisesti olet keskittynyt, joten pieni tuokio miettien, mitä oikein kirjoitat ja kelle, saattaisi olla todella paikallaan.


      • huolipois
        SivullinenWaan kirjoitti:

        Voi-Ystävä-Hyvä

        Erinomaisuudestasi, tai uskostasi siihen, huolimatta, oman itsensä esille tuominen ja korostaminen on kaikissa yhteyksissä vastenmielisin piirre, joka kertoo lähinnä leuhkaajan vajaasta itsetunnosta, arvostuksen kerjuusta tai jonkin sateisesta mielen sairaudesta.

        Asiantuntemuksestasi on heti sanottava, että et ilmeisesti edes ymmärrä, mistä ilmiöstä ilmastonmuutoksen ja hiilidioksidin yhteydessä on kyse.
        Puhut jatkuvasti ilmakehän kaasujen pitoisuuksista ja lämpökapasiteettisuhteista / sisällöstä,niiden kyvystä sitoa ja säätää lämpötilaa ja jopa että SI-järjestelmässä olisi joitain määritelmiä näistä ja jotain sekavaa "kaasuinsinööreistä" ? ?

        Hiilidioksidin vaikutusta perustellaan väitteellä että se absorboi maasta lähtevää säteilyä, estäen sen pääsyä suoraan avaruuteen.
        Tämä säteily on aallonpituudeltaan yli 1µm, ja sitä ei absorboi happi eikä typpi, tässäkin yhteydessä on linkki kaaviosta, josta asia ilmene.

        Lisävinkkinä vielä että osallistujia tuskin kiinnostaa ne asiat, joihin ensisijaisesti olet keskittynyt, joten pieni tuokio miettien, mitä oikein kirjoitat ja kelle, saattaisi olla todella paikallaan.

        On vaan kummallista, että kaikki jotka on kanssani samaa mieltä ilmastoasiassa, on diplomi-insinöörejä ja yksi on tekniikan tohtori. Kukaan heistä ei usko tuohon ilmastohömppään. Yksi yliopisti tenisen puolen mies ilmoitti kielteisen kantansa tällä viikkollla, ja kertoi, ettei yliopistolla juuri kukaan usko ilmastohömppään.

        Kaikki nämä tahot tietää ilmakehän kaasuista vähintään saman kuin minä, ja se tieto on siinä SI järjestelmässä. Jos minä sinun mielestäsi leuhkin, niin se nyt on vain sinun käsitykesi. Minun mielestäni leuhkimista on se, että kertoo itsestään jotakin, mikä ei ole totta. Kaikki minun kertomani on totta.

        Minun kokemukseni on se,. että positiivisen kertominen itsestä vituttaa juuri niitä, joilla on itsetuntop-ongelma. Minä ollen tehnyt ikäni sellaisia töitä,. että niistä ei tule mitään, jos on ongelmia itsetunnon ja itsetuntemuksen kanssa.

        Sekin olisi tietysti sinun mielestäsi vain leuhkimista, jos kertoisin, mitä kaikkea olen tehnyt. Oliko sekin sinun mielestäsi vain leuhkimista,. että olen käynyt defibrillointi ja intubointikurssin. Tiedätkö sinä edes,mitä ne sanat tarkoittaa. Nykyiset sairaankuljettajat joita kehuin kyllä tietää.

        Sitäpaitsi minut on testattu kolme kertaa, joten kyllä' se itsetunto-ongelma niissä testeissä olisi tullut ilmi. Edelleenkin tuo sinun näkemyksesi on vain typerää suomalaista moralismia. Minä olen kuule lukenut psykologiaakin aika paljon. Molralisteillehan on tyypillistä juuri tuollainen reagointi.

        Sinulle tuntuu olevan iso olngelma, että jotku muut tietää jotakin, mitä sinä et tiedä. Minä olen sellaisista ottanut aina oppia, enkä halveksinut heitä. Valitettavan usein on vaan joutunut opettelemaan vaikeita asioita aivan itse ilman opastusta.


      • sairasautonkuljettaja
        huolipois kirjoitti:

        On vaan kummallista, että kaikki jotka on kanssani samaa mieltä ilmastoasiassa, on diplomi-insinöörejä ja yksi on tekniikan tohtori. Kukaan heistä ei usko tuohon ilmastohömppään. Yksi yliopisti tenisen puolen mies ilmoitti kielteisen kantansa tällä viikkollla, ja kertoi, ettei yliopistolla juuri kukaan usko ilmastohömppään.

        Kaikki nämä tahot tietää ilmakehän kaasuista vähintään saman kuin minä, ja se tieto on siinä SI järjestelmässä. Jos minä sinun mielestäsi leuhkin, niin se nyt on vain sinun käsitykesi. Minun mielestäni leuhkimista on se, että kertoo itsestään jotakin, mikä ei ole totta. Kaikki minun kertomani on totta.

        Minun kokemukseni on se,. että positiivisen kertominen itsestä vituttaa juuri niitä, joilla on itsetuntop-ongelma. Minä ollen tehnyt ikäni sellaisia töitä,. että niistä ei tule mitään, jos on ongelmia itsetunnon ja itsetuntemuksen kanssa.

        Sekin olisi tietysti sinun mielestäsi vain leuhkimista, jos kertoisin, mitä kaikkea olen tehnyt. Oliko sekin sinun mielestäsi vain leuhkimista,. että olen käynyt defibrillointi ja intubointikurssin. Tiedätkö sinä edes,mitä ne sanat tarkoittaa. Nykyiset sairaankuljettajat joita kehuin kyllä tietää.

        Sitäpaitsi minut on testattu kolme kertaa, joten kyllä' se itsetunto-ongelma niissä testeissä olisi tullut ilmi. Edelleenkin tuo sinun näkemyksesi on vain typerää suomalaista moralismia. Minä olen kuule lukenut psykologiaakin aika paljon. Molralisteillehan on tyypillistä juuri tuollainen reagointi.

        Sinulle tuntuu olevan iso olngelma, että jotku muut tietää jotakin, mitä sinä et tiedä. Minä olen sellaisista ottanut aina oppia, enkä halveksinut heitä. Valitettavan usein on vaan joutunut opettelemaan vaikeita asioita aivan itse ilman opastusta.

        Onko ylivertainen hitsari päässyt itseään parempaan seuraan kun seurustelee peräti diplomi-insinöörien ja tohtorien kanssa.


      • huolipois
        sairasautonkuljettaja kirjoitti:

        Onko ylivertainen hitsari päässyt itseään parempaan seuraan kun seurustelee peräti diplomi-insinöörien ja tohtorien kanssa.

        Niiden diplomoinsinöörie kanssa nyt on tullut seurusteltua opiskelussa ja työssä. Työssä tuli seurusteltua lääkäreidenkin kanssa. Onko siinä muka jotain ihmeellistä. Onko sinulla joku alemmuuskompleksi.

        Minulle se on ollut jokapäiväistä elämää seurustella itseäni viisaampien kanssa ja oppia heiltä. Tolhtori taas selvitti pyynnöstäni muutamia asioita hiilidioksidista. Etkö sinä ole koskaan päässyt seurustelemaan tohtoritason kanssa. Minulla on akateemisia suvussakin.

        Se on minulle aivan jokapäiväinen normaali asia. Minä voin keskustella vaikka presidentin kanssa. Ainakin minä voin hänelle tai presidentinkanslialle esittää oman kantani. Minulla ei ole koskaan ollut sitä herrapelkoa. On minun näkemyksiä otettu huomioonkin useita kertoja.


      • ilmastonmuutoskriitikko
        sairasautonkuljettaja kirjoitti:

        Onko ylivertainen hitsari päässyt itseään parempaan seuraan kun seurustelee peräti diplomi-insinöörien ja tohtorien kanssa.

        Terve vaan valeprofiili. Jos sinä nyt jotain tietäisit sairaankuljetyksesta, niin sinä tietäisit, ettei sairasautonkuljettajia ole nykyään olemassa. Kaikkien täytyy pystyä kaikkeen. Välillä ajetaan ja välillä hoidetaan. Siihen oli kehitys menossa jo 1990-luvulla.

        Periaatteessa on mahdollista, että isoissa palokunnissa jonku viimeisenä lähtevän auton kuskina on joku, jolla ei ole hoitokokemusta. Firmassa. jossa minä olin, on nykyään pelkästään sairaanhoitajia, ja porukkaa johtaa lääkäri ainakin etänä.

        Pojista tuli sairaanhoitajia ja tytöistä lääkäreitä. Vanhemmillakin on hoitoalan koulutus. Olen aikanaan sekä ajanut että hoitanut. Molemmissa tarvitaan kovaa kanttia ja osaamista. Muutaman kerran oli hälyytysajossa hengenlähtö lähellä muiden töpeksimisen takia.

        Vaikka on vilkut katolla ja sireeni huutaa, niin aina joutuu ottamaan koko vastuun yksin itselle. Ei niihin muihin voi luottaa. Joskus olin siviiliautolla liikkeellä, ja huomasin taustapeilistä siniset vilkut kilometrin päässä takanani.

        Seurasin piruuttain, koska muut huomaa. Kyllä se ambulanssi ne yllätti kaikki. Minut on kerran hälyytysajossa ollut poliisiauto yllättänyt, ja sillonkin ehdin väistään. Olen ajanut 46 vuotta.
        Katsotaanpa iskeekö Dunning Kruger kouluttaja, jolla on se tauti itsellä.


      • Anonyymi
        SivullinenWaan kirjoitti:

        Antaisin vihjeen että älä kirjoita novellia, jos vastaat johonkin kommenttiin, vain harvat viitsivät lukea ylipitkiä viestejä ja näin se ei tavoita kohdetta.

        Nasevampi kompakti vastaus riittää, suurin osa kykenee jonkin verran itsekin ajattelemaan ja liian yksityiskohtainen selostus tuntuu rasittavalta.

        Tietysti vielä tärkeämpää on kirjoittaa jotain muuta kuin huuhaata ja silkkaajaskaa.


      • Anonyymi

        OHHOHH! Jopa tuli paksu väite! Tyypillistä alarmismia ja AGW:tä! Hiilidioksidihihhulit väittävät, että hiilidioksidi lämmittää. Ei lämmitä hiilidioksidi. Aurinko lämmittää. Meret varastoivat auringon lämmön. Yöllä merivesi ei jäähdy paljon paskaakaan ja päivällä aurinko lämmittää taas. Napa-alueilla merivesi kyllä jäähtyy ja meri jäätyy kun yö kestää kuukausia. Meret ovat maapallon keskuslämmitys.

        Maapallo on kyllä joskus jäätynyt kokonaan. Näitä "lumipallomaa"-vaiheita on ollut yksi tai kaksi. Kaikkein kovimmat AGW-intoilijat väittivät tietenkin, että jäätyminen johtui hiilidioksidin puutteesta. Nyt tuo hihhulinäkemys näyttää sentään olevan katoamassa vähin äänin ja nykyisin jäätymisen syynä pidetään maan ja merien pinnan saavuttavan auringonsäteilyn heikkenemistä kun ilmakehä on ollut täynnä tomua ja tuhkaa. Tämä ns "ydintalvi" on ihan empiirisesti todettu tosiasia. Todella suurien tulivuorenpurkausten jälkeen koko planeetta on viilennyt kuukausiksi. On ollut kesätön vuosi tai pari.


    • lämmön.nettosiirron.teho

      Kun puhutaan ilmakehästä takaisin maan pintaan kohdistuvasta pitkäaaltoisesta lämpösäteilystä eli ns. takaisinsäteilystä niin aika usein jää täsmentämättä se, miltä nimenomaiselta korkeudelta kyseinen säteily oikeastaan emittoituu. Korjataan puute tässä kohdassa.

      Mittausten perusteella tiedetään, että suurin osa maan tai meren pintaan osuvasta takaisinsäteilystä on peräisin ilmakehän alimmasta 500 metrin paksuisesta kerroksesta. Sen yläpuolinen osa vastaa vain noin 15...20% osuudesta takaisinsäteilyn siirtämän lämmön kokonaismäärästä. Maan pintaa lähin 10 metrin paksuinen kerros ilmakehästä tuottaa noin 32...36%:sta takaisinsäteilystä eli noin kolmasosan siitä. Nämä tulokset löytyvät artikkelista 10.1016/0169-8095(89)90023-9 (Schmetz, J. 1989) sivulta 304.

      Mitä enemmän ilmakehässä on kasvihuonekaasuja (H2O, CO2) sitä lyhyemmän matkan absorptiokaistojen kohdalle sattuva pitkäaaltoinen säteily voi ilmakehässä lämpöä siirtää kerrallaan ennenkuin säteily absorboituu ja lämpö siirtyy takaisin kaasun lämpöliikkeeksi.

      Asiaa on oppikirjanomaisesti käsitelty tarkemmin artikkelissa, jonka doi koodi on 10.1016/B978-0-12-385954-9.00009-5 . Artikkelin tai oikeammin kirjan luvun 9 nimi suomennettuna on "Pinnan pitkäaaltoisen säteilyn budjetti" ja se on julkaistu osana kaukokartoitusta käsittelevää kirjaa vuonna 2012. Mainituilla koodeilla löytyvät molemmat artikkelit sci-hubin puolelta.

      • lämmön.nettosiirron.teho

        Kasvihuonekaasujen lisääntyminen lyhentää siis matkaa, jonka pitkäaaltoinen ja kasvihuonekaasujen absorptiospektrin alueelle osuva lämpösäteily pääsee etenemään absorboitumatta. Tämä vaikuttaa ulospäin lähtevään lämmön nettomääräisen siirtymisen tehoon. Tapahtumaketju menee näin:

        - kasvihuonekaasun molekyyli A absorboi pitkäaaltoisen lämpösäteilyn fotonin

        - Molekyylin A värähtely- ja pyörimisenergia lisääntyy (kvanttiluvut kasvavat).

        - joku toinen ilmakehän kaasumolekyyli törmää molekyyliin A,

        - törmäyksessä A:n värähtely-, pyörimisliike ja etenevä liike muuttuvat. Ylimääräinen energia siirtyy A:lta tuolle toiselle molekyylille.

        - toinen molekyyli jakaa/tasaa energiaansa seuraavassa törmäyksissä muille molekyyleille.

        - Nyt alkuperäisen kvantin energia on varastoituneena suuren molekyylimäärän lämpöliikkeeseen.

        - satunnaisissa törmäyksissä tuosta energiasta lopulta päätyy samanaikaisesti niin paljon jollekin kasvihuonekaasun molekyylille B, että sillä olisi mahdollisuus säteillä pitkäaaltoisen infrapunan fotoni.

        - Törmäyksessä B menettää energiansa säteilemättä törmäyksessä muille molekyyleille tai sitten se ehtii säteillä ennen törmäystä.

        - Toistetaan edellistä kolmea kohtaa kunnes joku energiaa saanut kasvihuonekaasun molekyyli Z lopulta ehtii säteillä pitkäaaltoisen infrapunasäteilyn fotonin ennen seuraavaa törmäystä.

        Huomattavaa tuossa on se, että 1) prosessissa kuluu runsaasti aikaa, 2) uuden fotonin lähtösuunta on satunnainen eli se voi esimerkiksi kääntyä 180 astetta ja 3) vaikka säteilyn alun perin absorboi vesi niin lähettävä molekyyli voikin olla hiilidioksidia tai päinvastoin eli fotonin aallonpituuskaan ei ole enää sama kuin aiemmin. Absorption ja emission välinen matka on maan pinnan lähellä noin kolmasosalle tapahtumista luokkaa 10 metriä ja vain 15...20% säteilystä pääsee etenemään yli 500 m matkan. Loppu noin 50% absorboituu 10 ja 500 metrin välillä.

        Huomattavaa on myös se, että fotonin säteileminen on tilastollinen ilmiö eli tapahtuu tietyllä todennäköisyydellä aikayksikköä kohti. Erityisesti se ei edellytä sitä että jonkun tietyn ja nimenomaisen molekyylin olisi oltava suurienergisessä tilassa keskimääräisen säteilyyn kuuluvan ajan verran. Riittää, että suuresta joukosta molekyylejä aina osa viettää aikaa suurienergisessä tilassa törmäysten välillä.


      • Mainitsemani kaukokartoitusta käsittelevä artikkeli "Pinnan pitkäaaltoisen säteilyn budjetti" löytyy tällä hetkellä tämän linkin kautta:

        https://sci-hub.se/10.1016/B978-0-12-385954-9.00009-5

        Kannattaa googlata sci-hub mikäli tämä kyseisen sivuston haara sattuu sulkeutumaan.


      • Anonyymi
        Aesdifaeger kirjoitti:

        Mainitsemani kaukokartoitusta käsittelevä artikkeli "Pinnan pitkäaaltoisen säteilyn budjetti" löytyy tällä hetkellä tämän linkin kautta:

        https://sci-hub.se/10.1016/B978-0-12-385954-9.00009-5

        Kannattaa googlata sci-hub mikäli tämä kyseisen sivuston haara sattuu sulkeutumaan.

        https://keskustelu.suomi24.fi/t/17035856/avustusanomus

        Tuossa uudempi keskustelu fysiikka - palstalla aiheesta kaasujen emissiokertoimista NTP - olosuhteissa ( NTP = 20 ⁰C astetta, 101.325 kPa ilmanpaine)


      • Anonyymi
        lämmön.nettosiirron.teho kirjoitti:

        Kasvihuonekaasujen lisääntyminen lyhentää siis matkaa, jonka pitkäaaltoinen ja kasvihuonekaasujen absorptiospektrin alueelle osuva lämpösäteily pääsee etenemään absorboitumatta. Tämä vaikuttaa ulospäin lähtevään lämmön nettomääräisen siirtymisen tehoon. Tapahtumaketju menee näin:

        - kasvihuonekaasun molekyyli A absorboi pitkäaaltoisen lämpösäteilyn fotonin

        - Molekyylin A värähtely- ja pyörimisenergia lisääntyy (kvanttiluvut kasvavat).

        - joku toinen ilmakehän kaasumolekyyli törmää molekyyliin A,

        - törmäyksessä A:n värähtely-, pyörimisliike ja etenevä liike muuttuvat. Ylimääräinen energia siirtyy A:lta tuolle toiselle molekyylille.

        - toinen molekyyli jakaa/tasaa energiaansa seuraavassa törmäyksissä muille molekyyleille.

        - Nyt alkuperäisen kvantin energia on varastoituneena suuren molekyylimäärän lämpöliikkeeseen.

        - satunnaisissa törmäyksissä tuosta energiasta lopulta päätyy samanaikaisesti niin paljon jollekin kasvihuonekaasun molekyylille B, että sillä olisi mahdollisuus säteillä pitkäaaltoisen infrapunan fotoni.

        - Törmäyksessä B menettää energiansa säteilemättä törmäyksessä muille molekyyleille tai sitten se ehtii säteillä ennen törmäystä.

        - Toistetaan edellistä kolmea kohtaa kunnes joku energiaa saanut kasvihuonekaasun molekyyli Z lopulta ehtii säteillä pitkäaaltoisen infrapunasäteilyn fotonin ennen seuraavaa törmäystä.

        Huomattavaa tuossa on se, että 1) prosessissa kuluu runsaasti aikaa, 2) uuden fotonin lähtösuunta on satunnainen eli se voi esimerkiksi kääntyä 180 astetta ja 3) vaikka säteilyn alun perin absorboi vesi niin lähettävä molekyyli voikin olla hiilidioksidia tai päinvastoin eli fotonin aallonpituuskaan ei ole enää sama kuin aiemmin. Absorption ja emission välinen matka on maan pinnan lähellä noin kolmasosalle tapahtumista luokkaa 10 metriä ja vain 15...20% säteilystä pääsee etenemään yli 500 m matkan. Loppu noin 50% absorboituu 10 ja 500 metrin välillä.

        Huomattavaa on myös se, että fotonin säteileminen on tilastollinen ilmiö eli tapahtuu tietyllä todennäköisyydellä aikayksikköä kohti. Erityisesti se ei edellytä sitä että jonkun tietyn ja nimenomaisen molekyylin olisi oltava suurienergisessä tilassa keskimääräisen säteilyyn kuuluvan ajan verran. Riittää, että suuresta joukosta molekyylejä aina osa viettää aikaa suurienergisessä tilassa törmäysten välillä.

        Toisessa keskustelussa

        https://keskustelu.suomi24.fi/t/17927096/ilmasto-ei-lampia-maa-lampiaa#comment-121925243

        tuli esille lainaus sivustolta, jossa tätä asiaa on käsitelty ja rautalankaa väännetty:

        https://kysymykset.kysyilmastosta.fi/t/co2-n-lammityksen-vaikutusmekanismi/740

        Tuossa siis korjataan virheitä vilisevästä Arvolan blogista palstalle usein kopioitu niinikään virheellinen väite hiilidioksidin kyvyttömyydestä emittoida säteilyä alailmakehässä (joka siis on väärin, CO2 emittoi säteilyä alailmakehässä ongelmitta niin teorian kuin mittaustenkin perusteella) johtuen törmäyksistä muihin kaasumolekyyleihin.

        Samaa asiaa käsittelin itse palstalla vuonna 2018:
        https://keskustelu.suomi24.fi/t/15411377/ilmastonmuutos-alusta-alkaen#comment-95070678


    • lämmön.nettosiirron.teho

      Keskustelussa "Ilmastonmuutoksen kumoaminen on helppoa" viitattiin virheelliseen ajatukseen siitä, että viileämmän ympäristön lämpötila ei vaikuttaisi lämmön nettosiirron tehoon kuumasta kappaleesta ympäristöönsä, löytyy lyhytlinkistä urly piste fi kauttaviiva 17nU

      • Anonyymi

        Tarkemmin "takaisinsäteilyuuni" - väitteen virheitä käsitellään tässä viestissä:
        https://keskustelu.suomi24.fi/t/15499762/ilmastonmuutosopin-kumoaminen-on-helppoa-#comment-95976922


    • lämmön.nettosiirron.teho

      Keskustelussa "Avaruus todistaa." yritettiin taas esittää ajatusta siitä, että takaisinsäteily eli viileämmän ympäristön kuumempaan kappaleeseen palauttama teho olisi jotenkin virheellinen ajatus. Keskustelu löytyy lyhytlinkillä urly piste fi kauttaviiva 17ub

    • lämmön.nettosiirron.teho

      Keskustelussa "Vastauksia luvassa" kyseltiin lämmön siirtymisestä ilmakehässä pitkäaaltoisen säteilyn avulla. Keskustelu löytyy lyhytlinkillä urly piste fi kauttaviiva 19wy

      Kyseinen keskustelu on toistaiseksi 24/7 talletuksessa.

    • lämmön.nettosiirron.teho

      Ikkunalasista valmistetun lasilevyn läpäisy (transmissio) pitkäaaltoiselle infrapunaiselle säteilylle löytyy lasintuottajan sivuilta vuonna 2012 arkistoidusta dokumentista urly piste fi kauttaviiva 1aJA

      Tuossa siis on mitattuna eripaksuisten ikkunalasien läpäisyä 2.5µm pidemmille aallonpituuksille. Läpäisy näyttää kuvassa 1. esimerkiksi 3 mm paksuiselle lasille romahtavan 2.7...2.8 mikrometrin välillä tasaantuen noin 25% eli 0.25 paikkeille välille 2.8...4.2µm. Aallonpituuden kasvaessa läpäisy putoaa edelleen niin, että 5µm kohdalla se olisi tuolle lasille alle 0.002.

      Ikkunalasin heijastavuus näyttää nousevan hyvin pienestä arvosta 8.2µm kohdalla 9.5µm kohdalle 0.3:een asti. Sen jälkeen se putoaa samalla lailla takaisin saavuttaen arvon 0.025 aallonpituuden 12.5µm kohdalla. Tämä näkyy kuvassa 3.

      • Anonyymi

    • lämmön.nettosiirron.teho

      Auringon säteilyspektri ilmakehän ulkopuolella ultravioletista pitkäaaltoiseen infrapunaan asti on tunnettu ja taulukoitu tietokantaan. Sen saa vaikkapa excel- taulukkoon tekstitiedostosta, joka löytyy NREL:n sivulta hakusanoilla

      MODTRAN ETR

      Tuossa siis on taulukossa pienenevän aaltoluvun mukaan tieto aallonpituudesta ja kyseisellä aallonpituudella vallitsevasta säteilytehosta yhden nanometrin kaistanleveyttä kohti (wattia per neliömetri). Taulukko alkaa aaltoluvusta 50000cm⁻¹ (20nm) ja jatkuu siitä aaltolukuun 50cm⁻¹ (20µm) asti aina yhden aaltoluvun askelin eli 50000, 49999,49998, ... , 52, 51, 50cm⁻¹

      Tuon avulla on helpohko laskea paljonko Auringon säteilytehoa tulee millekin aallonpituuskaistalle ilmakehän ulkopuolella.

    • eddeefrfrde

      Auringon lämmön/säteilyn kohde on suurimmaksi osaksi vettä ja sen lämpö siirtyy suureksi osaksi kylmempiin syviin vesiin. Meret tasaavat lämpötiloja ja varastoivat sitä. Sama tapahtuma vesihöyryllä.

      Hiilidioksidi ei varastoi lämpöä käytännössä yhtään kun sitä on myös niin vähän.

      • Anonyymi

        Juu, CO2 ei ole lämmön varasto vaan energian siirtäjä muiden kasvihuonekaasujen kera. Ne siirtävät energiaa pitkäaaltoisesta Maapallon pinnan ja pilvien lähettämästä infrapunaisesta lämpösäteilystä ilmakehän kaasumolekyylien lämpöliikkeeseen ja myös toiseen suuntaan eli ilmakehän lämmöstä pitkäaaltoiseksi infrapunasäteilyksi. Toimivat kuten radiolähettimen antennit. Lämpövarastona ilmakehässä toimivat typpi, happi ja vesihöyry.


    • Muistutetaan siitä, miten lämpösäteily siirtää energiaa. Samaan liittyvää keskustelua on käyty myös otsikolla "Olisiko teillä näyttää hiilidoksidin absorbiokaavaa"

      • Anonyymi

        Yhden kommentoijan mukaan fysiikka ei tarvittaisi ollenkaan palstalla joka todistaa AGW ilmiöstä ja muut eivät saisi puheenvuoroa ollenkaan.Outoa toimintaa demokratiassa.


      • Anonyymi
        Anonyymi kirjoitti:

        Yhden kommentoijan mukaan fysiikka ei tarvittaisi ollenkaan palstalla joka todistaa AGW ilmiöstä ja muut eivät saisi puheenvuoroa ollenkaan.Outoa toimintaa demokratiassa.

        AGW-ilmiöstä todistava palsta on todella tärkeä. Se on melkein yhtä tärkeä kuin IPCC ja IPCC:n julkaisema luettelo seurakunnan hartaille uskovaisille luettavaksi sallituista kirjoista.
        Reaalimaailman kanssa jollain AGW:llä ei ole tietenkään mitään tekemistä.


    • Anonyymi

      Lämmön nettosiirron teho tuli esille keskustelussa, jonka otsikko kuuluu "Mitenkäs lämpö siirtyykään?"

    • Anonyymi

      Nyt on pilvistä. Ulkona ilma on tavallisen ikkunaan asennetun lämpömittarin mukaan noin 15ºC ja maahan seinään ja maahan osoittaessa IR - lämpömittarin mielestä lämpötila on 14ºC. Kun hetki sitten osoitin IR-lämpömittarin kohti pilveä niin pilven lämpötilaksi mittari kertoi 11ºC. Pilvien välistä tuli hetkeksi esille sinistä taivasta jossa oli vielä pieniä pilvenhautuvia jäljellä. Sinisen taivaan suunnassa mittari näytti -35ºC (MIINUS 35 astetta) eli reilusti kovan pakkasen puolelle. Kun taivas on kunnolla pilvetön niin IR - mittari näyttää pienintä mahdollista lukemaansa.

      Maan- tai merenpinnan lämpösäteilyn kannalta katsottuna kirkkaan taivaan lämpötila on reilusti pakkasen puolella muualla kuin Auringon tai pilvien suunnassa.

      • Anonyymi

        Suoritin saman lämpötilamittauksen nyt päivän puolella auringon paistaessa. Fluken IR-lämpömittarin mielestä kirkas sininen taivas suoraan yläpuolella klo 20:00 oli lämpötilaltaan -38 astetta samaan aikaan kun varjon puolella maan lämpötila on 21 astetta.


      • Anonyymi
        Anonyymi kirjoitti:

        Suoritin saman lämpötilamittauksen nyt päivän puolella auringon paistaessa. Fluken IR-lämpömittarin mielestä kirkas sininen taivas suoraan yläpuolella klo 20:00 oli lämpötilaltaan -38 astetta samaan aikaan kun varjon puolella maan lämpötila on 21 astetta.

        Paljonko se -38 asteinen hiilidioksidin säteily lämittää?


      • Anonyymi
        Anonyymi kirjoitti:

        Paljonko se -38 asteinen hiilidioksidin säteily lämittää?

        Laske itse. Mittari mittasi ilmakehästä takaisinsäteilevän kokonaistehon tuosta suunnasta ja sen perusteella kertoi kuvitelmansa säteilylähteen lämpötilasta. Minulla oli tuossa emissiokertoimelle asetettu arvo 0.95 eli Stefan-Boltzmannin lain avulla saat lämpötilasta ja emissiokertoimesta laskettua takaisintulevan säteilyn tehon kyseisestä suunnasta.

        Kun mittaria kallisti lähemmäs horisonttia alkoi sen näyttämä lämpötila nousta. Kun mittari osoitti 30 astetta horisontin yläpuolelle pohjoiseen oli lämpötilalukema -20 astetta. Takaisinsäteilyähän tulee kaikista suunnista, ei ainoastaan suoraan ylhäältäpäin.


      • Anonyymi
        Anonyymi kirjoitti:

        Paljonko se -38 asteinen hiilidioksidin säteily lämittää?

        Enemmän se lämmittää kuin aikoinaan, jolloin hiilidioksidia oli vähemmän ja jolloin samassa mittauksessa olisi -38 asteen sijaan näkynyt esimerkiksi -40 astetta. Tasapainolämpötila on siksi noussut.


      • Anonyymi
        Anonyymi kirjoitti:

        Laske itse. Mittari mittasi ilmakehästä takaisinsäteilevän kokonaistehon tuosta suunnasta ja sen perusteella kertoi kuvitelmansa säteilylähteen lämpötilasta. Minulla oli tuossa emissiokertoimelle asetettu arvo 0.95 eli Stefan-Boltzmannin lain avulla saat lämpötilasta ja emissiokertoimesta laskettua takaisintulevan säteilyn tehon kyseisestä suunnasta.

        Kun mittaria kallisti lähemmäs horisonttia alkoi sen näyttämä lämpötila nousta. Kun mittari osoitti 30 astetta horisontin yläpuolelle pohjoiseen oli lämpötilalukema -20 astetta. Takaisinsäteilyähän tulee kaikista suunnista, ei ainoastaan suoraan ylhäältäpäin.

        Kun kallistit mittaria lähemmäksi horisonttia niin se mittasi lämpötilan alemmasta ilmakerroksesta. Totta kai se silloin näyttää lämpimämpää lukemaa. Oletko noin yksinkertainen vai oletko nauttinut liikaa hiilidioksidipitoisia juomia.


      • Anonyymi
        Anonyymi kirjoitti:

        Enemmän se lämmittää kuin aikoinaan, jolloin hiilidioksidia oli vähemmän ja jolloin samassa mittauksessa olisi -38 asteen sijaan näkynyt esimerkiksi -40 astetta. Tasapainolämpötila on siksi noussut.

        Höpö höpö. Ei se -38 tai 40 asteinen yläilmakehän pakkanen lämmitä yhtään maanpintaa.


      • Anonyymi
        Anonyymi kirjoitti:

        Höpö höpö. Ei se -38 tai 40 asteinen yläilmakehän pakkanen lämmitä yhtään maanpintaa.

        Katsopa tuosta ylempää keskustelusta käsite lämmön nettosiirron teho. Lue viesti, jonka on kirjoittanut nimimerkki lämmön.nettosiirron.teho ajankohtana 24.10.2018 12:55

        Kerro omin sanoin miten pinnan lähettämän ja vastaanottaman lämpösäteilyn tehot liittyvät lämmön nettosiirron tehoon. Sitä ennen keskustelu kanssasi aiheesta on ajan tuhlausta.


      • Anonyymi
        Anonyymi kirjoitti:

        Katsopa tuosta ylempää keskustelusta käsite lämmön nettosiirron teho. Lue viesti, jonka on kirjoittanut nimimerkki lämmön.nettosiirron.teho ajankohtana 24.10.2018 12:55

        Kerro omin sanoin miten pinnan lähettämän ja vastaanottaman lämpösäteilyn tehot liittyvät lämmön nettosiirron tehoon. Sitä ennen keskustelu kanssasi aiheesta on ajan tuhlausta.

        Mitään lämmön takaisinsäteilyä ei ole. Tuo yläilmakehän pakkanen ei lämmitä yhtään maanpintaa. Lämpö siirtyy vain kylmempään suuntaan.


      • Anonyymi
        Anonyymi kirjoitti:

        Mitään lämmön takaisinsäteilyä ei ole. Tuo yläilmakehän pakkanen ei lämmitä yhtään maanpintaa. Lämpö siirtyy vain kylmempään suuntaan.

        Selitys ilmiölle on tuossa edellä mainitussa viestissä joka on lähetetty 24.10.2018 klo 12:55. Jos ei kiinnosta lukea sitä niin se on oma valintasi. Minun valintani on olla keskustelematta niiden kanssa jotka eivät vaivaudu lukemaan saamiaan vastauksia.


    • Englanninkielisen Wikipedian lämpösäteilyä käsittelevä sivu sisältää viitteestään [3] kyseisen kirjan kappaleen 23 "Fotonit" alusta sivulta 247 lainattua sisältöä, joka suoraan kirjasta suomennettuna kuuluisi näin:

      "Kaikki absoluuttisen nollapisteen yläpuolella oleva aine lähettää sähkömagneettista säteilyä. Tuota säteilyä kutsutaan lämpösäteilyksi."

      Tuo Wikipedian viite [3] on englanninkielinen Stephen ja Katerine Blundellin vuonna 2006 julkaisema oppikirja (ISBN 9780198567707), jonka nimi suomennettuna voisi olla "Lämpöfysiikan käsitteitä".

      Wikipedian tekstissä hieman alempana kerrotaan Maapallon ilmakehän olevan kasvihuonekaasuja lukuunottamatta läpinäkyvää pitkäaaltoiselle infrapunasäteilylle eli Maapallon pinnan lähettämälle aallonpituudeltaan luokkaa yli 2.5 µm olevalle lämpösäteilylle. Palstan keskusteluissa on Wikipediassa mainitusta ilmakehän läpinäkyvyydestä huolimatta ilmennyt väärinkäsitystä sen suhteen, että "lähettää ja vastaanottaa lämpösäteilyä" virheellisesti tarkoittaisi kaikkien kaasujen säteilevän ja vastaanottavan lämpösäteilyä myös pitkäaaltoisen infrapunan puolella.

      Vain osa kaasuista pystyy emittoimaan ja absorboimaan yli 2.5 µm aallonpituuksilla eli pitkäaaltoisella infrapunalla. Näitä kaasuja kutsutaan kasvihuonekaasuiksi. Jokaisella aineella on sille ominainen emissiviteetti, joka voi riippua voimakkaastikin aallonpituudesta. Kaasujen tapauksessa emissiviteetti on laajoilla aallonpituusalueilla nollassa ja poikkeaa nollasta vain spektrin piikkien kohdalla. Resiprookkisuuden (Kirchhoffin lämpösäteilylaki, kirjan kappale 23.3 sivu 250) vuoksi kaasut sekä lähettävät että vastaanottavat lämpösäteilyä samoilla aallonpituuksilla eli rajoitukset koskevat sekä emissiota että absorptiota.

      Samaisessa oppikirjassa [3] on ilmakehän ominaisuuksille kokonainen oma kappaleensa 37 Maapallon ilmakehä. Siinä on kohdassa 37.3 Kasvihuoneilmiö myös selvitetty eri kaasujen roolia lämpösäteilyn kannalta katsottuna.

      Kirjan kappaleessa 37.3 sivulla 427 kerrotaan, että molekyylillä on oltava dipolimomentti jotta sen värähtelyt voisivat kytkeytyä suoraan inrfapunasäteilyyn. Kappaleessa mainitaan, että happi- ja typpimolekyyleillä O2 ja N2 ei ole värähdellessään dipolimomenttia josta syystä johtuen ne eivät kytkeydy infrapunasäteilyyn eivätkä siten vaikuta Maapallon pinnan lähettämään pitkäaaltoiseen infrapunasäteilyyn. Kasvihuonekaasuiksi kutsutuilla molekyyleillä on jko pysyvä dipolimomentti (H2O) tai ne tietyissä värähtelymoodeissaan tuottavat dipolimomentin (CO2).

      Ilmakehän kaasuista happi ja typpi eivät siis termofysiikan oppikirjan mukaan pysty lähettämään tai vastaanottamaan Maapallon pinnan lähettämää pitkäaaltoisen infrapunan alueelle osuvaa lämpösäteilyä.

    • Lämpösäteilyn kautta tapahtuvaa lämmön siirtoa on tutkittu pitkään. Sattuneista syistä johtuen avaruustekniikassa asia on erityisen merkittävä joten siihen liittyen löytyy NASA:lta julkaisuja.

      Netistä löytyy NASA:n palvelimelta 1970 - luvulta vapaasti luettavissa oleva kolmen kirjan sarja, joka käsittelee aihetta tuon ajan tiedon mukaisesti. Dokumentit löytyvät tekemällä google - haun alla olevilla hakusanoilla:

      NASA-SP-164-VOL-3 pdf

      Vaihtamalla ylläoleviin tilalle VOL-2 ja VOL-1 saa kirjasarjan kaksi ensimmäistä teosta näkyviin. Kaasujen roolia lämpösäteilyn absorboijana, emittoijana ja sirottajana käsittelee tuo mainitun kirjasarjan kolmas osa.

      • Anonyymi

        Hyvin todennäköisesti niin.

        Veikkaisin kuitenkin että palstan lukijoista kukaan viitsii ryhtyä niistä opiskelemaan termodynamiikkaa tai lämmönsiirron tutkimus- ja mittaustuloksia.

        Palstalla esitetty kysymys oli yksinkertaisesti, mistä johtuu väitetty ominaisuus että vain kasvihuonekaasut voisivat säteillä lämpösäteilyä, siis mikä mekanismi lopettaa toisissa kaasuissa hiukkasten lämpöliikkeen ja toisissa ei.


      • Anonyymi
        Anonyymi kirjoitti:

        Hyvin todennäköisesti niin.

        Veikkaisin kuitenkin että palstan lukijoista kukaan viitsii ryhtyä niistä opiskelemaan termodynamiikkaa tai lämmönsiirron tutkimus- ja mittaustuloksia.

        Palstalla esitetty kysymys oli yksinkertaisesti, mistä johtuu väitetty ominaisuus että vain kasvihuonekaasut voisivat säteillä lämpösäteilyä, siis mikä mekanismi lopettaa toisissa kaasuissa hiukkasten lämpöliikkeen ja toisissa ei.

        "mistä johtuu väitetty ominaisuus että vain kasvihuonekaasut voisivat säteillä lämpösäteilyä"

        Tarkennus: "Vain kasvihuonekaasut voivat säteillä lämpösäteilyä pitkäaaltoisen infrapunan aallonpituuksilla." Syitä voi hakea molekyylien fysiikasta, asia selitetty kirjasarjan kolmannessa osassa.

        Väärinkäsitys: "siis mikä mekanismi lopettaa toisissa kaasuissa hiukkasten lämpöliikkeen ja toisissa ei". Tuo on virheellinen ajatus, kuten kirjasarjan kolmannessa osassa ja aiemmin esille otetussa oppikirjassa kerrotaan. Kun molekyylillä ei ole dipolimomenttia niin sen lämpöliike ei kytkeydy pitkäaaltoisen infrapuna-alueen sähkömagneettiseen säteilyyn.

        Kysymyksiin on jo vastattu tuossa yllä olevassa 30.06.2019 nimimerkillä Aesdifaeger lähetetyssä viestissä. Nyt aiheesta kiinnostuneille löytyy siis mainitun vuoden 2006 termofysiikan oppikirjan lisäksi 1970 - luvulla NASA:n lämpösäteilyyn liittyen julkaisemat kirjat, jossa samaa käsitellään käytännönläheisemmin.


      • Anonyymi
        Anonyymi kirjoitti:

        "mistä johtuu väitetty ominaisuus että vain kasvihuonekaasut voisivat säteillä lämpösäteilyä"

        Tarkennus: "Vain kasvihuonekaasut voivat säteillä lämpösäteilyä pitkäaaltoisen infrapunan aallonpituuksilla." Syitä voi hakea molekyylien fysiikasta, asia selitetty kirjasarjan kolmannessa osassa.

        Väärinkäsitys: "siis mikä mekanismi lopettaa toisissa kaasuissa hiukkasten lämpöliikkeen ja toisissa ei". Tuo on virheellinen ajatus, kuten kirjasarjan kolmannessa osassa ja aiemmin esille otetussa oppikirjassa kerrotaan. Kun molekyylillä ei ole dipolimomenttia niin sen lämpöliike ei kytkeydy pitkäaaltoisen infrapuna-alueen sähkömagneettiseen säteilyyn.

        Kysymyksiin on jo vastattu tuossa yllä olevassa 30.06.2019 nimimerkillä Aesdifaeger lähetetyssä viestissä. Nyt aiheesta kiinnostuneille löytyy siis mainitun vuoden 2006 termofysiikan oppikirjan lisäksi 1970 - luvulla NASA:n lämpösäteilyyn liittyen julkaisemat kirjat, jossa samaa käsitellään käytännönläheisemmin.

        Sotket nyt absorption ja lämpösäteilyn toisiinsa.

        Lämpösäteily jakautuu tietyn tilastollisen jakauman mukaan lähes koko skaalan alalle ja lämpötila määrittää maksimin ja tehon, kyse on eri asiasta kuin absorptio, joka voi olla molekyylirakenteesta riippuva ja vain rajatulla aallonpituudella.


      • Anonyymi
        Anonyymi kirjoitti:

        Sotket nyt absorption ja lämpösäteilyn toisiinsa.

        Lämpösäteily jakautuu tietyn tilastollisen jakauman mukaan lähes koko skaalan alalle ja lämpötila määrittää maksimin ja tehon, kyse on eri asiasta kuin absorptio, joka voi olla molekyylirakenteesta riippuva ja vain rajatulla aallonpituudella.

        Blundellien termofysiikan kirjan saa luettavakseen Libgen - palvelusta tekemällä siellä haun nimellä "Blundell". Miksi kopioisin kirjaa kohta kohdalta suomennettuna tänne palstalle kun sinulla itselläsi on mahdollisuus lukea se itse? Jos ei kiinnosta niin paljoa että viitsisit niin se on oma valintasi.

        Kuten jo useasti mainittu Kirchhoffin lämpösäteilylain perusteella materiaali emittoi ja absorboi lämpösäteilyä samoilla aallonpituuksilla eli aineen emissiivisyys riippuu aallonpituudesta ja määrää sekä lämpösäteily lähettämisen että sen vastaanottamisen. Kaasumolekyylien lämpösäteilyabsorptio tapahtuu kapeilla aallonpituusalueilla joten ne emittoivat lämpösäteilyä vain samoilla kapeilla kaistoilla.

        Lue tuossa yllä annettua materiaalia ja jos siitä tulee kysyttävää niin kysy. Kerro omin sanoin mitä et ymmärtänyt ja millä sivulla mitä kirjaa oli kyseisen asian selitys joka jäi lukiessasi epäselväksi.

        Jos et vastauksissasi osoita lukeneesi selitystä kirjasta niin en tuhlaa aikaani vastaamiseen.


    • Anonyymi

      Muistutetaan siitä, että pitkäaaltoisen lämpösäteilyn absorboituminen vain tiettyihin kaasuihin (kasvihuonekaasut) on ollut tunnettu asia jo pitkään.

      • Anonyymi

        Pitänee paikkansa.

        Mutta eräät muut tähän liittyvät väitteet eivät.

        Kun käsitellään energian siirtymistä säteilyn avulla, oletetaan että vallitsee termodynaaminen tasapaino, eli aineeseen ei tule, eikä lähde muuta energiaa.
        Tällöin pätee että säteilevän energian suuruus on sama kuin absorboidun.
        Koska fotonin energia on suoraan suhteessa aallonpituuteen, on syntynyt virhearvio että säteilyn aallonpituus olisi oltava sama kuin mitä aine absorboi.

        Näin olisi, jos säteily olisi monokromaattista, jota lämpösäteily ei suinkaan ole.

        Toinen väärinkäsitys on, että jos aine ei absorboi säteilyä, se ei myöskään emittoi.

        Sekin pitää paikkansa, jos aineeseen ei tule mitään muuta energiaa, ja sen lämpötila on 0 K, siis tilanne joka on vain teoreettinen, eli kaikki aineet (yli 0 K) säteilevät lämpösäteilyä tietyllä lämpötilaan liittyvällä spektrillä.


      • Anonyymi
        Anonyymi kirjoitti:

        Pitänee paikkansa.

        Mutta eräät muut tähän liittyvät väitteet eivät.

        Kun käsitellään energian siirtymistä säteilyn avulla, oletetaan että vallitsee termodynaaminen tasapaino, eli aineeseen ei tule, eikä lähde muuta energiaa.
        Tällöin pätee että säteilevän energian suuruus on sama kuin absorboidun.
        Koska fotonin energia on suoraan suhteessa aallonpituuteen, on syntynyt virhearvio että säteilyn aallonpituus olisi oltava sama kuin mitä aine absorboi.

        Näin olisi, jos säteily olisi monokromaattista, jota lämpösäteily ei suinkaan ole.

        Toinen väärinkäsitys on, että jos aine ei absorboi säteilyä, se ei myöskään emittoi.

        Sekin pitää paikkansa, jos aineeseen ei tule mitään muuta energiaa, ja sen lämpötila on 0 K, siis tilanne joka on vain teoreettinen, eli kaikki aineet (yli 0 K) säteilevät lämpösäteilyä tietyllä lämpötilaan liittyvällä spektrillä.

        Kirchhoffin säteilylain mukaan aine säteilee ja vastaanottaa lämpösäteilyä samoilla (aineen elektronirakenteesta ja lämpötilasta riippuvilla) aallonpituuksilla. Jos se ei tietyssä lämpötilassa absorboi säteilyä jollakin aallonpituudella niin se ei kyseisessä lämpötilassa myöskään säteile kyseisellä aallonpituudella.

        Tuo on tiedetty jo vuodesta 1862, jolloin Gustav Kirchhoff julkaisi tutkimuksensa asiasta. Ei ole siis mikään uusi juttu. Lisää löytyy hakusanoilla Kirchhoffin säteilylaki.


      • Anonyymi
        Anonyymi kirjoitti:

        Kirchhoffin säteilylain mukaan aine säteilee ja vastaanottaa lämpösäteilyä samoilla (aineen elektronirakenteesta ja lämpötilasta riippuvilla) aallonpituuksilla. Jos se ei tietyssä lämpötilassa absorboi säteilyä jollakin aallonpituudella niin se ei kyseisessä lämpötilassa myöskään säteile kyseisellä aallonpituudella.

        Tuo on tiedetty jo vuodesta 1862, jolloin Gustav Kirchhoff julkaisi tutkimuksensa asiasta. Ei ole siis mikään uusi juttu. Lisää löytyy hakusanoilla Kirchhoffin säteilylaki.

        Ninn.

        Noita lämpötilariippuvia ominaisuuksia kutsutaan spektriviivoiksi, joiden avulla on mahdollista tunnistaa emittoiva aine, mutta mitä tekemistä niillä on kokonaissäteilyn,- absorption tai ilmastonmuutoksen kanssa ?


    • Anonyymi

      Muistutetaan siitä, mikä tuossa ylempää löytyvässä viestissä lämmön.nettosiirron.teho 29.10.2018 16:41 jo kerrottiin. Takaisinsäteilystä noin kolmasosa tulee alimmasta 10 metrin paksuisesta ilmakerroksesta. Näin siis ainakin silloin kun ollaan lähellä merenpinnan tasoa.


      • Anonyymi
        EelisPermanen kirjoitti:

        Tässäpä sitä takaisinsäteilyä.
        Tätähän se tarkoittaa.

        http://ilovemycarbondioxide.com/images/IPCC_oven.png

        Eli melkoista hömppää.
        Mistä se "takaisinsäteilyn" lisäenergia tulisi?
        Sitä ei koskaan osata kertoa.

        Virheellinen kuva. Kuvan uunissa kanasta lähtee energiaa ulospäin lämpösäteilynä ja sama säteily sitten heijastuu takaisin kanaan. Lämpöä siis menee ulospäin kanasta ihan sama määrä kuin mikä tulee takaisinkin eli kanan lämpötila pysyisi vakiona toisin kuin kuvassa väitetään. Lämmön nettosiirron teho on tuossa nolla.

        Mielenkiintoa tulee tilanteessa, jossa kappaletta lämmitetään vakioteholla ja sen annetaan jäähtyä lämpösäteilyn avulla viileampään ympäristöön. Se tilanne on käyty läpi yksityiskohtaisesti tässä keskustelussa ylempänä viesteissä, jotka on kirjoittanut nimimerkki lämmön.nettosiirron.teho päivämäärällä 24.10.2018.


      • Anonyymi
        Anonyymi kirjoitti:

        Virheellinen kuva. Kuvan uunissa kanasta lähtee energiaa ulospäin lämpösäteilynä ja sama säteily sitten heijastuu takaisin kanaan. Lämpöä siis menee ulospäin kanasta ihan sama määrä kuin mikä tulee takaisinkin eli kanan lämpötila pysyisi vakiona toisin kuin kuvassa väitetään. Lämmön nettosiirron teho on tuossa nolla.

        Mielenkiintoa tulee tilanteessa, jossa kappaletta lämmitetään vakioteholla ja sen annetaan jäähtyä lämpösäteilyn avulla viileampään ympäristöön. Se tilanne on käyty läpi yksityiskohtaisesti tässä keskustelussa ylempänä viesteissä, jotka on kirjoittanut nimimerkki lämmön.nettosiirron.teho päivämäärällä 24.10.2018.

        Ei energian säilyvyys tarkoita lämpötilan vakautta, energia voi sitoutua muihinin kohteisiin, kuten muodon tai solurakenteen muutoksiin.


      • Anonyymi
        Anonyymi kirjoitti:

        Ei energian säilyvyys tarkoita lämpötilan vakautta, energia voi sitoutua muihinin kohteisiin, kuten muodon tai solurakenteen muutoksiin.

        Kanan pinta on kutakuinkin musta kappale pitkäaaltoisen lämpösäteilyn aallonpituuksilla. Tasapainotilanteessa 22ºC lämpötilassa se lähettää ja vastaanottaa lämpösäteilyä noin 0.043W teholla jokaista neliösenttimetriä kohti (430 W per neliömetri). Kun lähtevä ja takaisintuleva teho on sama niin edes kappaleen pinnan uloimman kerroksen lämpötila ei muutu.

        Tässä uunin seinämät ovat niin lähellä, että energiaa ei juurikaan pääse varastoitumaan etenevään sähkömagneettiseen säteilyyn. Jos uuni olisi hyvin suuri kanaan verrattuna niin kana jäähtyisi siksi, että lähtevä lämpösäteily jäisi poukkoilemaan seinien välille osumatta kanaan kuin hyvin harvakseltaan eli energiaa varastoituisi sähkömagneettisiin aaltoihin.


      • Anonyymi
        Anonyymi kirjoitti:

        Kanan pinta on kutakuinkin musta kappale pitkäaaltoisen lämpösäteilyn aallonpituuksilla. Tasapainotilanteessa 22ºC lämpötilassa se lähettää ja vastaanottaa lämpösäteilyä noin 0.043W teholla jokaista neliösenttimetriä kohti (430 W per neliömetri). Kun lähtevä ja takaisintuleva teho on sama niin edes kappaleen pinnan uloimman kerroksen lämpötila ei muutu.

        Tässä uunin seinämät ovat niin lähellä, että energiaa ei juurikaan pääse varastoitumaan etenevään sähkömagneettiseen säteilyyn. Jos uuni olisi hyvin suuri kanaan verrattuna niin kana jäähtyisi siksi, että lähtevä lämpösäteily jäisi poukkoilemaan seinien välille osumatta kanaan kuin hyvin harvakseltaan eli energiaa varastoituisi sähkömagneettisiin aaltoihin.

        Tuosta näkee että olet pihalla kuin lintulauta. Väität, että broileri säteilee neliölle saman verran (430W) kuin saadaan kesäaamuna auringosta lämpöä Helsingissä.


      • Anonyymi
        Anonyymi kirjoitti:

        Kanan pinta on kutakuinkin musta kappale pitkäaaltoisen lämpösäteilyn aallonpituuksilla. Tasapainotilanteessa 22ºC lämpötilassa se lähettää ja vastaanottaa lämpösäteilyä noin 0.043W teholla jokaista neliösenttimetriä kohti (430 W per neliömetri). Kun lähtevä ja takaisintuleva teho on sama niin edes kappaleen pinnan uloimman kerroksen lämpötila ei muutu.

        Tässä uunin seinämät ovat niin lähellä, että energiaa ei juurikaan pääse varastoitumaan etenevään sähkömagneettiseen säteilyyn. Jos uuni olisi hyvin suuri kanaan verrattuna niin kana jäähtyisi siksi, että lähtevä lämpösäteily jäisi poukkoilemaan seinien välille osumatta kanaan kuin hyvin harvakseltaan eli energiaa varastoituisi sähkömagneettisiin aaltoihin.

        Pienimmät sähköpatterit ovat 400 wattisia. Lämmittääkö 10 kuollutta kanaa yhtä paljon?


      • Anonyymi
        Anonyymi kirjoitti:

        Pienimmät sähköpatterit ovat 400 wattisia. Lämmittääkö 10 kuollutta kanaa yhtä paljon?

        Kämppään ei siis kannata tuoda liian montaa pakastekanaa, ettei kämmpä lehahda tuleen.


      • Anonyymi
        Anonyymi kirjoitti:

        Kanan pinta on kutakuinkin musta kappale pitkäaaltoisen lämpösäteilyn aallonpituuksilla. Tasapainotilanteessa 22ºC lämpötilassa se lähettää ja vastaanottaa lämpösäteilyä noin 0.043W teholla jokaista neliösenttimetriä kohti (430 W per neliömetri). Kun lähtevä ja takaisintuleva teho on sama niin edes kappaleen pinnan uloimman kerroksen lämpötila ei muutu.

        Tässä uunin seinämät ovat niin lähellä, että energiaa ei juurikaan pääse varastoitumaan etenevään sähkömagneettiseen säteilyyn. Jos uuni olisi hyvin suuri kanaan verrattuna niin kana jäähtyisi siksi, että lähtevä lämpösäteily jäisi poukkoilemaan seinien välille osumatta kanaan kuin hyvin harvakseltaan eli energiaa varastoituisi sähkömagneettisiin aaltoihin.

        Heh, tuolla logiikalla suuren hallin keskelle laitettu 22 asteinen kana pamahtaisi umpijäähän hetkessä, eli absoluuttiseen nollapisteeseen, kun se säteilisi kaiken lämpöenergiansa pois.

        Voi teitä selittelijöitä.

        Painele nyt kiivaasti poistonappia.


      • Anonyymi
        Anonyymi kirjoitti:

        Tuosta näkee että olet pihalla kuin lintulauta. Väität, että broileri säteilee neliölle saman verran (430W) kuin saadaan kesäaamuna auringosta lämpöä Helsingissä.

        Esimerkin 22 - asteinen (295K) broileri säteilee ympäristöönsä noin 430 W jokaista broilerin pinnan neliömetriä kohti. Samaan aikaan 22 - asteinen broilerin ympäristö tuottaa lämpösäteilyä jota absorboituu broiskun pintaan noin 430 W jokaista broilerin pinnan neliömetriä kohti. Lähetetyn ja vastaanotetun tehon erotus on nolla eli lämmön nettosiirron teho on nollassa eli broilerin pinta ei jäähdy eikä lämpene. Kun broilerin pinta on neliömetriä pienempi on säteillyn ja vastaanotetun tehon määrät vastaavasti pienempiä.

        En minä voi sille mitään että et ole koskaan lukenut lämpöoppia edes ammattikoulun saati sitten ammattikorkeakoulun tai yliopiston oppikirjoista. Päätös olla oppimatta asioita on jokaisen oma valinta eikä sitä voi kukaan puolestasi perua.


      • Anonyymi
        Anonyymi kirjoitti:

        Esimerkin 22 - asteinen (295K) broileri säteilee ympäristöönsä noin 430 W jokaista broilerin pinnan neliömetriä kohti. Samaan aikaan 22 - asteinen broilerin ympäristö tuottaa lämpösäteilyä jota absorboituu broiskun pintaan noin 430 W jokaista broilerin pinnan neliömetriä kohti. Lähetetyn ja vastaanotetun tehon erotus on nolla eli lämmön nettosiirron teho on nollassa eli broilerin pinta ei jäähdy eikä lämpene. Kun broilerin pinta on neliömetriä pienempi on säteillyn ja vastaanotetun tehon määrät vastaavasti pienempiä.

        En minä voi sille mitään että et ole koskaan lukenut lämpöoppia edes ammattikoulun saati sitten ammattikorkeakoulun tai yliopiston oppikirjoista. Päätös olla oppimatta asioita on jokaisen oma valinta eikä sitä voi kukaan puolestasi perua.

        Juurihan selitit asian näin:
        "Jos uuni olisi hyvin suuri kanaan verrattuna niin kana jäähtyisi siksi, että lähtevä lämpösäteily jäisi poukkoilemaan seinien välille osumatta kanaan kuin hyvin harvakseltaan eli energiaa varastoituisi sähkömagneettisiin aaltoihin."

        Eli kanaparka säteilisi lämpönsä pois ja se lämpö jäisi poukkoilemaan sinne halliin varastoituneena sähkömagneettisiin aaltoihin.

        Jatka vaan opiskelua, niin saatat päästä sinne ammattikouluun. Suurtalouskokkeja tarvitaan aina.


      • Anonyymi
        Anonyymi kirjoitti:

        Juurihan selitit asian näin:
        "Jos uuni olisi hyvin suuri kanaan verrattuna niin kana jäähtyisi siksi, että lähtevä lämpösäteily jäisi poukkoilemaan seinien välille osumatta kanaan kuin hyvin harvakseltaan eli energiaa varastoituisi sähkömagneettisiin aaltoihin."

        Eli kanaparka säteilisi lämpönsä pois ja se lämpö jäisi poukkoilemaan sinne halliin varastoituneena sähkömagneettisiin aaltoihin.

        Jatka vaan opiskelua, niin saatat päästä sinne ammattikouluun. Suurtalouskokkeja tarvitaan aina.

        Kiitos kun tarjosit syyn kirjoittaa tämä asia esille. Huomasit varmaan lainaamassasi lauseessa kohdan "Jos uuni olisi hyvin suuri kanaan verrattuna".

        Hyvin suurella uunilla tarkoitetaan tässä niin suurta, että merkittävä osa kanan lämpöenergiasta mahtuisi etenevään sähkömagneettiseen aaltoliikkeeseen. Kvanttimekaniikan avulla tätä suuruusluokkaa voi arvioida:

        Mustan kappaleen lähettämän lämpösäteilyn aiheuttama säteilyn energiatiheys u yksiköissä joulea per kuutiometri on Planckin lain perusteella johdettavissa

        u = (4*σ/c)*T^4

        jossa

        σ = Stefan-Bolzmannin vakio 5.67E-8 W/m^2K^4
        c = valon nopeus tyhjiössä 299792458 m/s
        T = lämpösäteilyn lähteen absoluuttinen lämpötila (tässä T=295 K )

        Kaavan löytää googlettamalla englanniksi käännetyillä sanoilla SÄTEILY ENERGIA TIHEYS MUSTA KAPPALE tai lyhytlinkistä urly piste fi kauttaviiva 1o4M

        Tuosta saa laskemalla lämpösäteilyn energiatiheydeksi u = 5.72E-6 J/m^3

        Kokonaisella broilerilla on massaa luokkaa 1.4 kg. Oletetaan että sen lämpökapasiteetti on sama kuin 1.4 kilogrammalla vettä eli 1.4kg*(4186J/kgK) =5.86 kJ/K (vesikilo lämpenee 1 asteella kun sitä lämmitetään 4186 J energialla)

        Jotta kanan lämpötila laskisi esimerkiksi yhden asteen verran on sen säteiltävä ympäristöönsä 5.86 kilojoulea eli 5860 joulea energiaa. Kun tiedetään että lämpösäteilyn energiatiheys u on 22ºC lämpötilassa tuo 5.72E-6 J/m^3 niin mainittu 5860 J energiaa mahtuu lämpösäteilynä tilavuuteen 5860J / 5.72E-6J/m^3 = 1.02E9 m^3 eli vie tilaa aika tarkkaan yhden kuutiokilometrin verran.

        Jos siis broileri olisi keskellä kuutiokilometrin kokoista täydellisen heijastavista peiliseinistä valmistettua tyhjiökammiota niin sen lämpötila voisi laskea yhden asteen verran energian siirtymisestä kyseisen kammion täyttäväksi lämpösäteilyksi. Broilerin lämpötila laskisi aika paljon tuota enemmän broiskun pinnasta tyhjiöön höyrystyvän veden kuljettaman lämmön vuoksi.

        Laskelmassa ei ole huomioitu sitä, että kun kana jäähtyy niin sen lähettämän lämpösäteilyn energiatiheys vastaavasti pienenee. Jos kammio olisi paljon kuutiokilometriä suurempi niin tuo ilmiö hidastaisi jäähtymistä.

        Jos uuni olisikin vain sata metriä kanttiinsa oleva kuutio niin etenevään lämpösäteilyyn mahtuva energia jäähdyttäisi kanaa vaivaiset 0.001 astetta. Kymmenmetrisessä uunissa vaikutus olisi asteen miljoonasosa ja kuutiometrin kokoisessa uunissa asteen miljardisosa. Ammattikoulun keittiössä tästä ilmiöstä ei siis vielä tarvitse huolehtia varsinkin, kun siellä uunin seinät eivät ole täydellisen heijastavia ja uunissa sisällä tyhjiön sijaan ilmaa.


    • Anonyymi

      Ilmastonmuutoksissa pitkät ja lyhyet syklit ratkaisevat mihin suuntaan lämmöt lähtevät. On myös termostaatteja kuten merivirrat jotka tasoittavat tilannetta.

      • Anonyymi

        Arvolan blogissa on sen verran paljon virheitä että suosittelisin lukemaan asiat mieluummin meteorologian tai ilmakehän fysiikan oppikirjasta.


      • Anonyymi
        Anonyymi kirjoitti:

        Arvolan blogissa on sen verran paljon virheitä että suosittelisin lukemaan asiat mieluummin meteorologian tai ilmakehän fysiikan oppikirjasta.

        lue sinä vaan koululainen pitkistä ja lyhyistä sykleistä jotka vaikuttavat ilmastonMUUTOKSIIN.


      • Anonyymi
        Anonyymi kirjoitti:

        Arvolan blogissa on sen verran paljon virheitä että suosittelisin lukemaan asiat mieluummin meteorologian tai ilmakehän fysiikan oppikirjasta.

        Voisit tuonkin todistaa! mitä virheitä?


      • Anonyymi
        Anonyymi kirjoitti:

        Voisit tuonkin todistaa! mitä virheitä?

        Ei se osaa ikinä mitään selittää, kunhan mussuttaa.


      • Anonyymi
        Anonyymi kirjoitti:

        Arvolan blogissa on sen verran paljon virheitä että suosittelisin lukemaan asiat mieluummin meteorologian tai ilmakehän fysiikan oppikirjasta.

        Sinne sivulle voit mennä oikaisemaan kaikki havaitsemasi virheet.
        Niitä odotellessa.

        Taitaa taas olla melkoisen hiljaista.
        Vai paukkuuko täällä poistonappi? Kuuletteko?


      • Anonyymi
        Anonyymi kirjoitti:

        Sinne sivulle voit mennä oikaisemaan kaikki havaitsemasi virheet.
        Niitä odotellessa.

        Taitaa taas olla melkoisen hiljaista.
        Vai paukkuuko täällä poistonappi? Kuuletteko?

        Miksi tuhlata aikaa kun se on Arvolan oma blogi ja virheiden olemassaolosta on turhaan huomauteltu jo vuosien ajan. Jos hänellä on tässä Suomi24 ryhmässä käytävään keskusteluun sanottavaa niin tulkoon sen tänne sanomaan rekisteröidyllä nimimerkillä jonka hän omassa blogissaan varmentaa kuuluvan itselleen.


      • Anonyymi
        Anonyymi kirjoitti:

        Miksi tuhlata aikaa kun se on Arvolan oma blogi ja virheiden olemassaolosta on turhaan huomauteltu jo vuosien ajan. Jos hänellä on tässä Suomi24 ryhmässä käytävään keskusteluun sanottavaa niin tulkoon sen tänne sanomaan rekisteröidyllä nimimerkillä jonka hän omassa blogissaan varmentaa kuuluvan itselleen.

        Eli et kykene osoittamaan mitään virhettä.
        Niin arvelinkin.


    • Anonyymi

      On helppoa tehdä käytännön koe liittyen kappaleen jäähtymiseen lämpösäteilyn avulla tilanteessa, jossa sen ympäristö on kappaletta kylmempi. Tarvikkeet: Yksi pärstävärkki eli kasvot, yksi kaupan pakasteallas.

      Kasvojen ihon lämpötila on korkeampi kuin kaupan huoneenlämpötila. Lämpöä siirtyy koko ajan noin 32 - astesesta kasvojen ihosta kaupan 21 - asteiseen huonetilaan lämpösäteilyn välityksellä. Iho on suunnilleen musta kappale pitkäaaltoisen lämpösäteilyn alueella.

      Kasvot lähettävät ulospäin lämpösäteilyä noin 490 W teholla ihon pinnan neliömetriä kohti. Kaupan huonetilasta absorboituu lämpösäteilyä 425 W teholla ihon pinnan neliömetriä kohti. Erotus eli 65 W per neliömetri on se lämmön nettosiirron teho pinta-alaa kohti joka jäähdyttää kasvoja. Korvaava lämpö tulee ihoon kehon sisältä.

      Mene pakastealtaan luokse. Kylmä ilma on lämmintä ilmaa raskaampaa, joten kun avaat pakastealtaan ei kylmä ilma nouse sieltä ylöspäin kasvoja kohti. Tunnet kuitenkin kasvoissasi selvästi kylmyyden. Syy tuntemukseen on se, että ympäristöstä kasvoihin palaava lämpösäteily on nyt vähäisempää. Kylmä ympäristö säteilee vähemmän tehoa takaisinpäin joten kasvot jäähtyvät. Jään pinta on lämpösäteilyn kannalta katsottuna musta kappale.

      Pakastealtaassa on noin -18 astetta. Avoimena se lähettää lämpösäteilyä vain 240W per neliömetri. Jos olet suoraan altaan yläpuolella niin altaan suunnasta tulee kasvoihin nyt 425W - 240W = 185 W vähemmän tehoa per neliömetri kuin jos allas olisi suljettuna. Lämmön nettosiirron teho onkin nyt 65W/m^2 sijaan 250W/m^2 kasvoista ulospäin. Sen eron tuntee ja havainto on että pakastin huokuu kylmää.

      Kylmemmän ympäristön lämpötila (21 astetta tai -18 astetta) vaikuttaa siihen kuinka paljon on lämmön nettosiirron teho kasvoista ympäristöön.

      • Anonyymi

        Ihmisen tuntoaisti ei tunnista sähkömagneettista säteilyä, vain lämpötila antaa aistihavainnon.
        Säteilynä siirtyvän lämmön aistii vasta kun se on absorboitunut lämmöksi.

        Pakastearkun ja ulkoilman lämpötuntemukset iholla ovat lähes täysin väliaineen, yleensä ilma, siirtämää ja johtamaa lämpöä, säteilyn osuus on marginaalinen.

        Laita sormesi kylmäkaappiin 10 C, tunnet välittömästi viileyden, vaikka säteilylämmön siirtyminen on muutama mW, jota lämpömäärää et sormenpäässäsi kyllä tuntisi.


      • Anonyymi
        Anonyymi kirjoitti:

        Ihmisen tuntoaisti ei tunnista sähkömagneettista säteilyä, vain lämpötila antaa aistihavainnon.
        Säteilynä siirtyvän lämmön aistii vasta kun se on absorboitunut lämmöksi.

        Pakastearkun ja ulkoilman lämpötuntemukset iholla ovat lähes täysin väliaineen, yleensä ilma, siirtämää ja johtamaa lämpöä, säteilyn osuus on marginaalinen.

        Laita sormesi kylmäkaappiin 10 C, tunnet välittömästi viileyden, vaikka säteilylämmön siirtyminen on muutama mW, jota lämpömäärää et sormenpäässäsi kyllä tuntisi.

        Jos et usko oppikirjoihin etkä tajua miten infrapunalämpömittari toimii niin tässä testaus on helppoa.

        Arkkumallisen pakastimen yläpuolella havaitset nimenomaan lämpösäteilystä aiheutuvan eron ihon lämpötasapainossa kun kylmä ilma ei edelleenkään virtaa ylöspäin. Jos haluat testata saman kotioloissa hieman tarkemmin niin sekin onnistuu pienellä vaivalla:

        Vuoraa styroxista valmistettu kylmälaukku alumiinifoliolla. Leikkaa pahvilaatikosta laukulle kanneksi kehikko, joka on suurimmalta osaltaan auki. Peitä avoin osa kehikosta yhdellä kerroksella ohutta tuorekelmua. (elmukelmu). Tuorekelmu läpäisee varsin hyvin myös pitkäaaltoista lämpösäteilyä mutta ei läpäise ilmaa. Tuon läpäisykyvyn voi testata mittaamalla tuorekelmun lävitse kylmän pinnan lämpötilaa infrapunalämpömittarilla. Kelmu muuttaa tuloksia hieman mutta ei paljoa.

        Siirrä pakastimesta -20 asteista tavaraa kylmälaukkuun niin että sen pohja on täynnä pakasteita. Peitä kylmälaukun kansi rakentamallasi kannella. Vie kasvot saman tien laukun yläpuolelle ja tunnet miten pakasteet huokuvat edelleen kylmää vaikka välissä on ilmavirran estävä rakenne. Halutessasi voit tehdä saman kokeen niin että elmukelmusta tehty ilmavirran estävä kansi on suoraan kasvojen edessä ja kylmälaukku avoimena.

        Kun ilmavirta ei aiheuta hermojen havaitsemaa eroa kasvojen ihon ulkokerroksen lämpötilassa on syynä lämpösäteily.

        High - tech versio kokeesta olisi katkoa lämpösäteilyä ilmavirtaa estävän rakenteen takana olevalla varjostimella. Elmukelmu siis kasvojen eteen lämpösäteilyä läpi päästäväksi mutta tuulen estäväksi kypärän visiiriksi ja peitetään näkyvyys pakasteisiin
        muutamaksi sekunniksi pahvinkappaleella. Sitten pahvinkappale sivulle muutamaksi sekunniksi ja tunnetaan kuinka kylmyys alkaa vaikuttaa. Pahvi eteen takaisin ja kasvot lämpenevät.

        Muutamassa sekunnissa edes ilmavirran lämpö ei johtumalla etene kasvojen edessä olevan suojan ja kasvojen välisen ilmakerroksen läpi. Havaittu lämpötilaero aiheutuu siis lämpösäteilystä.

        Helpointa olisi tietenkin lukea samat asiat oppikirjoista ja uskoa mitä niissä on kerrottu mutta se ei ole yhtä opettavaista. Siksi on olemassa labrakursseja joissa jokainen itse tekee ne mittaukset joista kirjoissa esitetään teoria.

        Sinänsähän tässä ei ole mitään uutta kun kaikki huokeat liikeanturit (PIR), infrapunalämpömittarit ja jopa lämpökameratkin toimivat tällä samalla periaatteella. Niissä mittarin sisällä olevan anturin pinnan lämpötila muuttuu hieman sen mukaan kuinka paljon mittarin ympäristöstä tulee anturiin takaisin lämpösäteilyä. Näillä mittareilla ei ole mitään ongelmaa mitata myös anturia kylmemmän pinnan lämpötilaa eli kyllä, kappaletta viileämmän ympäristön lämpötila vaikuttaa siihen kuinka suuri on lämpösäteilyn kautta syntyvä lämmön nettosiirron teho kappaleesta ympäristöön.


      • Anonyymi
        Anonyymi kirjoitti:

        Jos et usko oppikirjoihin etkä tajua miten infrapunalämpömittari toimii niin tässä testaus on helppoa.

        Arkkumallisen pakastimen yläpuolella havaitset nimenomaan lämpösäteilystä aiheutuvan eron ihon lämpötasapainossa kun kylmä ilma ei edelleenkään virtaa ylöspäin. Jos haluat testata saman kotioloissa hieman tarkemmin niin sekin onnistuu pienellä vaivalla:

        Vuoraa styroxista valmistettu kylmälaukku alumiinifoliolla. Leikkaa pahvilaatikosta laukulle kanneksi kehikko, joka on suurimmalta osaltaan auki. Peitä avoin osa kehikosta yhdellä kerroksella ohutta tuorekelmua. (elmukelmu). Tuorekelmu läpäisee varsin hyvin myös pitkäaaltoista lämpösäteilyä mutta ei läpäise ilmaa. Tuon läpäisykyvyn voi testata mittaamalla tuorekelmun lävitse kylmän pinnan lämpötilaa infrapunalämpömittarilla. Kelmu muuttaa tuloksia hieman mutta ei paljoa.

        Siirrä pakastimesta -20 asteista tavaraa kylmälaukkuun niin että sen pohja on täynnä pakasteita. Peitä kylmälaukun kansi rakentamallasi kannella. Vie kasvot saman tien laukun yläpuolelle ja tunnet miten pakasteet huokuvat edelleen kylmää vaikka välissä on ilmavirran estävä rakenne. Halutessasi voit tehdä saman kokeen niin että elmukelmusta tehty ilmavirran estävä kansi on suoraan kasvojen edessä ja kylmälaukku avoimena.

        Kun ilmavirta ei aiheuta hermojen havaitsemaa eroa kasvojen ihon ulkokerroksen lämpötilassa on syynä lämpösäteily.

        High - tech versio kokeesta olisi katkoa lämpösäteilyä ilmavirtaa estävän rakenteen takana olevalla varjostimella. Elmukelmu siis kasvojen eteen lämpösäteilyä läpi päästäväksi mutta tuulen estäväksi kypärän visiiriksi ja peitetään näkyvyys pakasteisiin
        muutamaksi sekunniksi pahvinkappaleella. Sitten pahvinkappale sivulle muutamaksi sekunniksi ja tunnetaan kuinka kylmyys alkaa vaikuttaa. Pahvi eteen takaisin ja kasvot lämpenevät.

        Muutamassa sekunnissa edes ilmavirran lämpö ei johtumalla etene kasvojen edessä olevan suojan ja kasvojen välisen ilmakerroksen läpi. Havaittu lämpötilaero aiheutuu siis lämpösäteilystä.

        Helpointa olisi tietenkin lukea samat asiat oppikirjoista ja uskoa mitä niissä on kerrottu mutta se ei ole yhtä opettavaista. Siksi on olemassa labrakursseja joissa jokainen itse tekee ne mittaukset joista kirjoissa esitetään teoria.

        Sinänsähän tässä ei ole mitään uutta kun kaikki huokeat liikeanturit (PIR), infrapunalämpömittarit ja jopa lämpökameratkin toimivat tällä samalla periaatteella. Niissä mittarin sisällä olevan anturin pinnan lämpötila muuttuu hieman sen mukaan kuinka paljon mittarin ympäristöstä tulee anturiin takaisin lämpösäteilyä. Näillä mittareilla ei ole mitään ongelmaa mitata myös anturia kylmemmän pinnan lämpötilaa eli kyllä, kappaletta viileämmän ympäristön lämpötila vaikuttaa siihen kuinka suuri on lämpösäteilyn kautta syntyvä lämmön nettosiirron teho kappaleesta ympäristöön.

        Mainiota ! !

        Miten tuo alkeiden kertaus liittyy ihmisen tuntoaistiin tai siihen että säteilemällä siirtyvä lämpömäärä on pallomme pinnalla marginaalinen suhteessa muilla tavoilla siirtyvään .


      • Anonyymi
        Anonyymi kirjoitti:

        Mainiota ! !

        Miten tuo alkeiden kertaus liittyy ihmisen tuntoaistiin tai siihen että säteilemällä siirtyvä lämpömäärä on pallomme pinnalla marginaalinen suhteessa muilla tavoilla siirtyvään .

        Ai? Nytkö se kaiken polttava takaisinsäteily onkin marginaalista?


      • Anonyymi
        Anonyymi kirjoitti:

        Jos et usko oppikirjoihin etkä tajua miten infrapunalämpömittari toimii niin tässä testaus on helppoa.

        Arkkumallisen pakastimen yläpuolella havaitset nimenomaan lämpösäteilystä aiheutuvan eron ihon lämpötasapainossa kun kylmä ilma ei edelleenkään virtaa ylöspäin. Jos haluat testata saman kotioloissa hieman tarkemmin niin sekin onnistuu pienellä vaivalla:

        Vuoraa styroxista valmistettu kylmälaukku alumiinifoliolla. Leikkaa pahvilaatikosta laukulle kanneksi kehikko, joka on suurimmalta osaltaan auki. Peitä avoin osa kehikosta yhdellä kerroksella ohutta tuorekelmua. (elmukelmu). Tuorekelmu läpäisee varsin hyvin myös pitkäaaltoista lämpösäteilyä mutta ei läpäise ilmaa. Tuon läpäisykyvyn voi testata mittaamalla tuorekelmun lävitse kylmän pinnan lämpötilaa infrapunalämpömittarilla. Kelmu muuttaa tuloksia hieman mutta ei paljoa.

        Siirrä pakastimesta -20 asteista tavaraa kylmälaukkuun niin että sen pohja on täynnä pakasteita. Peitä kylmälaukun kansi rakentamallasi kannella. Vie kasvot saman tien laukun yläpuolelle ja tunnet miten pakasteet huokuvat edelleen kylmää vaikka välissä on ilmavirran estävä rakenne. Halutessasi voit tehdä saman kokeen niin että elmukelmusta tehty ilmavirran estävä kansi on suoraan kasvojen edessä ja kylmälaukku avoimena.

        Kun ilmavirta ei aiheuta hermojen havaitsemaa eroa kasvojen ihon ulkokerroksen lämpötilassa on syynä lämpösäteily.

        High - tech versio kokeesta olisi katkoa lämpösäteilyä ilmavirtaa estävän rakenteen takana olevalla varjostimella. Elmukelmu siis kasvojen eteen lämpösäteilyä läpi päästäväksi mutta tuulen estäväksi kypärän visiiriksi ja peitetään näkyvyys pakasteisiin
        muutamaksi sekunniksi pahvinkappaleella. Sitten pahvinkappale sivulle muutamaksi sekunniksi ja tunnetaan kuinka kylmyys alkaa vaikuttaa. Pahvi eteen takaisin ja kasvot lämpenevät.

        Muutamassa sekunnissa edes ilmavirran lämpö ei johtumalla etene kasvojen edessä olevan suojan ja kasvojen välisen ilmakerroksen läpi. Havaittu lämpötilaero aiheutuu siis lämpösäteilystä.

        Helpointa olisi tietenkin lukea samat asiat oppikirjoista ja uskoa mitä niissä on kerrottu mutta se ei ole yhtä opettavaista. Siksi on olemassa labrakursseja joissa jokainen itse tekee ne mittaukset joista kirjoissa esitetään teoria.

        Sinänsähän tässä ei ole mitään uutta kun kaikki huokeat liikeanturit (PIR), infrapunalämpömittarit ja jopa lämpökameratkin toimivat tällä samalla periaatteella. Niissä mittarin sisällä olevan anturin pinnan lämpötila muuttuu hieman sen mukaan kuinka paljon mittarin ympäristöstä tulee anturiin takaisin lämpösäteilyä. Näillä mittareilla ei ole mitään ongelmaa mitata myös anturia kylmemmän pinnan lämpötilaa eli kyllä, kappaletta viileämmän ympäristön lämpötila vaikuttaa siihen kuinka suuri on lämpösäteilyn kautta syntyvä lämmön nettosiirron teho kappaleesta ympäristöön.

        Joskus on jopa huvittavaa lukea näitä "keittiöfyysikoiden" käsityksiä fysiikasta.

        "Arkkumallisen pakastimen yläpuolella havaitset nimenomaan lämpösäteilystä aiheutuvan eron ihon lämpötasapainossa kun kylmä ilma ei edelleenkään virtaa ylöspäin. "

        Tämän johdannon jälkeen on aivan yhdentekevää, mitä on jatkossa, koko pohja putoaa jo tähän.

        Aivan aluksi et voi tietää mistään, onko lämmöntunne peräisin säteilystä.
        Väite että kylmä ilma ei voi nousta ylöspäin on pelkkä osaamattomien harhaluulo, jo nostekin syntyy tiheyserosta, eikä ilmamassan liike ole mikään sablunoitu vakio.
        Katso joskus, kun esimerkiksi hiilihappopakastimen kansi avataan, näet jopa omin silmin kuinka tiivistynyt vesihöyry nousee arkusta, on hiivatin kylmää, ja nousee silti ylöspäin, vai ?

        Hieman syventymistä asiaan auttaisi edes saamaan jotain järjellistä, näiden omien harhojen esittelyyn.


      • Anonyymi
        Anonyymi kirjoitti:

        Mainiota ! !

        Miten tuo alkeiden kertaus liittyy ihmisen tuntoaistiin tai siihen että säteilemällä siirtyvä lämpömäärä on pallomme pinnalla marginaalinen suhteessa muilla tavoilla siirtyvään .

        Tämän keskustelun aihe on energian siirtyminen lämpösäteilyn avulla kun siitä on vuosien varrella ollut varsin paljon vääriä kuvitelmia ja harhaluuloja.

        Esitetyn kokeen avulla voi ilman mittalaitteita kuka tahansa varmistua siitä, että kappaleen ulospäin lämpösäteilynä menettämä nettoenergia eli lämmön nettosiirron teho riippuu kappaleen ympäristön lämpötilasta silloinkin, kun ympäristö on kappaletta viileämpi. Tämän huomaa vertailussa huoneenlämpöinen ympäristä vastaan -20 asteinen ympäristö.

        Jos lämmön nettosiirron tehon on jostakin syystä johtuen pysyttävä vakiona niin silloin ympäristön lämmetessä on kappaleenkin lämmettävä lämmön nettosiirron tehon pitämiseksi vakiona.


      • Anonyymi
        Anonyymi kirjoitti:

        Ai? Nytkö se kaiken polttava takaisinsäteily onkin marginaalista?

        Ei nyt.

        Ilmakehän lämpöenergian oskillaatio on aina ollut mahtiosaltaan johtumista ja siirtymistä, säteily on sadelleen marginaalista.

        Vain pallomme ulkopuolinen energiavaihto on kokonaan säteilyn hartioilla.


      • Anonyymi
        Anonyymi kirjoitti:

        Joskus on jopa huvittavaa lukea näitä "keittiöfyysikoiden" käsityksiä fysiikasta.

        "Arkkumallisen pakastimen yläpuolella havaitset nimenomaan lämpösäteilystä aiheutuvan eron ihon lämpötasapainossa kun kylmä ilma ei edelleenkään virtaa ylöspäin. "

        Tämän johdannon jälkeen on aivan yhdentekevää, mitä on jatkossa, koko pohja putoaa jo tähän.

        Aivan aluksi et voi tietää mistään, onko lämmöntunne peräisin säteilystä.
        Väite että kylmä ilma ei voi nousta ylöspäin on pelkkä osaamattomien harhaluulo, jo nostekin syntyy tiheyserosta, eikä ilmamassan liike ole mikään sablunoitu vakio.
        Katso joskus, kun esimerkiksi hiilihappopakastimen kansi avataan, näet jopa omin silmin kuinka tiivistynyt vesihöyry nousee arkusta, on hiivatin kylmää, ja nousee silti ylöspäin, vai ?

        Hieman syventymistä asiaan auttaisi edes saamaan jotain järjellistä, näiden omien harhojen esittelyyn.

        Kun kirjasta toiseen toistuvaa lämmönsiirron oppikirjafysiikkaa kutsutaan harhoiksi niin vikaa joutuu etsimään muualta kuin kyseisistä oppikirjoista. Lämpösäteilyn kautta siirtyvä lämmön nettosiirron teho ja sen riippuvuus kappaleen ja ympäristön lämpötiloista on perusfysiikkaa. Joka aiheesta on kiinnostunut enemmän voi itse lukea asiat tässäkin keskustelussa mainituista oppikirjoista ja käsikirjoista.

        Tämä sama perusasioiden kiistäminen on toki nähty moneen kertaan eri foorumeilla eli ei mikään uusi ilmiö kysymyksessä. Tarkoituksena lienee tuhlata vastausten laatimiseen vaivaa näkevien aikaa.


      • Anonyymi
        Anonyymi kirjoitti:

        Tämän keskustelun aihe on energian siirtyminen lämpösäteilyn avulla kun siitä on vuosien varrella ollut varsin paljon vääriä kuvitelmia ja harhaluuloja.

        Esitetyn kokeen avulla voi ilman mittalaitteita kuka tahansa varmistua siitä, että kappaleen ulospäin lämpösäteilynä menettämä nettoenergia eli lämmön nettosiirron teho riippuu kappaleen ympäristön lämpötilasta silloinkin, kun ympäristö on kappaletta viileämpi. Tämän huomaa vertailussa huoneenlämpöinen ympäristä vastaan -20 asteinen ympäristö.

        Jos lämmön nettosiirron tehon on jostakin syystä johtuen pysyttävä vakiona niin silloin ympäristön lämmetessä on kappaleenkin lämmettävä lämmön nettosiirron tehon pitämiseksi vakiona.

        No ei !

        Millään ihmisaistien kokemuksilla ei voi erottaa sitä, kuinka lämpö on siirtynyt aistittavaksi.

        Kaikenlaisia teorioita voi omaksi huvikseen kehitellä, mutta edelleenkään ihminen ei kykene aistimaan säteilyä.

        Energian siirtyminen säteilemällä on yksi lämpöopin perusteita, mutta joku raja on siinä, mihin kaikkeen sitä yritetään sotkea.


      • Anonyymi
        Anonyymi kirjoitti:

        Jos et usko oppikirjoihin etkä tajua miten infrapunalämpömittari toimii niin tässä testaus on helppoa.

        Arkkumallisen pakastimen yläpuolella havaitset nimenomaan lämpösäteilystä aiheutuvan eron ihon lämpötasapainossa kun kylmä ilma ei edelleenkään virtaa ylöspäin. Jos haluat testata saman kotioloissa hieman tarkemmin niin sekin onnistuu pienellä vaivalla:

        Vuoraa styroxista valmistettu kylmälaukku alumiinifoliolla. Leikkaa pahvilaatikosta laukulle kanneksi kehikko, joka on suurimmalta osaltaan auki. Peitä avoin osa kehikosta yhdellä kerroksella ohutta tuorekelmua. (elmukelmu). Tuorekelmu läpäisee varsin hyvin myös pitkäaaltoista lämpösäteilyä mutta ei läpäise ilmaa. Tuon läpäisykyvyn voi testata mittaamalla tuorekelmun lävitse kylmän pinnan lämpötilaa infrapunalämpömittarilla. Kelmu muuttaa tuloksia hieman mutta ei paljoa.

        Siirrä pakastimesta -20 asteista tavaraa kylmälaukkuun niin että sen pohja on täynnä pakasteita. Peitä kylmälaukun kansi rakentamallasi kannella. Vie kasvot saman tien laukun yläpuolelle ja tunnet miten pakasteet huokuvat edelleen kylmää vaikka välissä on ilmavirran estävä rakenne. Halutessasi voit tehdä saman kokeen niin että elmukelmusta tehty ilmavirran estävä kansi on suoraan kasvojen edessä ja kylmälaukku avoimena.

        Kun ilmavirta ei aiheuta hermojen havaitsemaa eroa kasvojen ihon ulkokerroksen lämpötilassa on syynä lämpösäteily.

        High - tech versio kokeesta olisi katkoa lämpösäteilyä ilmavirtaa estävän rakenteen takana olevalla varjostimella. Elmukelmu siis kasvojen eteen lämpösäteilyä läpi päästäväksi mutta tuulen estäväksi kypärän visiiriksi ja peitetään näkyvyys pakasteisiin
        muutamaksi sekunniksi pahvinkappaleella. Sitten pahvinkappale sivulle muutamaksi sekunniksi ja tunnetaan kuinka kylmyys alkaa vaikuttaa. Pahvi eteen takaisin ja kasvot lämpenevät.

        Muutamassa sekunnissa edes ilmavirran lämpö ei johtumalla etene kasvojen edessä olevan suojan ja kasvojen välisen ilmakerroksen läpi. Havaittu lämpötilaero aiheutuu siis lämpösäteilystä.

        Helpointa olisi tietenkin lukea samat asiat oppikirjoista ja uskoa mitä niissä on kerrottu mutta se ei ole yhtä opettavaista. Siksi on olemassa labrakursseja joissa jokainen itse tekee ne mittaukset joista kirjoissa esitetään teoria.

        Sinänsähän tässä ei ole mitään uutta kun kaikki huokeat liikeanturit (PIR), infrapunalämpömittarit ja jopa lämpökameratkin toimivat tällä samalla periaatteella. Niissä mittarin sisällä olevan anturin pinnan lämpötila muuttuu hieman sen mukaan kuinka paljon mittarin ympäristöstä tulee anturiin takaisin lämpösäteilyä. Näillä mittareilla ei ole mitään ongelmaa mitata myös anturia kylmemmän pinnan lämpötilaa eli kyllä, kappaletta viileämmän ympäristön lämpötila vaikuttaa siihen kuinka suuri on lämpösäteilyn kautta syntyvä lämmön nettosiirron teho kappaleesta ympäristöön.

        Jos se kylmä ilma pysyy siellä arkkumallisessa pakastimessa niin miksi valmistaja on laittanut siihen pakastimeen kannen.


      • Anonyymi
        Anonyymi kirjoitti:

        Jos se kylmä ilma pysyy siellä arkkumallisessa pakastimessa niin miksi valmistaja on laittanut siihen pakastimeen kannen.

        Oliskohan siksi, että kansi estää ohikulkijoiden ja huoneen ilmanvaihdon aiheuttamia ilmavirtoja liikuttamasta lämmintä ilmaa pakastimen ilmatilan yläosaan ja vastaavasti kylmää ilmaa pakastimen ilmatilasta reunan yli lattialle. Samalla se vähentää ympäristön lämpösäteilystä pakastimeen sisälle päätyvää lämpöä. Kuten tuossa aiemmin laskeskeltiin niin lämmön nettosiirron tehoa on ympäristöstä tarjolla noin 250 W per pakastimen lokeron avoin pinta-ala. Toisin kun lämmin huoneilma tuo lämpösäteily kantaa pakastimen pohjalle asti.

        On näitä pakastelaareja joskus nähty käytössä ihan avoiminakin mutta kuluttavat silloin tietenkin paljon enemmän energiaa.


      • Anonyymi
        Anonyymi kirjoitti:

        Oliskohan siksi, että kansi estää ohikulkijoiden ja huoneen ilmanvaihdon aiheuttamia ilmavirtoja liikuttamasta lämmintä ilmaa pakastimen ilmatilan yläosaan ja vastaavasti kylmää ilmaa pakastimen ilmatilasta reunan yli lattialle. Samalla se vähentää ympäristön lämpösäteilystä pakastimeen sisälle päätyvää lämpöä. Kuten tuossa aiemmin laskeskeltiin niin lämmön nettosiirron tehoa on ympäristöstä tarjolla noin 250 W per pakastimen lokeron avoin pinta-ala. Toisin kun lämmin huoneilma tuo lämpösäteily kantaa pakastimen pohjalle asti.

        On näitä pakastelaareja joskus nähty käytössä ihan avoiminakin mutta kuluttavat silloin tietenkin paljon enemmän energiaa.

        Olet edelleenkin pihalla. Säteilyn lämmönsiirtoteho alhaisissa lämpötiloissa eli esimerkiksi huoneenlämmössä on pieni verrattuna johtumalla ja kuljettumalla tapahtuvaan lämmönsiirtoon. Ei tuollaista 250 W nettosiirtotehoa säteilyssä pakastimeen päin ole olemassa. Väitit, että ihmisen iho säteilee lämpöä 490 wattia neliölle. Ihmisen ihon pinta-ala voi olla 2 neliömetriä, joten väitteesi mukaan ihminen säteilee n. 1000 wattia lämpöä. Kun ihmisen lämmönhukasta vain puolet on säteilyä niin väitteesi mukaan ihminen lämmittää yhtä paljon kuin 2000 watin lämpöpatteri, joten olet pahasti pielessä säteilyhömpötyksinesi.


      • Anonyymi
        Anonyymi kirjoitti:

        Olet edelleenkin pihalla. Säteilyn lämmönsiirtoteho alhaisissa lämpötiloissa eli esimerkiksi huoneenlämmössä on pieni verrattuna johtumalla ja kuljettumalla tapahtuvaan lämmönsiirtoon. Ei tuollaista 250 W nettosiirtotehoa säteilyssä pakastimeen päin ole olemassa. Väitit, että ihmisen iho säteilee lämpöä 490 wattia neliölle. Ihmisen ihon pinta-ala voi olla 2 neliömetriä, joten väitteesi mukaan ihminen säteilee n. 1000 wattia lämpöä. Kun ihmisen lämmönhukasta vain puolet on säteilyä niin väitteesi mukaan ihminen lämmittää yhtä paljon kuin 2000 watin lämpöpatteri, joten olet pahasti pielessä säteilyhömpötyksinesi.

        En osaa auttaa sinua kun et halua lukea mitä oppikirjassa lämpösäteilystä kerrotaan. Se ei tietenkään ole minun ongelmani kun en onneksi ole vastuussa sinun osaamisestasi tai opettamisestasi.

        Älä jatkossa koe tarpeelliseksi vastailla sellaisiin viesteihin joita et ymmärrä ja joiden ymmärtäminen edellyttäisi oppikirjojen lukemista. Tuhlaat vain omaa aikaasi.


      • Anonyymi
        Anonyymi kirjoitti:

        En osaa auttaa sinua kun et halua lukea mitä oppikirjassa lämpösäteilystä kerrotaan. Se ei tietenkään ole minun ongelmani kun en onneksi ole vastuussa sinun osaamisestasi tai opettamisestasi.

        Älä jatkossa koe tarpeelliseksi vastailla sellaisiin viesteihin joita et ymmärrä ja joiden ymmärtäminen edellyttäisi oppikirjojen lukemista. Tuhlaat vain omaa aikaasi.

        Jospa kaataisit itsellesi vaan.

        Jos lukee jonkun kirjan, joka käsittelee spesifisesti jotain aihetta, niin olisi ymmärrettävä kuinka se suhtautuu todellisuuteen ja mikä on sen merkitys.

        Säteilyn ja johtumisen välistä suuruussuhdetta voit testata itse menemällä vaikka 40 asteen pakkaseen seisomaan.
        Ihmiskehon ja ulkoilman välisen lämpötilan mukaan säteilylämpöä pitäisi siirtyä 350 W/m², myös lämpimästi pukeutuneena.
        Ilman vaatetusta kylmä ilma olisi kosketuksessa ihoon ja lämpö siirtyisi johtumalla, väittäisin että ero on aika selvä !

        Lisäksi koko keskustelua häiritsee yhden osa-alueen väärä painotus.
        Jos on lukenut S-B-yhtälön ja soveltaa sitä sellaisenaan vain kahden kappaleen välille, on ymmärtänyt asian väärin.
        Teoria on teoriaa, käytännössä säteilyä on joka paikassa ja kaikenlaista, aina avaruuden taustasäteilystä lähtien, ne kaikki kuljettavat energiaa täysin kontrolloimattomassa sekasotkussa, eli pakastekaappi ja "säteilyn tunnistaja" eivät ole yksin, tai pysty havainnoimaan varsinaisesti vain toistensa tai muutakaan säteilyä.


      • Anonyymi
        Anonyymi kirjoitti:

        Jospa kaataisit itsellesi vaan.

        Jos lukee jonkun kirjan, joka käsittelee spesifisesti jotain aihetta, niin olisi ymmärrettävä kuinka se suhtautuu todellisuuteen ja mikä on sen merkitys.

        Säteilyn ja johtumisen välistä suuruussuhdetta voit testata itse menemällä vaikka 40 asteen pakkaseen seisomaan.
        Ihmiskehon ja ulkoilman välisen lämpötilan mukaan säteilylämpöä pitäisi siirtyä 350 W/m², myös lämpimästi pukeutuneena.
        Ilman vaatetusta kylmä ilma olisi kosketuksessa ihoon ja lämpö siirtyisi johtumalla, väittäisin että ero on aika selvä !

        Lisäksi koko keskustelua häiritsee yhden osa-alueen väärä painotus.
        Jos on lukenut S-B-yhtälön ja soveltaa sitä sellaisenaan vain kahden kappaleen välille, on ymmärtänyt asian väärin.
        Teoria on teoriaa, käytännössä säteilyä on joka paikassa ja kaikenlaista, aina avaruuden taustasäteilystä lähtien, ne kaikki kuljettavat energiaa täysin kontrolloimattomassa sekasotkussa, eli pakastekaappi ja "säteilyn tunnistaja" eivät ole yksin, tai pysty havainnoimaan varsinaisesti vain toistensa tai muutakaan säteilyä.

        Katsopa mitä löydät kun googletat hakusanoilla

        NETTOMÄÄRÄINEN LÄMMÖN SIIRTYMINEN PINNALTA TOISELLE

        Siitä on hyvä aloittaa. Kun perusasiat ovat hallussa (Stefan-Boltzmannin laki, lämpösäteily myös kylmemmiltä pinnoilta kuumemman pinnan suuntaan) niin asioiden ymmärtäminen on paljon helpompaa.


      • Anonyymi
        Anonyymi kirjoitti:

        En osaa auttaa sinua kun et halua lukea mitä oppikirjassa lämpösäteilystä kerrotaan. Se ei tietenkään ole minun ongelmani kun en onneksi ole vastuussa sinun osaamisestasi tai opettamisestasi.

        Älä jatkossa koe tarpeelliseksi vastailla sellaisiin viesteihin joita et ymmärrä ja joiden ymmärtäminen edellyttäisi oppikirjojen lukemista. Tuhlaat vain omaa aikaasi.

        Sinä väitit: "Kasvot lähettävät ulospäin lämpösäteilyä noin 490 W teholla ihon pinnan neliömetriä kohti" Eli kun ihmisessä on ihoa n. 2 neliötä niin säteilyteho olisi noin 1000 wattia. Netin ihmeellisestä maailmasta löytyy tieto jonka mukaan keskikokoisen ihmisen aineenvaihdunnan teho, eli se teho millä ihminen myös lämmittää ympäristöään:
        nukkuessa 85W
        toimistotyö 105-125W
        kotityöt 165W
        Tuosta näkee, että olet todella hukassa. Heitä oppikirjasi roskikseen ja ala miettimään asiaa uudelleen käytännön pohjalta.


      • Anonyymi
        Anonyymi kirjoitti:

        Katsopa mitä löydät kun googletat hakusanoilla

        NETTOMÄÄRÄINEN LÄMMÖN SIIRTYMINEN PINNALTA TOISELLE

        Siitä on hyvä aloittaa. Kun perusasiat ovat hallussa (Stefan-Boltzmannin laki, lämpösäteily myös kylmemmiltä pinnoilta kuumemman pinnan suuntaan) niin asioiden ymmärtäminen on paljon helpompaa.

        Kuten jo totesin, kaada noviisi vain itsellesi.

        Ei kaikkien tarvitse kaivella Wikipediasta jo aikaa tiedossa olleita lämpöopin alkeita.
        Sinulle on nyt käynyt kuten edellä kuvasin, kun pohjatieto kokonaisuudesta on puutteellinen, niin täytyy hirttäytyä nyansseihin ja paloihin joistain lukemistaan erikoistapauksista joita löytää internetistä.

        Usko kun sinua neuvotaan, kaikki aineet eivät ole verrattavissa mustaan kappaleeseen, kaikkien emiessiivisyys ei ole 1 ja mikään kappale ei ole ainoana säteilijänä tässä universumissa.

        Jutuistasi päätellen kuulut tyypillisesti joukkoon jolta koulut on aikoinaan jäänyt matalammille portaille, ja nyt kun olet löytänyt suppeita jutunpätkiä aiheesta, olet mielestäsi oivaltanut jotain ainutlaatuista ja huomiota hakeaksesi esittelet väärinkäsityksiäsi, ilmeisesti olettaen että kaikki palstan lukijat ovat kaltaisiasi yksinkertaisia wannabefyysikoita.

        Ehkäpä joku muu palsta olisi sopivampi reuhaamisellesi, täällä käy joskus aikuisia ihmisiäkin.


      • Anonyymi
        Anonyymi kirjoitti:

        Katsopa mitä löydät kun googletat hakusanoilla

        NETTOMÄÄRÄINEN LÄMMÖN SIIRTYMINEN PINNALTA TOISELLE

        Siitä on hyvä aloittaa. Kun perusasiat ovat hallussa (Stefan-Boltzmannin laki, lämpösäteily myös kylmemmiltä pinnoilta kuumemman pinnan suuntaan) niin asioiden ymmärtäminen on paljon helpompaa.

        Minäkin puutun keskusteluun kertomalla sähköpatteristani, jonka koko on n. 1*0.4 m ja yksilevyinen, teho 1500W.

        Jos oletetaan että huonelämpötila olisi 15 C, niin tehon siirtyminen säteilemällä S-B säännön mukaan edellyttäisi että pintalämpötilan olisi oltava 180 C, joten kyllä suurin osa lämmöstä siirtyy johtumalla ja siirtymällä, siksi myös ilmavirran estämiseksi peittokielto.
        Sisätiloissa pintalämmön olisi oltava alle 60 C, joten säteilyteho olisi alle 300 W ja sekään ei ole kuin laskennallinen säteilyteho, lämmön siirtoteho on sitten taas eri juttu.
        Kun tietää, kuinka vähän ilma absorboi tuota matalaa lämpösäteilyä, niin lukipa siellä Google haussa mitä tahansa, niin aikuisten maailmassa säteilylämmön osuus on marginaalinen, me kun olemme aina jonkinlaisen aineen ympäröimänä.


      • Anonyymi
        Anonyymi kirjoitti:

        Sinä väitit: "Kasvot lähettävät ulospäin lämpösäteilyä noin 490 W teholla ihon pinnan neliömetriä kohti" Eli kun ihmisessä on ihoa n. 2 neliötä niin säteilyteho olisi noin 1000 wattia. Netin ihmeellisestä maailmasta löytyy tieto jonka mukaan keskikokoisen ihmisen aineenvaihdunnan teho, eli se teho millä ihminen myös lämmittää ympäristöään:
        nukkuessa 85W
        toimistotyö 105-125W
        kotityöt 165W
        Tuosta näkee, että olet todella hukassa. Heitä oppikirjasi roskikseen ja ala miettimään asiaa uudelleen käytännön pohjalta.

        Kasvot lähettävät lämpösäteilyä ympäristöön jollakin teholla. Millä teholla ympäristön lämpösäteily tuo lämpöä takaisin kasvoihin? Lähtevän ja takaisintulevan säteilyn tehojen erotus kertoo millä teholla lämpö siirtyy kasvoista niitä viileämpään ympäristöön. Sitä erotusta kutsutaan nimellä lämmön nettomääräisen siirtymisen teho.


      • Anonyymi
        Anonyymi kirjoitti:

        Kasvot lähettävät lämpösäteilyä ympäristöön jollakin teholla. Millä teholla ympäristön lämpösäteily tuo lämpöä takaisin kasvoihin? Lähtevän ja takaisintulevan säteilyn tehojen erotus kertoo millä teholla lämpö siirtyy kasvoista niitä viileämpään ympäristöön. Sitä erotusta kutsutaan nimellä lämmön nettomääräisen siirtymisen teho.

        Vedetäänpäs poikaset välillä vähän henkeä.

        Tuo pitää paikkansa suljetussa tilanteessa, jossa molemmat kappaleet käyttäytyvät mustan kappaleen tavoin, ovat niin lähellä toisiaan että säteilyä ei suuntaudu ohi, eikä välillä ole mitään materiaa tai kaasua, joka osallistuisi säteilyn kulkuun tai tehoon.

        Käytännössä tällaisen tilanne on mahdoton, kaikki säteilijät säteilevät joka suuntaan, ja säteilyn intensiteetti(teho/ala) pienenee etäisyyden neliöön, ja matkalla kaikki välillä olevat aineet osallistuvat omalla osallaan tapahtumiin.

        Ei tarvita kovinkaan suurta neroutta ymmärtämään että tilanne on niin kaoottinen että täytyy olla tosi yksinkertainen esittäessään lukuarvoja kuinka kasvojen säteily vaikuttaa pakastinarkkuun tai muuta yhtä älytöntä.


      • Anonyymi
        Anonyymi kirjoitti:

        Vedetäänpäs poikaset välillä vähän henkeä.

        Tuo pitää paikkansa suljetussa tilanteessa, jossa molemmat kappaleet käyttäytyvät mustan kappaleen tavoin, ovat niin lähellä toisiaan että säteilyä ei suuntaudu ohi, eikä välillä ole mitään materiaa tai kaasua, joka osallistuisi säteilyn kulkuun tai tehoon.

        Käytännössä tällaisen tilanne on mahdoton, kaikki säteilijät säteilevät joka suuntaan, ja säteilyn intensiteetti(teho/ala) pienenee etäisyyden neliöön, ja matkalla kaikki välillä olevat aineet osallistuvat omalla osallaan tapahtumiin.

        Ei tarvita kovinkaan suurta neroutta ymmärtämään että tilanne on niin kaoottinen että täytyy olla tosi yksinkertainen esittäessään lukuarvoja kuinka kasvojen säteily vaikuttaa pakastinarkkuun tai muuta yhtä älytöntä.

        En oikein ymmärrä mitä ongelmaa tässä nyt on. Energian siirtyminen lämpösäteilyn avulla on teollisuuden sovelluksissa varsin arkipäiväinen asia ja on sekä oppikirjatasolla että käsikirjoissa selitetty useaan kertaan. Tuossa ylempänä esille otettu esimerkki kasvoista on ihan tuon saman periaatteen mukaisesti laskettu. Esimerkiksi ylempänä viestissä Aesdifaeger 30.06.2019 22:07 mainitussa Blundellien oppikirjassa asia on selkeästi käsitelty.

        Oletko sitä mieltä että oppikirjoissa esitetty tapa käsitellä lämpösäteilyyn liittyvää lämmönsiirtoa on väärä? Sellainen mielipide tarvitsisi perusteluikseen muutakin kuin vain mutua kun näitä laskukaavoja kuitenkin käytetään koko ajan teollisuustuotteiden ja tutkimuskäytössä olevien laitteiden suunnittelussa.

        Pintojen näkyvyyteen liittyvät korjaustekijät on tietenkin otettava huomioon kun geometria on monimutkaisempi. Sitä varten käytetään simulointiohjelmia tai valmiita taulukkokirjojen kaavoja. Perustilanne on kuitenkin aina sama Stefan-Boltzmannin lain mukainen: Kuuma kappale siirtää lämpöä ympäristöönsä lämpösäteilyn kautta ja viileämpi ympäristö siirtää lämpöä takaisin takaisin kappaleeseen lämpösäteily avulla lämmön nettomääräisen siirtymisen tehon ollessa näiden kahden erotus.


      • Anonyymi
        Anonyymi kirjoitti:

        En oikein ymmärrä mitä ongelmaa tässä nyt on. Energian siirtyminen lämpösäteilyn avulla on teollisuuden sovelluksissa varsin arkipäiväinen asia ja on sekä oppikirjatasolla että käsikirjoissa selitetty useaan kertaan. Tuossa ylempänä esille otettu esimerkki kasvoista on ihan tuon saman periaatteen mukaisesti laskettu. Esimerkiksi ylempänä viestissä Aesdifaeger 30.06.2019 22:07 mainitussa Blundellien oppikirjassa asia on selkeästi käsitelty.

        Oletko sitä mieltä että oppikirjoissa esitetty tapa käsitellä lämpösäteilyyn liittyvää lämmönsiirtoa on väärä? Sellainen mielipide tarvitsisi perusteluikseen muutakin kuin vain mutua kun näitä laskukaavoja kuitenkin käytetään koko ajan teollisuustuotteiden ja tutkimuskäytössä olevien laitteiden suunnittelussa.

        Pintojen näkyvyyteen liittyvät korjaustekijät on tietenkin otettava huomioon kun geometria on monimutkaisempi. Sitä varten käytetään simulointiohjelmia tai valmiita taulukkokirjojen kaavoja. Perustilanne on kuitenkin aina sama Stefan-Boltzmannin lain mukainen: Kuuma kappale siirtää lämpöä ympäristöönsä lämpösäteilyn kautta ja viileämpi ympäristö siirtää lämpöä takaisin takaisin kappaleeseen lämpösäteily avulla lämmön nettomääräisen siirtymisen tehon ollessa näiden kahden erotus.

        Ei asiassa mitään epäselvää ole, olet vain ohittanut sen että on eri asia käsitellä ilmiötä erillisenä tapahtumana kuin sen esiintymistä osana muussa yhteydessä.
        Reaalimaailmassa ei ole esimerkin tapaista kahden kohteen keskinäistä säteilytehonsiirtoa, koska kaikki aineet säteilevät lämpösäteilyä joka suuntaan ja ovat osana kokonaisuudessa.

        Teollisuuden käyttö on eri asia, emission kiihdytys on tunnettu jo kauan (vrt laser).


      • Anonyymi
        Anonyymi kirjoitti:

        Ei asiassa mitään epäselvää ole, olet vain ohittanut sen että on eri asia käsitellä ilmiötä erillisenä tapahtumana kuin sen esiintymistä osana muussa yhteydessä.
        Reaalimaailmassa ei ole esimerkin tapaista kahden kohteen keskinäistä säteilytehonsiirtoa, koska kaikki aineet säteilevät lämpösäteilyä joka suuntaan ja ovat osana kokonaisuudessa.

        Teollisuuden käyttö on eri asia, emission kiihdytys on tunnettu jo kauan (vrt laser).

        Kirjoitat emission kiihdytyksestä jolla ei ole mitään asian kanssa tekemistä. Yrität tehdä asiasta monimutkaista voidaksesi väittää oppikirjojen ja käsikirjojen olevan väärässä. Ei se mitään, niinhän se alipainehitsarikin on tehnyt jo pitkän aikaa. Sille harva enää vastailee.

        Jatkossa kun taas esität tunnetun fysiikan vastaisia kommentteja on vastauksen puuttuminen merkki siitä että kukaan ei viitsi vastata. Kun näköjään torjut tiedon niin turha sitä on sinulle tyrkyttää. Viesteihisi tietoa sisältävät vastaukset eivät ole osoitettuja sinulle vaan mahdollisille muille lukijoille.


      • Anonyymi
        Anonyymi kirjoitti:

        Kirjoitat emission kiihdytyksestä jolla ei ole mitään asian kanssa tekemistä. Yrität tehdä asiasta monimutkaista voidaksesi väittää oppikirjojen ja käsikirjojen olevan väärässä. Ei se mitään, niinhän se alipainehitsarikin on tehnyt jo pitkän aikaa. Sille harva enää vastailee.

        Jatkossa kun taas esität tunnetun fysiikan vastaisia kommentteja on vastauksen puuttuminen merkki siitä että kukaan ei viitsi vastata. Kun näköjään torjut tiedon niin turha sitä on sinulle tyrkyttää. Viesteihisi tietoa sisältävät vastaukset eivät ole osoitettuja sinulle vaan mahdollisille muille lukijoille.

        Vielä tämän mainitsen. Jospa ihmettelijät ja tieteen kiistäjät kokeilisitte lukea mitä Blundellien oppikirjassa kerrotaan lämpösäteilystä. Tai vilkaisisitte mitä Lienhard kirjoittaa omassa lämmönsiirron oppikirjassaan jonka hän on laittanut kaikkien vapaasti saataville nettiin MIT:n verkkosivustolle:

        https://ahtt.mit.edu/

        Lämpösäteilyn alkeiden perusteet löytyvät luvussa 1 alkaen sivulta 26 jatkuen sivulle 34. Siellä on näyttää olevan ainakin yksi versio tuosta aiemmin esittämästäni pakastimen lämpösäteilyn havaitsemisesta. Ympäristön ja kappaleen lämpötilojen neljänsien potenssien erotus tulee esille kaavassa (1.31), jossa siis lasketaan lämmön nettomääräisen siirron teho. Varsinainen lämpösäteilyä käsittelevä yksityiskohtaisempi esitys on sivuilla 537 - 612.

        Kaikki tämä on yliopistotason peruskurssien asiaa.


      • Anonyymi
        Anonyymi kirjoitti:

        Vedetäänpäs poikaset välillä vähän henkeä.

        Tuo pitää paikkansa suljetussa tilanteessa, jossa molemmat kappaleet käyttäytyvät mustan kappaleen tavoin, ovat niin lähellä toisiaan että säteilyä ei suuntaudu ohi, eikä välillä ole mitään materiaa tai kaasua, joka osallistuisi säteilyn kulkuun tai tehoon.

        Käytännössä tällaisen tilanne on mahdoton, kaikki säteilijät säteilevät joka suuntaan, ja säteilyn intensiteetti(teho/ala) pienenee etäisyyden neliöön, ja matkalla kaikki välillä olevat aineet osallistuvat omalla osallaan tapahtumiin.

        Ei tarvita kovinkaan suurta neroutta ymmärtämään että tilanne on niin kaoottinen että täytyy olla tosi yksinkertainen esittäessään lukuarvoja kuinka kasvojen säteily vaikuttaa pakastinarkkuun tai muuta yhtä älytöntä.

        Minä peesaan sinua. Nuo teoriauskovaiset ja oppikirjauskovaiset on vaikeita tapauksia. Joku joskus muinoin on yrittänyt selittää jotakin, ja se avuton yritys on edelleen oppikirjoissa.

        Sitten myöhemmin on tullut insinööri tai jokin muu keksijä, ja keksinyt käytännön sovellutuksen, joka on ristiriidassa sen vanhan teorian kanssa. Teoriauskovainen ei suostu edes uskomaan, että sellainen keksintö on olemasssa.

        Oletkohan väitellyt samaisen tyypin kanssa, jonka kanssa minä huvikseni väittelin muutamia kuukausia. Ei se oppinut yhtään mitään, mutta minulla nyt on tätä aikaa. Minuun se viittaa tuossa myöhemminj.

        Mikähän siinä on, että jotku ei uskalla yhtään kyseenalaistaa vanhoja teorioita.
        Kyllä minulla ainakin käytännön kokemuksen pohjalta on se käsitys, että kylmä nimenomaan absorboi lämpöä eikä säteile lämpöä.

        Kylmäkin kyllä säteilee lämpöä vielä kylmempään päin, mutta ei lämpimämpään päin. Vaikka yläilmakehä on kylmä. se säteilee lämpöä avaruuteen, joka on vielä paljon kylmempi. Todellisuus tässä maailmassa ratkaisee eikä pöljät vanhat teoriat.

        Tuo alipainehitsari ym nimittely on oikein typeryyden huippu. Ilmeisesti kyseinen tyyppi on niin totaalisen tietämätön, ettei edes tiedä että molemmat on olemassa. Oman tietämättömkyytensä ylentäminen tietämykseksi on typeryyden huippu.

        Luin pari päivää sitten Terveyskirjasto Duodecimistä sivupersoonahäiriöstä. Se sopisi tuollaisiin, jotka kieltää todellisuuden. Sellaisessa tilassa ihminen saattaa olla aivan irrallaan todellisuudesta, eikä muista siitä jälkenpäin yhtään mitään.

        Sivupersoonalla, joka pomppaa esille on aivan erilaiset muistot kuin pääpersoonalla. Pääpersoona ei edes tiedä sivupersoonan olemassaolosta. Sen tietää vain ne joihin sivupersoonan inhottava toiminta kohdistuu.

        Tuollaisia sivupersoonia kehittää lapsuudessa ja nuoruudessa koetut traumat. Olen muutaman tuollaisen kohdannut elämäni aikana. Ne on varsin ikäviä kokemuksia, kun asianomainen ei itse edes muista, mitä on tehnyt. Kohde kyllä taatusti muistaa.

        Valitettavasti uskon, että ilmastohourinta aiheuttaa joillekin niin pahan kriisin, että heille kehittyy sivuopersoonahäiriö. Sellaisen kanssa on mahdotonta käydä keskustelua.

        Sivupersoonja tulee päälle muutamassa sekunnissa, ja ajatukset kiertää aina samaa kehää, ja suhtautuminen on vihamielistä. Asiallinen keskustelu on mahdotonta.


    • Anonyymi

      Ilmastonmuutoksissa on tärkeintä syklisyys ja sen vaikutus ilmastoon.
      Muulla ei ole niin suurta merkitystä.

    • Anonyymi

      Muistutetaan lämpösäteilyn roolista ilmakehän ilmiöissä.

      • Anonyymi

        Muistutetaan keskustelusta. Tässä yhteydessä muistutan myös siitä, että lämpösäteily on sähkömagneettista säteilyä ja etenee siten valon nopeudella toisin kuin kaasussa molekyylien törmäysten välityksellä siirtyvä lämpö.

        Lisää tietoa löytyy hakusanoilla

        lämpösäteily wikipedia


      • Anonyymi
        Anonyymi kirjoitti:

        Muistutetaan keskustelusta. Tässä yhteydessä muistutan myös siitä, että lämpösäteily on sähkömagneettista säteilyä ja etenee siten valon nopeudella toisin kuin kaasussa molekyylien törmäysten välityksellä siirtyvä lämpö.

        Lisää tietoa löytyy hakusanoilla

        lämpösäteily wikipedia

        Joku on nostanut vanhan keskustelun samaan aiheeseen liittyen.

        https://keskustelu.suomi24.fi/t/10173953/takaisinsateily---kollimaattori-haastettu

        Siellä toisteltu tuttua mikroaaltouunivertausta, josta hukassa on lämmän nettosiirron teho - käsite.


      • Anonyymi

    • Anonyymi

      Lämpösäteilyn roolia ilmakehän ilmiöissä käsittelee vuodelta 2002 oleva artikkeli 10.1016/S0074-6142(02)80016-X

      jonka siis löytää sci-hub - palvelun kautta luettavaksi:

      https://sci-hub.st/10.1016/S0074-6142(02)80016-X

      Tuo sci-hub - sivuston osoite kannattaa käydä tarkistamassa wikipediasta sci-hub:in kohdalta kun se vaihtuu ajoittain.

    • Anonyymi
      • Anonyymi

        Mikä idea on linkittää toiseen ketjuun?


      • Anonyymi
        Anonyymi kirjoitti:

        Mikä idea on linkittää toiseen ketjuun?

        No tietysti se, että siellä alarmistijönssi taas inttää kaikkea sitä, mistä ei mitään ymmärrä.


      • Anonyymi
        Anonyymi kirjoitti:

        No tietysti se, että siellä alarmistijönssi taas inttää kaikkea sitä, mistä ei mitään ymmärrä.

        Lämmön nettosiirto on ihan asiaa, mutta ei sen selittäminen kokonaista ketjua vaadi.

        Jokainen yli 0 K:n lämpötilassa oleva kappale säteilee infrapunasäteilyä.
        Kuumemman kappaleen säteilyn intensiteetti on suurempaa.
        Jäähtyykö vai lämpeääkö kappale säteilyvirrassa, riippuu kappaleen säteilyn nettovirrasta, eli kappaleen absorboiman ja emittoivan säteilytehon erosta.

        Mihin tässä pitkää ketjua tarvittiin?


      • Anonyymi
        Anonyymi kirjoitti:

        Lämmön nettosiirto on ihan asiaa, mutta ei sen selittäminen kokonaista ketjua vaadi.

        Jokainen yli 0 K:n lämpötilassa oleva kappale säteilee infrapunasäteilyä.
        Kuumemman kappaleen säteilyn intensiteetti on suurempaa.
        Jäähtyykö vai lämpeääkö kappale säteilyvirrassa, riippuu kappaleen säteilyn nettovirrasta, eli kappaleen absorboiman ja emittoivan säteilytehon erosta.

        Mihin tässä pitkää ketjua tarvittiin?

        "Mihin tässä pitkää ketjua tarvittiin?"

        Samaa minäkin ihmettelen ja olen kuitenkin itse se joka ketjun aloitti. Lämpösäteilyn ja aineen vuorovaikutukseen tuntuu liittyvän monilla runsaasti väärinkäsityksiä, joita on usein käytetty johtamaan keskustelijoita harhaan. Näiden väärinkäsitysten korjaaminen lähdeviitteet mainiten on kohtuullisen työlästä joten sen tekeminen uudelleen ja uudelleen eri sanoin olisi ajan tuhlaamista.

        Tähän keskusteluun olen kerännyt lämpösäteilyyn liittyvää tietoa ja linkannut mukaan niitä keskusteluita joissa lämpösäteilyyn liittyviä asioita on viime aikoina käsitelty. Näin samoja asioita ei tarvitse kirjoittaa auki joka kerta erikseen vaan lukijat voivat halutessaan käydä katsomassa kootusti mitä asiasta on viimeisten muutaman vuoden aikana kirjoiteltu.

        Jos aihe ei kiinnosta niin kukaan ei pakota avaamaan näitä linkkejä. Trollauksen kannalta katsottuna tälläisten keskustelukoosteiden olemassaolo on tietenkin ikävää mutta he joutuvat siihen tottumaan.


      • Anonyymi
        Anonyymi kirjoitti:

        "Mihin tässä pitkää ketjua tarvittiin?"

        Samaa minäkin ihmettelen ja olen kuitenkin itse se joka ketjun aloitti. Lämpösäteilyn ja aineen vuorovaikutukseen tuntuu liittyvän monilla runsaasti väärinkäsityksiä, joita on usein käytetty johtamaan keskustelijoita harhaan. Näiden väärinkäsitysten korjaaminen lähdeviitteet mainiten on kohtuullisen työlästä joten sen tekeminen uudelleen ja uudelleen eri sanoin olisi ajan tuhlaamista.

        Tähän keskusteluun olen kerännyt lämpösäteilyyn liittyvää tietoa ja linkannut mukaan niitä keskusteluita joissa lämpösäteilyyn liittyviä asioita on viime aikoina käsitelty. Näin samoja asioita ei tarvitse kirjoittaa auki joka kerta erikseen vaan lukijat voivat halutessaan käydä katsomassa kootusti mitä asiasta on viimeisten muutaman vuoden aikana kirjoiteltu.

        Jos aihe ei kiinnosta niin kukaan ei pakota avaamaan näitä linkkejä. Trollauksen kannalta katsottuna tälläisten keskustelukoosteiden olemassaolo on tietenkin ikävää mutta he joutuvat siihen tottumaan.

        Luin palstaa ensimmäisiä kertoja kai jo kymmenisen vuotta sitten. Silloin joku vielä väitti lämpösäteilyn kulkevan vain kuumasta kylmään. Kyllästyttyäni ylipitkään vääntöön päivänselvästä asiasta jätin palstan seuraamisen vain silloin tällöin tapahtuviin vilkaisuihin.

        Yhteen aikaan jankutettiin Taka-Hikiän männäviikon lämpötiloista, niin kuin sillä olisi globaalin ilmaston kannalta mitään tekemistä. Sitäkin jatkui kuitenkin useana vuotena.
        Jos palstalla pitää palata kymmenen vuoden takaisiin ketjuihin, niin onko silloin opittu yhtään mitään?

        Toisaalta täällä on ainakin yksi kirjoittaja, joka kuvittelee varsin suuria omista tiedoistaan, vaikka hänen fysiikan tiedoillaan ei läpäisisi ede peruskoulua. Sellaisten selvästi höperöiden dementikkojen kohdalla ei toivoa oppimisesta enää ole.
        Kyse onkin lähinnä siitä, onko tällä ilmastonmuutoksen vastustajissa enää jäljellä pelkästään höperöt. Sitä se alkaa muuallakin maailmassa vaikuttaa. Ne ilmastonmuutosta epäilleet henkilöt, joilla on kyky omaksua uutta ja päivittää mielipiteitään, ovat sen todennäköisesti jo tehneet.

        Mark Twain: "Älä alennu keskustelemaan idioottien kanssa samalla tasolla. Häviät varmasti, koska heillä on siitä elinikäinen kokemus."


      • Anonyymi
        Anonyymi kirjoitti:

        Luin palstaa ensimmäisiä kertoja kai jo kymmenisen vuotta sitten. Silloin joku vielä väitti lämpösäteilyn kulkevan vain kuumasta kylmään. Kyllästyttyäni ylipitkään vääntöön päivänselvästä asiasta jätin palstan seuraamisen vain silloin tällöin tapahtuviin vilkaisuihin.

        Yhteen aikaan jankutettiin Taka-Hikiän männäviikon lämpötiloista, niin kuin sillä olisi globaalin ilmaston kannalta mitään tekemistä. Sitäkin jatkui kuitenkin useana vuotena.
        Jos palstalla pitää palata kymmenen vuoden takaisiin ketjuihin, niin onko silloin opittu yhtään mitään?

        Toisaalta täällä on ainakin yksi kirjoittaja, joka kuvittelee varsin suuria omista tiedoistaan, vaikka hänen fysiikan tiedoillaan ei läpäisisi ede peruskoulua. Sellaisten selvästi höperöiden dementikkojen kohdalla ei toivoa oppimisesta enää ole.
        Kyse onkin lähinnä siitä, onko tällä ilmastonmuutoksen vastustajissa enää jäljellä pelkästään höperöt. Sitä se alkaa muuallakin maailmassa vaikuttaa. Ne ilmastonmuutosta epäilleet henkilöt, joilla on kyky omaksua uutta ja päivittää mielipiteitään, ovat sen todennäköisesti jo tehneet.

        Mark Twain: "Älä alennu keskustelemaan idioottien kanssa samalla tasolla. Häviät varmasti, koska heillä on siitä elinikäinen kokemus."

        Itse vastailen näihin silloin kun siihen on samanaikaisesti kiinnostusta ja aikaa. Jätän yleensä kokonaan huomioimatta kaksi keskustelijoiden kategoriaa: Ne jotka ovat puhtaasti troIlaamassa ja ne jotka ovat ylivertaisuusharhan vallassa.

        Kun vastaan aikaani käyttäen niin vastaan siksi, että se on hyvä tekosyy itse opiskella lisää aiheeseeen liittyvää. Lukijoitahan näillä Suomi24 keskusteluilla täällä on minimaalinen määrä kiitos moderoinnin, jossa asiallisia viestejä poist(at)etaan ja asiattomat jäävät palstalle.

        Googlen hakukoneessa nämä viestit kuitenkin näkyvät ulkopuolellekin, joten hyvä vastaus ainakin teoriassa voisi olla jollekulle hyödyksi. Siksi verkostoin keskustelut niin että jos tänne eksyy voi aiempaan tai samanaikaiseen viittaavien linkkien löytää suoraan saman aiheen käsittelyä. Kun siis tällä palstalla ei historiaa pääse selaamaan kuin 30 sivua taaksepäin, mikä tällä hetkellä vastaa noin yhtä vuotta ja varhaisinta näkyvää päivämäärää 23.11.2019.

        Jos katsoo keskusteluiden saamien lukukertojen määriä niin aika puhdasta ajanhukkaahan tänne kirjoittelu on. Varsinkin, kun vanhemmista keskusteluista suuri osa lukukerroista on ollut hakukoneiden robottien vierailuita.


      • Anonyymi
        Anonyymi kirjoitti:

        Itse vastailen näihin silloin kun siihen on samanaikaisesti kiinnostusta ja aikaa. Jätän yleensä kokonaan huomioimatta kaksi keskustelijoiden kategoriaa: Ne jotka ovat puhtaasti troIlaamassa ja ne jotka ovat ylivertaisuusharhan vallassa.

        Kun vastaan aikaani käyttäen niin vastaan siksi, että se on hyvä tekosyy itse opiskella lisää aiheeseeen liittyvää. Lukijoitahan näillä Suomi24 keskusteluilla täällä on minimaalinen määrä kiitos moderoinnin, jossa asiallisia viestejä poist(at)etaan ja asiattomat jäävät palstalle.

        Googlen hakukoneessa nämä viestit kuitenkin näkyvät ulkopuolellekin, joten hyvä vastaus ainakin teoriassa voisi olla jollekulle hyödyksi. Siksi verkostoin keskustelut niin että jos tänne eksyy voi aiempaan tai samanaikaiseen viittaavien linkkien löytää suoraan saman aiheen käsittelyä. Kun siis tällä palstalla ei historiaa pääse selaamaan kuin 30 sivua taaksepäin, mikä tällä hetkellä vastaa noin yhtä vuotta ja varhaisinta näkyvää päivämäärää 23.11.2019.

        Jos katsoo keskusteluiden saamien lukukertojen määriä niin aika puhdasta ajanhukkaahan tänne kirjoittelu on. Varsinkin, kun vanhemmista keskusteluista suuri osa lukukerroista on ollut hakukoneiden robottien vierailuita.

        Tuo itseopiskelu on hyvä syy. Kun törmää yllättäviin vääriin väitteisiin, niin helpointa on sivuttaa ne huuhaana. Mutta jos perustelee väitteen esittäjälle, miksi väite on väärä, joutuu itsekin miettimään asiaa pintaa syvemmältä. On eri asia tietää jokin ja kyetä opettamaan sama. Opettaminen vaatii syvempää sisäistämistä, kuin pelkkä tieto asiasta.

        Joillekin ihmisille opettaminen on jo mahdotonta. Heidät kannattaa jättää omaan arvoonsa.


      • Anonyymi
        Anonyymi kirjoitti:

        Tuo itseopiskelu on hyvä syy. Kun törmää yllättäviin vääriin väitteisiin, niin helpointa on sivuttaa ne huuhaana. Mutta jos perustelee väitteen esittäjälle, miksi väite on väärä, joutuu itsekin miettimään asiaa pintaa syvemmältä. On eri asia tietää jokin ja kyetä opettamaan sama. Opettaminen vaatii syvempää sisäistämistä, kuin pelkkä tieto asiasta.

        Joillekin ihmisille opettaminen on jo mahdotonta. Heidät kannattaa jättää omaan arvoonsa.

        Korjaan. Aina voi opettaa, osa ihmisistä ei vaan kykene oppimaan. Tuo alipainehitsari on selvästi yksi niistä.


      • Anonyymi
        Anonyymi kirjoitti:

        Tuo itseopiskelu on hyvä syy. Kun törmää yllättäviin vääriin väitteisiin, niin helpointa on sivuttaa ne huuhaana. Mutta jos perustelee väitteen esittäjälle, miksi väite on väärä, joutuu itsekin miettimään asiaa pintaa syvemmältä. On eri asia tietää jokin ja kyetä opettamaan sama. Opettaminen vaatii syvempää sisäistämistä, kuin pelkkä tieto asiasta.

        Joillekin ihmisille opettaminen on jo mahdotonta. Heidät kannattaa jättää omaan arvoonsa.

        Minä olen joskus karsinut vastailuani vaatimalla kysymysiä esittävää tekemään itsekin työtä eli lukemaan läksynsä. Jos tai siis kun hän ei osoita tutustuneensa edellisessä vastauksessa annettuun materiaaliin niin en jatka vastailua hänen esittämiinsä kysymyksiin. Tämä karsii niitä kyselijöitä (troIleja) joilla tavoitteena on tuhlata asiallisesti vastaajien aikaa turhaan kirjoitteluun.


      • Anonyymi
        Anonyymi kirjoitti:

        Minä olen joskus karsinut vastailuani vaatimalla kysymysiä esittävää tekemään itsekin työtä eli lukemaan läksynsä. Jos tai siis kun hän ei osoita tutustuneensa edellisessä vastauksessa annettuun materiaaliin niin en jatka vastailua hänen esittämiinsä kysymyksiin. Tämä karsii niitä kyselijöitä (troIleja) joilla tavoitteena on tuhlata asiallisesti vastaajien aikaa turhaan kirjoitteluun.

        Hyvä periaate. Itsepäisenä ihmisenä minun on kuitenkin vaikea jättää ketjun "viimeiseksi sanaksi" väite, joka on aivan päin honkia. Samoja kysymyksi papukaijan lailla toistelevat on helpompi jättää omiin oloihinsa.


      • Anonyymi
        Anonyymi kirjoitti:

        Hyvä periaate. Itsepäisenä ihmisenä minun on kuitenkin vaikea jättää ketjun "viimeiseksi sanaksi" väite, joka on aivan päin honkia. Samoja kysymyksi papukaijan lailla toistelevat on helpompi jättää omiin oloihinsa.

        Tuosta pois oppiminen ottaa aikansa: "Itsepäisenä ihmisenä minun on kuitenkin vaikea jättää ketjun "viimeiseksi sanaksi" väite, joka on aivan päin honkia.

        Sen taidon kyllä oppii ajan kuluessa vietettyään riittävän monta kertaa osana keskustelua pilaavaa ikiliikkujaa. Katso kuinka paljon aikaa käytit ja katso oliko vastailusta hyötyä muille kuin sille joka haluaa karkottaa lukijoita. Minusta troIIin esittämä virheellinen väite viimeisenä on vähemmän haitallinen kuin keskustelun täyttyminen kinastelusta trolIin kanssa. Viimeisen sanan pääsee sanomaan vain omassa blogissaan.


      • Anonyymi
        Anonyymi kirjoitti:

        Tuosta pois oppiminen ottaa aikansa: "Itsepäisenä ihmisenä minun on kuitenkin vaikea jättää ketjun "viimeiseksi sanaksi" väite, joka on aivan päin honkia.

        Sen taidon kyllä oppii ajan kuluessa vietettyään riittävän monta kertaa osana keskustelua pilaavaa ikiliikkujaa. Katso kuinka paljon aikaa käytit ja katso oliko vastailusta hyötyä muille kuin sille joka haluaa karkottaa lukijoita. Minusta troIIin esittämä virheellinen väite viimeisenä on vähemmän haitallinen kuin keskustelun täyttyminen kinastelusta trolIin kanssa. Viimeisen sanan pääsee sanomaan vain omassa blogissaan.

        Onko alipainehitsari trolli vai trollin kanssa jankuttaja vai jotain ihan muuta?


      • Anonyymi
        Anonyymi kirjoitti:

        Onko alipainehitsari trolli vai trollin kanssa jankuttaja vai jotain ihan muuta?

        On se.


      • Anonyymi
        Anonyymi kirjoitti:

        Onko alipainehitsari trolli vai trollin kanssa jankuttaja vai jotain ihan muuta?

        Ylivertaisuusharhasta on kysymys eli Dunning-Kruger tapaus.


    • Anonyymi

      Ilmakehän kaasujen lämpösäteilystä alkoi keskustelu fysiikka - palstalla. Vastauksissa mainittu tuo kaukokartoitusartikkeli ja Howell, Daun, Siegel, Mengüç Radiation heat transfer 7th edition (2021) kappale 9.4, molemmat linkkeineen joista artikkelin ja kirjan saa ladattua.

      https://keskustelu.suomi24.fi/t/17035856/avustusanomus-

      • Lämpösäteilyä voi lähettää kappale tai aine, jolla on nollasta poikkeavaa emissiivisyyttä jollakin aallonpituudella. Mustalla kappaleella emissiivisyys ε on ykkönen kaikilla aallonpituuksilla. Mikään kappale tai aine ei termodynaamisessa tasapainotilassa ollessaan lähetä lämpösäteilyä millään aallonpituudella enempää kuin samassa lämpötilassa oleva musta kappale.

        Mustan kappaleen lähettämän lämpösäteilyn intensiteetti U(λ,T) avaruuskulmaa (steradiaania) ja aallonpituutta kohti on Planckin lain mukaan tunnetusti

        U(λ,T) = (2hc²/λ⁵)/(exp(hc/λkT)-1),

        missä c on valon nopeus, h on Planckin vakio (h=6.62607015×10⁻³⁴ J/Hz), k on Boltzmannin vakio (k=1.38064852×10⁻²³ J/K), T lämpötila kelvineinä ja λ aallonpituus metreinä.

        Steradiaani (sr) on avaruuskulman yksikkö ja joka suuntaan katsottu avaruuskulma. Pallon keskipisteeseen joka suunnasta näkyvän pallopinnan avaruuskulma on 4π suuruinen eli noin 12.566 steradiaania.

        Tuohon lukuarvot sijoittamalla on helppo nähdä, että huoneenlämpöisen mustan kappaleen emissioteho U ultraviolettialueella on häviävän pieni eli vain UV-alueen tai näkyvän valon emissioviivoja omaavat kaasut eivät pysty säteilemään lämpösäteilyä huoneenlämmössä. Lyhytaaltoisen infrapunan alueella olevistakaan viivoista ei ole merkittävää apua.

        Jos summaa (eli integroi) tuon U(λ,T) yli kaikkien aallonpituuksien ja yli koko avaruuskulman saa lopputulokseksi tutun Stefan-Boltzmannin lain eli säteilijän ulospäin näkyvää pinta-alaa kohti mustan kappaleen lähettämän säteilyn intensiteetti on J (wattia per neliömetri)

        J = σT⁴, jossa Stefan-Boltzmannin vakio σ on

        σ = 2π⁵k⁴/15c²h³ eli likimain 5.670374×10⁻⁸ W/m²K⁴

        Wienin siirtymälain mukaan mukaan mustan kappaleen spektrin maksimi-intensiteetti sattuu lämpötilassa T aallonpituudelle λₒ = 0.002898/T jossa T on lämpötila kelvineinä ja λₒ maksimi-intensiteetin aallonpituus metreinä. Kun lämpötila on 293 K niin maksimin aallonpituus on 0.00000989m eli 9.89µm.

        Nämä kaikki ovat mustan kappaleen säteilyyn liittyviä asioita, jotka käsitellään yliopistotason oppikirjoissa kirjasta riippuen suppeammin tai laajemmin.


      • Aesdifaeger kirjoitti:

        Lämpösäteilyä voi lähettää kappale tai aine, jolla on nollasta poikkeavaa emissiivisyyttä jollakin aallonpituudella. Mustalla kappaleella emissiivisyys ε on ykkönen kaikilla aallonpituuksilla. Mikään kappale tai aine ei termodynaamisessa tasapainotilassa ollessaan lähetä lämpösäteilyä millään aallonpituudella enempää kuin samassa lämpötilassa oleva musta kappale.

        Mustan kappaleen lähettämän lämpösäteilyn intensiteetti U(λ,T) avaruuskulmaa (steradiaania) ja aallonpituutta kohti on Planckin lain mukaan tunnetusti

        U(λ,T) = (2hc²/λ⁵)/(exp(hc/λkT)-1),

        missä c on valon nopeus, h on Planckin vakio (h=6.62607015×10⁻³⁴ J/Hz), k on Boltzmannin vakio (k=1.38064852×10⁻²³ J/K), T lämpötila kelvineinä ja λ aallonpituus metreinä.

        Steradiaani (sr) on avaruuskulman yksikkö ja joka suuntaan katsottu avaruuskulma. Pallon keskipisteeseen joka suunnasta näkyvän pallopinnan avaruuskulma on 4π suuruinen eli noin 12.566 steradiaania.

        Tuohon lukuarvot sijoittamalla on helppo nähdä, että huoneenlämpöisen mustan kappaleen emissioteho U ultraviolettialueella on häviävän pieni eli vain UV-alueen tai näkyvän valon emissioviivoja omaavat kaasut eivät pysty säteilemään lämpösäteilyä huoneenlämmössä. Lyhytaaltoisen infrapunan alueella olevistakaan viivoista ei ole merkittävää apua.

        Jos summaa (eli integroi) tuon U(λ,T) yli kaikkien aallonpituuksien ja yli koko avaruuskulman saa lopputulokseksi tutun Stefan-Boltzmannin lain eli säteilijän ulospäin näkyvää pinta-alaa kohti mustan kappaleen lähettämän säteilyn intensiteetti on J (wattia per neliömetri)

        J = σT⁴, jossa Stefan-Boltzmannin vakio σ on

        σ = 2π⁵k⁴/15c²h³ eli likimain 5.670374×10⁻⁸ W/m²K⁴

        Wienin siirtymälain mukaan mukaan mustan kappaleen spektrin maksimi-intensiteetti sattuu lämpötilassa T aallonpituudelle λₒ = 0.002898/T jossa T on lämpötila kelvineinä ja λₒ maksimi-intensiteetin aallonpituus metreinä. Kun lämpötila on 293 K niin maksimin aallonpituus on 0.00000989m eli 9.89µm.

        Nämä kaikki ovat mustan kappaleen säteilyyn liittyviä asioita, jotka käsitellään yliopistotason oppikirjoissa kirjasta riippuen suppeammin tai laajemmin.

        Katsotaan edellisen perusteella miten kaasun spektriviivojen sijainti vaikuttaa kaasun emissiviteettiin matalassa lämpötilassa. Spektriviivojensa kohdalla kaasu voi paksuna kerroksena olla korkeintaan yhtä tehokas säteilijä kuin musta kappale. Spektriviivojen välisellä aallonpituusalueella kaasu ei säteile käytännössä lainkaan. Tehdään siis oletus, että kaasu käyttäytyy spektriviivojensa kohdalla kuten musta kappale (paksu kerros kaasua). Tu oletus on Howellin oppikirjassa erään laskentamenetelmistä perusteena.

        Planckin lakia on ikävä pyöritellä ilman taulukkolaskentaohjelmaa. Esimerkiksi tällä sivulla olevalla laskurilla saa Planckin säteilylaista mustan kappaleen spektrin intensiteetit esille ja voi laskea myös valitulle aallonpituusvälille osuvan säteilyn tehon.

        https://www.spectralcalc.com/blackbody_calculator/blackbody.php

        Otetaan huoneenlämpöinen kappale (20 astetta) eli T=293K ja emissiivisyys ε=1 eli musta kappale. Kun valitsee aallonpituusalueeksi ultravioletista hyvin pitkäaaltoiseen infrapunaan eli 0.01µm (lower limit) ja 1000µm (upper limit) niin säteilytehoa tuolle välille tulee 133.027W/m² steradiaania kohti. Tuo vastaa käytännössä koko mustan kappaleen spektriä.

        Kun valitsee aallonpituusalueeksi 0.4 µm ja 2 µm eli lyhytaaltoisen infrapunan ja näkyvän valon niin säteilytehoksi tulee 0.0000074 W/m² steradiaania kohti.
        Kun valitsee aallonpituusalueeksi 0.01 µm ja 0.4 µm eli ultravioletin säteilyn niin säteilytehoksi tulee 1.9E-46 W/m² steradiaania kohti. Tuo tarkoittaa lukua, jossa on desimaalipisteen jälkeen 45 nollaa ennenkuin tulee ensimmäinen nollasta poikkeava numero.

        On siis varsin selvää, että huoneenlämmössä olevan pelkkiä UV-alueen absorptioviivoja omaavan kaasun (N₂) emissiviteetti lämpösäteilylle on niin paljon pienempi kuin mustan kappaleen emissiviteetti että käytännössä sitä voi pitää nollan suuruisena.

        Hapella (O₂) sen sijaan on emissioviivoja noin 1.26...1.28 µm ja 2µm paikkeilla (2.02 ... 2.06 µm), kts esim https://hal-insu.archives-ouvertes.fr/insu-02151491/document

        Mustan kappaleen spektrin säteilyteho on tuon laskurin mukaan välillä 2.02...2.06 µm vain 4.8E-6 W/m² steradiaania kohti. Jos happi käyttäytyisi tuolla spektrin välillä kuten täydellisen musta kappale (mitä se ei tee) niin hyvin paksun happikerroksen emissiviteetti ε huoneenlämmössä voisi olla enintään

        ε = 4.8E-6/133.027 = 3.6E-8 = 0.000000036

        Paksun happikerroksen läpäisevyys huoneenlämpötilassa olevan pinnan ulospäin lähtevälle mustan kappaleen lämpösäteilylle olisi siis 1-0.000000037 eli 99.999996%.

        Paksun typpikerroksen läpäisevyys samaiselle lämpösäteilylle olisi 99.9999999999999...% eli sen emissiviteetti ε olisi hyvin tarkkaan nollan suuruinen.

        On siis varsin selvää, että happi ja typpi eivät Maapallon pinnan ulospäin lähettämää säteilyä pysäytä pinnan lämpöbudjetin ja ilmaston kannalta katsottuna merkittävissä määrin. Alaspäin tulevan Auringon säteilyn osalta hapen lähi-infrapuna-alueen absorptiolla sen sijaan on jonkin verran vaikutusta.

        Ilmakehän hiilidioksidin emissiviteetin käsittely huoneenlämmössä tuolla yksinkertaistetulla tavalla olisi sikäli hankalaa että a) kaasulla on selkeästi Maapallon pintaa vastaavan mustan kappaleen spektrin alueelle osuvia absorptioviivoja ja b) ilmakehässä oleva hiilidioksidi ei laskelmien kannalta katsottuna ole paksu kun sen jokainen absorptioviiva ympäristöineen ei ole saturoitunut. Howellin kirjassa on laskettu sivulla 426 esimerkki 9.3 kuuman (830K), hyvin paksun ja 10 ilmakehän paineessa olevan hiilidioksidikerroksen emissiviteetistä yksinkertaistetun mallin mukaisesti.


      • Anonyymi
        Aesdifaeger kirjoitti:

        Katsotaan edellisen perusteella miten kaasun spektriviivojen sijainti vaikuttaa kaasun emissiviteettiin matalassa lämpötilassa. Spektriviivojensa kohdalla kaasu voi paksuna kerroksena olla korkeintaan yhtä tehokas säteilijä kuin musta kappale. Spektriviivojen välisellä aallonpituusalueella kaasu ei säteile käytännössä lainkaan. Tehdään siis oletus, että kaasu käyttäytyy spektriviivojensa kohdalla kuten musta kappale (paksu kerros kaasua). Tu oletus on Howellin oppikirjassa erään laskentamenetelmistä perusteena.

        Planckin lakia on ikävä pyöritellä ilman taulukkolaskentaohjelmaa. Esimerkiksi tällä sivulla olevalla laskurilla saa Planckin säteilylaista mustan kappaleen spektrin intensiteetit esille ja voi laskea myös valitulle aallonpituusvälille osuvan säteilyn tehon.

        https://www.spectralcalc.com/blackbody_calculator/blackbody.php

        Otetaan huoneenlämpöinen kappale (20 astetta) eli T=293K ja emissiivisyys ε=1 eli musta kappale. Kun valitsee aallonpituusalueeksi ultravioletista hyvin pitkäaaltoiseen infrapunaan eli 0.01µm (lower limit) ja 1000µm (upper limit) niin säteilytehoa tuolle välille tulee 133.027W/m² steradiaania kohti. Tuo vastaa käytännössä koko mustan kappaleen spektriä.

        Kun valitsee aallonpituusalueeksi 0.4 µm ja 2 µm eli lyhytaaltoisen infrapunan ja näkyvän valon niin säteilytehoksi tulee 0.0000074 W/m² steradiaania kohti.
        Kun valitsee aallonpituusalueeksi 0.01 µm ja 0.4 µm eli ultravioletin säteilyn niin säteilytehoksi tulee 1.9E-46 W/m² steradiaania kohti. Tuo tarkoittaa lukua, jossa on desimaalipisteen jälkeen 45 nollaa ennenkuin tulee ensimmäinen nollasta poikkeava numero.

        On siis varsin selvää, että huoneenlämmössä olevan pelkkiä UV-alueen absorptioviivoja omaavan kaasun (N₂) emissiviteetti lämpösäteilylle on niin paljon pienempi kuin mustan kappaleen emissiviteetti että käytännössä sitä voi pitää nollan suuruisena.

        Hapella (O₂) sen sijaan on emissioviivoja noin 1.26...1.28 µm ja 2µm paikkeilla (2.02 ... 2.06 µm), kts esim https://hal-insu.archives-ouvertes.fr/insu-02151491/document

        Mustan kappaleen spektrin säteilyteho on tuon laskurin mukaan välillä 2.02...2.06 µm vain 4.8E-6 W/m² steradiaania kohti. Jos happi käyttäytyisi tuolla spektrin välillä kuten täydellisen musta kappale (mitä se ei tee) niin hyvin paksun happikerroksen emissiviteetti ε huoneenlämmössä voisi olla enintään

        ε = 4.8E-6/133.027 = 3.6E-8 = 0.000000036

        Paksun happikerroksen läpäisevyys huoneenlämpötilassa olevan pinnan ulospäin lähtevälle mustan kappaleen lämpösäteilylle olisi siis 1-0.000000037 eli 99.999996%.

        Paksun typpikerroksen läpäisevyys samaiselle lämpösäteilylle olisi 99.9999999999999...% eli sen emissiviteetti ε olisi hyvin tarkkaan nollan suuruinen.

        On siis varsin selvää, että happi ja typpi eivät Maapallon pinnan ulospäin lähettämää säteilyä pysäytä pinnan lämpöbudjetin ja ilmaston kannalta katsottuna merkittävissä määrin. Alaspäin tulevan Auringon säteilyn osalta hapen lähi-infrapuna-alueen absorptiolla sen sijaan on jonkin verran vaikutusta.

        Ilmakehän hiilidioksidin emissiviteetin käsittely huoneenlämmössä tuolla yksinkertaistetulla tavalla olisi sikäli hankalaa että a) kaasulla on selkeästi Maapallon pintaa vastaavan mustan kappaleen spektrin alueelle osuvia absorptioviivoja ja b) ilmakehässä oleva hiilidioksidi ei laskelmien kannalta katsottuna ole paksu kun sen jokainen absorptioviiva ympäristöineen ei ole saturoitunut. Howellin kirjassa on laskettu sivulla 426 esimerkki 9.3 kuuman (830K), hyvin paksun ja 10 ilmakehän paineessa olevan hiilidioksidikerroksen emissiviteetistä yksinkertaistetun mallin mukaisesti.

        Jotain ongelmia mielenterveyden kanssa, vai ?

        Jotenkin ymmärrettävää että joillekin omat kirjoituksensa ovat niin mieluisia että ne on nostettava lähes viikoittain ja useilla palstoilla, mutta tapaisesi "riehunta" ei enää ole lähelläkään normaalia käyttäytymistä.


      • Anonyymi
        Aesdifaeger kirjoitti:

        Katsotaan edellisen perusteella miten kaasun spektriviivojen sijainti vaikuttaa kaasun emissiviteettiin matalassa lämpötilassa. Spektriviivojensa kohdalla kaasu voi paksuna kerroksena olla korkeintaan yhtä tehokas säteilijä kuin musta kappale. Spektriviivojen välisellä aallonpituusalueella kaasu ei säteile käytännössä lainkaan. Tehdään siis oletus, että kaasu käyttäytyy spektriviivojensa kohdalla kuten musta kappale (paksu kerros kaasua). Tu oletus on Howellin oppikirjassa erään laskentamenetelmistä perusteena.

        Planckin lakia on ikävä pyöritellä ilman taulukkolaskentaohjelmaa. Esimerkiksi tällä sivulla olevalla laskurilla saa Planckin säteilylaista mustan kappaleen spektrin intensiteetit esille ja voi laskea myös valitulle aallonpituusvälille osuvan säteilyn tehon.

        https://www.spectralcalc.com/blackbody_calculator/blackbody.php

        Otetaan huoneenlämpöinen kappale (20 astetta) eli T=293K ja emissiivisyys ε=1 eli musta kappale. Kun valitsee aallonpituusalueeksi ultravioletista hyvin pitkäaaltoiseen infrapunaan eli 0.01µm (lower limit) ja 1000µm (upper limit) niin säteilytehoa tuolle välille tulee 133.027W/m² steradiaania kohti. Tuo vastaa käytännössä koko mustan kappaleen spektriä.

        Kun valitsee aallonpituusalueeksi 0.4 µm ja 2 µm eli lyhytaaltoisen infrapunan ja näkyvän valon niin säteilytehoksi tulee 0.0000074 W/m² steradiaania kohti.
        Kun valitsee aallonpituusalueeksi 0.01 µm ja 0.4 µm eli ultravioletin säteilyn niin säteilytehoksi tulee 1.9E-46 W/m² steradiaania kohti. Tuo tarkoittaa lukua, jossa on desimaalipisteen jälkeen 45 nollaa ennenkuin tulee ensimmäinen nollasta poikkeava numero.

        On siis varsin selvää, että huoneenlämmössä olevan pelkkiä UV-alueen absorptioviivoja omaavan kaasun (N₂) emissiviteetti lämpösäteilylle on niin paljon pienempi kuin mustan kappaleen emissiviteetti että käytännössä sitä voi pitää nollan suuruisena.

        Hapella (O₂) sen sijaan on emissioviivoja noin 1.26...1.28 µm ja 2µm paikkeilla (2.02 ... 2.06 µm), kts esim https://hal-insu.archives-ouvertes.fr/insu-02151491/document

        Mustan kappaleen spektrin säteilyteho on tuon laskurin mukaan välillä 2.02...2.06 µm vain 4.8E-6 W/m² steradiaania kohti. Jos happi käyttäytyisi tuolla spektrin välillä kuten täydellisen musta kappale (mitä se ei tee) niin hyvin paksun happikerroksen emissiviteetti ε huoneenlämmössä voisi olla enintään

        ε = 4.8E-6/133.027 = 3.6E-8 = 0.000000036

        Paksun happikerroksen läpäisevyys huoneenlämpötilassa olevan pinnan ulospäin lähtevälle mustan kappaleen lämpösäteilylle olisi siis 1-0.000000037 eli 99.999996%.

        Paksun typpikerroksen läpäisevyys samaiselle lämpösäteilylle olisi 99.9999999999999...% eli sen emissiviteetti ε olisi hyvin tarkkaan nollan suuruinen.

        On siis varsin selvää, että happi ja typpi eivät Maapallon pinnan ulospäin lähettämää säteilyä pysäytä pinnan lämpöbudjetin ja ilmaston kannalta katsottuna merkittävissä määrin. Alaspäin tulevan Auringon säteilyn osalta hapen lähi-infrapuna-alueen absorptiolla sen sijaan on jonkin verran vaikutusta.

        Ilmakehän hiilidioksidin emissiviteetin käsittely huoneenlämmössä tuolla yksinkertaistetulla tavalla olisi sikäli hankalaa että a) kaasulla on selkeästi Maapallon pintaa vastaavan mustan kappaleen spektrin alueelle osuvia absorptioviivoja ja b) ilmakehässä oleva hiilidioksidi ei laskelmien kannalta katsottuna ole paksu kun sen jokainen absorptioviiva ympäristöineen ei ole saturoitunut. Howellin kirjassa on laskettu sivulla 426 esimerkki 9.3 kuuman (830K), hyvin paksun ja 10 ilmakehän paineessa olevan hiilidioksidikerroksen emissiviteetistä yksinkertaistetun mallin mukaisesti.

        Tässä viimeisessä kappaleessa oli siis tarkoitus tuoda esiin että hiilidioksidin vaikutuksen määrittäminen ilmakehän lämpötilaan on laskelmilla täysin epämääräistä, vai?
        Jotain on jossain laskettu täysin ilmakehää vastaamattomilla oletuksilla.
        Mitään perusteluja ei ole esittää näiden teorioiden valossa että hiilidioksidi aiheuttaisi ilmakehän lämpenemistä.
        Teoria siitä siis kuuluisi ennemminkin uskontopalstalle kuin fysiikasta keskusteluun.


      • Anonyymi
        Aesdifaeger kirjoitti:

        Katsotaan edellisen perusteella miten kaasun spektriviivojen sijainti vaikuttaa kaasun emissiviteettiin matalassa lämpötilassa. Spektriviivojensa kohdalla kaasu voi paksuna kerroksena olla korkeintaan yhtä tehokas säteilijä kuin musta kappale. Spektriviivojen välisellä aallonpituusalueella kaasu ei säteile käytännössä lainkaan. Tehdään siis oletus, että kaasu käyttäytyy spektriviivojensa kohdalla kuten musta kappale (paksu kerros kaasua). Tu oletus on Howellin oppikirjassa erään laskentamenetelmistä perusteena.

        Planckin lakia on ikävä pyöritellä ilman taulukkolaskentaohjelmaa. Esimerkiksi tällä sivulla olevalla laskurilla saa Planckin säteilylaista mustan kappaleen spektrin intensiteetit esille ja voi laskea myös valitulle aallonpituusvälille osuvan säteilyn tehon.

        https://www.spectralcalc.com/blackbody_calculator/blackbody.php

        Otetaan huoneenlämpöinen kappale (20 astetta) eli T=293K ja emissiivisyys ε=1 eli musta kappale. Kun valitsee aallonpituusalueeksi ultravioletista hyvin pitkäaaltoiseen infrapunaan eli 0.01µm (lower limit) ja 1000µm (upper limit) niin säteilytehoa tuolle välille tulee 133.027W/m² steradiaania kohti. Tuo vastaa käytännössä koko mustan kappaleen spektriä.

        Kun valitsee aallonpituusalueeksi 0.4 µm ja 2 µm eli lyhytaaltoisen infrapunan ja näkyvän valon niin säteilytehoksi tulee 0.0000074 W/m² steradiaania kohti.
        Kun valitsee aallonpituusalueeksi 0.01 µm ja 0.4 µm eli ultravioletin säteilyn niin säteilytehoksi tulee 1.9E-46 W/m² steradiaania kohti. Tuo tarkoittaa lukua, jossa on desimaalipisteen jälkeen 45 nollaa ennenkuin tulee ensimmäinen nollasta poikkeava numero.

        On siis varsin selvää, että huoneenlämmössä olevan pelkkiä UV-alueen absorptioviivoja omaavan kaasun (N₂) emissiviteetti lämpösäteilylle on niin paljon pienempi kuin mustan kappaleen emissiviteetti että käytännössä sitä voi pitää nollan suuruisena.

        Hapella (O₂) sen sijaan on emissioviivoja noin 1.26...1.28 µm ja 2µm paikkeilla (2.02 ... 2.06 µm), kts esim https://hal-insu.archives-ouvertes.fr/insu-02151491/document

        Mustan kappaleen spektrin säteilyteho on tuon laskurin mukaan välillä 2.02...2.06 µm vain 4.8E-6 W/m² steradiaania kohti. Jos happi käyttäytyisi tuolla spektrin välillä kuten täydellisen musta kappale (mitä se ei tee) niin hyvin paksun happikerroksen emissiviteetti ε huoneenlämmössä voisi olla enintään

        ε = 4.8E-6/133.027 = 3.6E-8 = 0.000000036

        Paksun happikerroksen läpäisevyys huoneenlämpötilassa olevan pinnan ulospäin lähtevälle mustan kappaleen lämpösäteilylle olisi siis 1-0.000000037 eli 99.999996%.

        Paksun typpikerroksen läpäisevyys samaiselle lämpösäteilylle olisi 99.9999999999999...% eli sen emissiviteetti ε olisi hyvin tarkkaan nollan suuruinen.

        On siis varsin selvää, että happi ja typpi eivät Maapallon pinnan ulospäin lähettämää säteilyä pysäytä pinnan lämpöbudjetin ja ilmaston kannalta katsottuna merkittävissä määrin. Alaspäin tulevan Auringon säteilyn osalta hapen lähi-infrapuna-alueen absorptiolla sen sijaan on jonkin verran vaikutusta.

        Ilmakehän hiilidioksidin emissiviteetin käsittely huoneenlämmössä tuolla yksinkertaistetulla tavalla olisi sikäli hankalaa että a) kaasulla on selkeästi Maapallon pintaa vastaavan mustan kappaleen spektrin alueelle osuvia absorptioviivoja ja b) ilmakehässä oleva hiilidioksidi ei laskelmien kannalta katsottuna ole paksu kun sen jokainen absorptioviiva ympäristöineen ei ole saturoitunut. Howellin kirjassa on laskettu sivulla 426 esimerkki 9.3 kuuman (830K), hyvin paksun ja 10 ilmakehän paineessa olevan hiilidioksidikerroksen emissiviteetistä yksinkertaistetun mallin mukaisesti.

        Onhan ihan mukavaa että joku tuuraa allipainehitsaria kun alipainehitsari on jostain syystä estynyt julkaisemasta jokapäiväisiä huumoripläjäyksiään.


      • Anonyymi
        Anonyymi kirjoitti:

        Tässä viimeisessä kappaleessa oli siis tarkoitus tuoda esiin että hiilidioksidin vaikutuksen määrittäminen ilmakehän lämpötilaan on laskelmilla täysin epämääräistä, vai?
        Jotain on jossain laskettu täysin ilmakehää vastaamattomilla oletuksilla.
        Mitään perusteluja ei ole esittää näiden teorioiden valossa että hiilidioksidi aiheuttaisi ilmakehän lämpenemistä.
        Teoria siitä siis kuuluisi ennemminkin uskontopalstalle kuin fysiikasta keskusteluun.

        Viimeinen kappale alkaa lauseella: "Ilmakehän hiilidioksidin emissiviteetin käsittely huoneenlämmössä tuolla yksinkertaistetulla tavalla olisi sikäli hankalaa että [...]"

        Miten ihmeessä tuosta päättelit että hiilidioksidin emissiviteettiä ei voisi ollenkaan laskea? Sen laskeminen olisi hankalaa "tuolla yksinkertaistetulla tavalla" eli käyttäen menetelmää kohdassa "Sum of gray gases". Löytyy muitakin menetelmiä jotka ovat monimutkaisempia ja joissa on käytännössä pakko käyttää HITRAN - tietokantaa.

        Mikäli aihe kiinnostaa niin Howellin kirjassa ja kaukokartoitusartikkelissa on asiasta kerrottu lisää. Lue sieltä ja esitä kysymykset lukemasi perusteella. Jos olet liian laiska niihin tutustuaksesi (läksyt yhä tekemättä) niin se on edelleenkin sinun oma ongelmasi eikä edellytä esimerkiksi minulta mitään toimenpiteitä.


    • Anonyymi
      • Anonyymi

        Linkittämäsi aloitus oli ainakin päin seiniä. Jos ketjussa oli jotain järkevää, niin mikset kopsannut sitä? Kuka jaksaa kahlata läpi ketjun, josta jo aloituksen perusteella suuri osa on ilmeisesti sitä ihtiään.


      • Anonyymi
        Anonyymi kirjoitti:

        Linkittämäsi aloitus oli ainakin päin seiniä. Jos ketjussa oli jotain järkevää, niin mikset kopsannut sitä? Kuka jaksaa kahlata läpi ketjun, josta jo aloituksen perusteella suuri osa on ilmeisesti sitä ihtiään.

        Kyllä tuossa aloituksessa on asiaa. James Hansen aikoinaan ennusti että Arktiset jäät sulavat jos hiilidioksidin määrä kasvaa.
        Mahtaa alarmistipelleä harmittaa kun ennustelut ovat menneet pielenn.


      • Anonyymi
        Anonyymi kirjoitti:

        Linkittämäsi aloitus oli ainakin päin seiniä. Jos ketjussa oli jotain järkevää, niin mikset kopsannut sitä? Kuka jaksaa kahlata läpi ketjun, josta jo aloituksen perusteella suuri osa on ilmeisesti sitä ihtiään.

        Pahoittelen. Tuossa edellä piti olla lainausmerkit eli keskustelun sijaan "keskustelua".


    • Anonyymi

      Vaikka päivällä on 20 lämmintä ja tulee selkeä kesäyö niin halla voi aamuyöstä vieraiĺla. Aika herkästi tuo lämpö karkaa kun on sopivat olosuhteet

      • Anonyymi

        Kun vielä tiedetään että hiilidioksidin määrä en vuorokauden mittaan vaihtele, niin helposti huomataan että hiilidioksidin vaikutus Maapallon säteilemään IR-säteilyyn ja kasvihuoneilmiöön on olematon.


    • Anonyymi

      Joku aloitti uuden keskustelun kaasujen ominaisuuksista liittyen kasvihuoneilmiöön. Tähän mennessä tuolla ei ole tullut juurikaan uutta näkökulmaa asioihin. Keskustelu on vahvasti ylihuomenna Iso-Britanniassa päättyvän COP26 ilmastokonferenssin politiikan värittämää ilmeisesti asiaan kuuluvine trollauksineen.

      https://keskustelu.suomi24.fi/t/17187781/tietoa-kaasuista

      • Anonyymi

        Vidunko väliä.


    • Anonyymi

      Ilmakehän typen (N2) ja hapen (O2) vaikutus Maapallon lämpösäteilybudjettiin on käsitelty tässä artikkelissa vuodelta 2012:

      "The natural greenhouse effect of atmospheric oxygen (O2) and nitrogen (N2)"
      https://agupubs.onlinelibrary.wiley.com/doi/full/10.1029/2012GL051409

      Nekään eivät siis ole täysin vailla merkitystä infrapunaisen säteilyn absorboinnissa varsinkin tarkasteltaessa Maapallosta ulospäin lähtevää lämpösäteilyä. Artikkelin mukaan "We have found that on global average under clear-sky conditions the OLR is reduced due to O₂ by 0.11 Wm⁻² and due to N₂ by 0.17 Wm⁻²."

      Nykyisellään (2020) ihmisen aiheuttama kasvihuonekaasujen pitoisuuksien lisääntyminen on kasvattanut ilmakehän säteilypakotetta yli 3 Wm⁻² josta hiilidioksidin määrän kasvu aiheuttaa 2.0 Wm⁻², metaanin määrän kasvu 0.5 Wm⁻² ja CFC-yhtisteet yhdessä typpioksidien kanssa loput 0.5 Wm⁻². Nuo tiedot on koottu Wikipedian sivulle radiative forcing ja alun perin julkaistu tässä kirjallisuusviitteessä:

      https://en.wikipedia.org/wiki/Radiative_forcing#cite_note-EPA_NOAA_ClimateForcing1979-37

      Ilmakehän typen ja hapen vakiona pysynyt vaikutus on yhteensä 0.28 Wm⁻² eli alle kymmenesosa siitä, minkä lisyksen (AWG) ihminen on omilla toimillaan tähän mennessä aiheuttanut.

    • Kaikki lämpö siirtyy loppujen lopuksi säteilemällä avaruuteen.
      Myös konvektiossa siirtyvä siirtyy yläilmakerroksesta säteilynä.

    • Anonyymi

      Kun happi ja typpi eivät kykene emittoimaan lämpöään, pitää lämmön siirtyä ensin konvektion avulla kasvihuonekaasuihin.

    • Anonyymi
    • Anonyymi
      • Anonyymi

    • Anonyymi

      Maapallon ilmakehän ulkopuolella mitatun Auringon säteilyn spektrin aallonpituuden funktiona eriteltynä voi ladata koneelleen excelin tai libreoffice calc - ohjelman ymmärtämässä taulukkomuodossa tästä:

      https://www.nrel.gov/grid/solar-resource/spectra-astm-e490.html

      Klikkaa tuosta linkkiä "Download 2000 ASTM Standard ..." ja avaa tiedosto Excelillä tai libreoffice Calc - ohjelmalla.

    • Anonyymi
    • Anonyymi
      • Anonyymi

        Niin ja sama väärinkäsitys oli taas esillä, että lämpö siirtyisi ilmakehässä vain säteilemällä, ja volyymisuhteet karkasivat ymmärryshorisontin taakse.


      • Anonyymi
        Anonyymi kirjoitti:

        Niin ja sama väärinkäsitys oli taas esillä, että lämpö siirtyisi ilmakehässä vain säteilemällä, ja volyymisuhteet karkasivat ymmärryshorisontin taakse.

        Kukahan sellaista esittää? On totta, että pitkäaaltoinen maasta lähtevä säteily suuntautuu avaruuteen ja sitä absrboi vain joukko kasvihuonekaasuja. Eihän se sulje pois kulkeutumista ja johtumista.
        Jos taas katsotaan maapalloa termodynamisena järjestelmänä, niin se luovuttaa lämpöä ainoastaan säteilemällä.


    • Anonyymi
    • Anonyymi
    • Anonyymi
    • Anonyymi
    • Anonyymi

      https://keskustelu.suomi24.fi/t/15428403/tiede-ja-vaitteet-ilmaston-viilenemisesta#comment-122686391

      Tuossa keskustelussa joku virheellisesti väittää linkkiä antamatta jollakin taajuudella tulevan sähkömagneettisen säteilyn absorboituvan kolmen settimetrin matkalla ilmakehässä NTP - olosuhteissa.

      Ehdotin keskustelun jatkamista aiheeseen liittyvässä säikeessä (tässä) mutta saa nähdä oliko kyse vain trollauksesta.

      • Anonyymi

      • Anonyymi

        Ymmärsitkö lainkaan lukemaasi, se 3 cm keskimääräinen matka on jopa erikseen selitetty.

        NTP tila on mainittu, koska moolitilavuus ei ole eri olosuhteissa vakio, joten koko ihmettelysi on samassa suhteessa kuin kirjoittamasi esitys "Energian siirtyminen lämpösäteilyn avulla", joten pidän opiskelusi tässä vaiheessa vetäytymistäsi asiakeskustelusta, kaikkien kannalta järkevänä.


      • Anonyymi
        Anonyymi kirjoitti:

        Ymmärsitkö lainkaan lukemaasi, se 3 cm keskimääräinen matka on jopa erikseen selitetty.

        NTP tila on mainittu, koska moolitilavuus ei ole eri olosuhteissa vakio, joten koko ihmettelysi on samassa suhteessa kuin kirjoittamasi esitys "Energian siirtyminen lämpösäteilyn avulla", joten pidän opiskelusi tässä vaiheessa vetäytymistäsi asiakeskustelusta, kaikkien kannalta järkevänä.

        Viestissäsi sanot säteilyn absorboituvan keskimäärin 3 cm matkalla NTP olosuhteista ilmakehää, Jotta tästä päästäisiin eteenpäin niin haluaisin tietää tämän:

        Millä taajuudella tai aallonpituudella oleva sähkömagneettinen säteily sinun mielestäsi absorboituu 3 cm matkalla ilmakehää? Tuo kun tarkoittaa absorptiokerrointa μ, joka olisi luokkaa suurempi kuin 0.3 cm⁻¹

        I(x)/I(0) = exp(-µx)

        Yksittäisien ultraviolettialueen absorptiopiikkien kohdalla tuo voi toteutua. Molekyläärisen hapen ja typen absorptiopiikit lyhytaaltoisen ultravioletin säteilyn aallonpituuksilla ovat hyvin voimakkaita.

        Maapallon pinnan ulospäin lähettämän lämpösäteilyn aallonpituuksilla noin voimakasta absorptioita ei käytännössä tapahdu. Kasvihuonekaasujen konsentraatiot eivät tuohon riitä kuin korkeintaan jonkun hyvin kapean spektripiikin kohdalla eli pitkäaaltoinen infrapuna pääsee suurella osalla kaistaa käytännössä vaimentumatta koko ilmakehän lävitse. Tarkkoja ilman absortiospektrejä pääset katsomaan Hitranin kautta.

        https://hitran.org


      • Anonyymi
        Anonyymi kirjoitti:

        Viestissäsi sanot säteilyn absorboituvan keskimäärin 3 cm matkalla NTP olosuhteista ilmakehää, Jotta tästä päästäisiin eteenpäin niin haluaisin tietää tämän:

        Millä taajuudella tai aallonpituudella oleva sähkömagneettinen säteily sinun mielestäsi absorboituu 3 cm matkalla ilmakehää? Tuo kun tarkoittaa absorptiokerrointa μ, joka olisi luokkaa suurempi kuin 0.3 cm⁻¹

        I(x)/I(0) = exp(-µx)

        Yksittäisien ultraviolettialueen absorptiopiikkien kohdalla tuo voi toteutua. Molekyläärisen hapen ja typen absorptiopiikit lyhytaaltoisen ultravioletin säteilyn aallonpituuksilla ovat hyvin voimakkaita.

        Maapallon pinnan ulospäin lähettämän lämpösäteilyn aallonpituuksilla noin voimakasta absorptioita ei käytännössä tapahdu. Kasvihuonekaasujen konsentraatiot eivät tuohon riitä kuin korkeintaan jonkun hyvin kapean spektripiikin kohdalla eli pitkäaaltoinen infrapuna pääsee suurella osalla kaistaa käytännössä vaimentumatta koko ilmakehän lävitse. Tarkkoja ilman absortiospektrejä pääset katsomaan Hitranin kautta.

        https://hitran.org

        Miksi linkität taas tutkielmia säteilyn suuruudesta, onko kohdallasi edelleen ongelmia ymmärtää, että energiaa siirtyy vain sen taajuuden osalta, mitä vastaanottaja kykenee absorboimaan.

        "Musta kappale", jonka varaan rakennat teoriaasi on fiktiota, samoin kuin sen pohjalle laskettu teho, ja lämpötilaperusteinen spektrin laajuus.

        Esimerkiksi maanpinnan (290 K)
        S_B kaavan mukaan säteilyteho on 400 W/m², se jakautuu Wienin siirtymälain mukaan 4...60 µ alueelle, CO2 ansorboi aluetta 15 µ, ja jos sen kaistaleveys on 1 µ se voisi maksimissaan absorboida 5.5 W/m² tehon, mutta CO2 .n absorptioteho ei ole 100 %, ja samala taajuutta absorptioi myös H2O, määrältään moninkertaisena.

        Tuon oli määrä selittää, että lämmön siirtyminen säteilyssä on aivan muuta kuin säteilytehon funktio, ja koska lämpö on molekyylien liikettä, niin se on vain yksi osa konduktiota, joka myös itse aiheuttaa lämpösäteilyä,- jne .., niin yksinkertaiset kuvitelmat lämmön siirrosta ovat aika huvittavia.

        Toinen juttu on se 3 cn, johon näytät hirttäytyneen, se on tietyssä tilassa arvioitu keskimääräinen arvo, koko spekrille, ei mikään yksittäisen taajuuden.
        Tämänkin tiedon olisi saanut pelkästään lukemalla edelliset viestit.

        Lopuksi, jos mieltäsi jäi jokin vielä vaivaamaan, avaa uusi keskustelu, tämän ketjun jatkumona tunnen olevani jossain pilailupalstalla.


      • Anonyymi
        Anonyymi kirjoitti:

        Miksi linkität taas tutkielmia säteilyn suuruudesta, onko kohdallasi edelleen ongelmia ymmärtää, että energiaa siirtyy vain sen taajuuden osalta, mitä vastaanottaja kykenee absorboimaan.

        "Musta kappale", jonka varaan rakennat teoriaasi on fiktiota, samoin kuin sen pohjalle laskettu teho, ja lämpötilaperusteinen spektrin laajuus.

        Esimerkiksi maanpinnan (290 K)
        S_B kaavan mukaan säteilyteho on 400 W/m², se jakautuu Wienin siirtymälain mukaan 4...60 µ alueelle, CO2 ansorboi aluetta 15 µ, ja jos sen kaistaleveys on 1 µ se voisi maksimissaan absorboida 5.5 W/m² tehon, mutta CO2 .n absorptioteho ei ole 100 %, ja samala taajuutta absorptioi myös H2O, määrältään moninkertaisena.

        Tuon oli määrä selittää, että lämmön siirtyminen säteilyssä on aivan muuta kuin säteilytehon funktio, ja koska lämpö on molekyylien liikettä, niin se on vain yksi osa konduktiota, joka myös itse aiheuttaa lämpösäteilyä,- jne .., niin yksinkertaiset kuvitelmat lämmön siirrosta ovat aika huvittavia.

        Toinen juttu on se 3 cn, johon näytät hirttäytyneen, se on tietyssä tilassa arvioitu keskimääräinen arvo, koko spekrille, ei mikään yksittäisen taajuuden.
        Tämänkin tiedon olisi saanut pelkästään lukemalla edelliset viestit.

        Lopuksi, jos mieltäsi jäi jokin vielä vaivaamaan, avaa uusi keskustelu, tämän ketjun jatkumona tunnen olevani jossain pilailupalstalla.

        "onko kohdallasi edelleen ongelmia ymmärtää, että energiaa siirtyy vain sen taajuuden osalta, mitä vastaanottaja kykenee absorboimaan."

        Energiaa siirtyy ilmakehästä avaruuteen vain säteilemällä, joten kerrotko mikä avaruudessa on se vastaanottaja joka tämän kaiken energian siis absorboi?

        Minuakin kiinnostaisi tietää mistä tuo keskimääräinen säteilyn absorptio NTP:ssä 3 sentin matkalla on napattu - toistaiseksi selittelyt huonolla kirjoitustaidolla eivät ole vakuuttaneet.


      • Anonyymi
        Anonyymi kirjoitti:

        "onko kohdallasi edelleen ongelmia ymmärtää, että energiaa siirtyy vain sen taajuuden osalta, mitä vastaanottaja kykenee absorboimaan."

        Energiaa siirtyy ilmakehästä avaruuteen vain säteilemällä, joten kerrotko mikä avaruudessa on se vastaanottaja joka tämän kaiken energian siis absorboi?

        Minuakin kiinnostaisi tietää mistä tuo keskimääräinen säteilyn absorptio NTP:ssä 3 sentin matkalla on napattu - toistaiseksi selittelyt huonolla kirjoitustaidolla eivät ole vakuuttaneet.

        Keskustelun jatkaminen ei kannata ennen kuin tuo 3cm väitteelle löytyy jotakin näyttöä. Ilmanhan pitäisi olla tuolloin olla kuin tummaa savua.

        Jos ilma oikeasti absorboisi säteilystä puolet 3cm matkalla niin tuhannen watin valonlähde ei näkyisi edes metrin päähän. Kun siis tuohon metriin mahtuisi 33 peräkkäistä säteilyn intensiteetin puolittumista.

        Nyt väitteen esittäjä laitapa linkki tutkimukseen josta tuo väittämäsi 3cm matka on peräisin. Jatketaan keskustelua tästä sitten kun olemme muutkin kyseisen tiedonlähteen päässeet tarkistamaan.


      • Anonyymi
        Anonyymi kirjoitti:

        Keskustelun jatkaminen ei kannata ennen kuin tuo 3cm väitteelle löytyy jotakin näyttöä. Ilmanhan pitäisi olla tuolloin olla kuin tummaa savua.

        Jos ilma oikeasti absorboisi säteilystä puolet 3cm matkalla niin tuhannen watin valonlähde ei näkyisi edes metrin päähän. Kun siis tuohon metriin mahtuisi 33 peräkkäistä säteilyn intensiteetin puolittumista.

        Nyt väitteen esittäjä laitapa linkki tutkimukseen josta tuo väittämäsi 3cm matka on peräisin. Jatketaan keskustelua tästä sitten kun olemme muutkin kyseisen tiedonlähteen päässeet tarkistamaan.

        Kyllä tuon täytyy olla totta. Eihän ihmisen tupruttelujen aiheuttamaa ilmastonlämpenemistä muuten olisi olemassa.


      • Anonyymi
        Anonyymi kirjoitti:

        Keskustelun jatkaminen ei kannata ennen kuin tuo 3cm väitteelle löytyy jotakin näyttöä. Ilmanhan pitäisi olla tuolloin olla kuin tummaa savua.

        Jos ilma oikeasti absorboisi säteilystä puolet 3cm matkalla niin tuhannen watin valonlähde ei näkyisi edes metrin päähän. Kun siis tuohon metriin mahtuisi 33 peräkkäistä säteilyn intensiteetin puolittumista.

        Nyt väitteen esittäjä laitapa linkki tutkimukseen josta tuo väittämäsi 3cm matka on peräisin. Jatketaan keskustelua tästä sitten kun olemme muutkin kyseisen tiedonlähteen päässeet tarkistamaan.

        Säteilyä tulee töki muualtakin kuin maan pinnalta tai auringosta. Jokaikinen molekyyli on potentiaalinen säteilylähde, jota ympäröi sen 3 cm säteellä 2,8 e+21 molekyyliä, ja jos ihminen näkisi kaikki molekyylit, olisimme "sokeita"

        3 cm arvoitustasi saat hakea itse, haulla "fotone age on air" löytynee 0,1 ns .


      • Anonyymi
        Anonyymi kirjoitti:

        Säteilyä tulee töki muualtakin kuin maan pinnalta tai auringosta. Jokaikinen molekyyli on potentiaalinen säteilylähde, jota ympäröi sen 3 cm säteellä 2,8 e 21 molekyyliä, ja jos ihminen näkisi kaikki molekyylit, olisimme "sokeita"

        3 cm arvoitustasi saat hakea itse, haulla "fotone age on air" löytynee 0,1 ns .

        Pilailupalstalta tosiaan näyttää, kun väitetyn "tiedon" todisteeksi annetaan tuollaista dataa, hakuohjeita! Mitähän kieltä tuo edes on...?


    • Anonyymi

      Tuo kummallisen 3cm väitteen esittäjä näyttää sopivasti unohtaneen sen, että todistuksen taakka on aina väitteen esittäjällä eikä sitä voi siirtää lukijoille.

      https://fi.wikipedia.org/wiki/Todistuksen_taakka

      Poikkeuksellinen väite (kuten tuo 3cm) vaatii poikkeukselliset todisteet. Ja niiiden todisteiden esittäminen on väitteen esittäjän tehtävä. Mikään ylimalkainen kehoitus googlettaa ei tietenkään kelpaa vaan pitää olla täsmällinen linkki/viitetieto.

      • Anonyymi

        'Kelpaako linkki omaan edelliseen paskanjauhantaan' itsesi tavoin, vai minkä tyylistä linkkiä kaipaat.
        Tuo 3 sm oli vain laskettu matka 0,1 ns keskimääräisestä oletetusta ajasta, jolla itsellään ei ole mitään merkitystä millekään muulle kuin typerälle inttämisellesi.

        Kyseinen arvio ei liity mitenkään mihinkään tapahtumaan, keskeinen ruikutus keskittynee nyt nolon "asiantuntijan" säälittäviin yrityksiin pelastaa kasvojaan ja kuvitelmiin typeryyksistänsä venyttämisen edes hiukan, toivoen sen palauttavan uskottavuuttaan.

        Toivotan onnea tulevaisuudellesi, tulet todennäköisesti tarvitsevasi juuri sitä.


      • Anonyymi
        Anonyymi kirjoitti:

        'Kelpaako linkki omaan edelliseen paskanjauhantaan' itsesi tavoin, vai minkä tyylistä linkkiä kaipaat.
        Tuo 3 sm oli vain laskettu matka 0,1 ns keskimääräisestä oletetusta ajasta, jolla itsellään ei ole mitään merkitystä millekään muulle kuin typerälle inttämisellesi.

        Kyseinen arvio ei liity mitenkään mihinkään tapahtumaan, keskeinen ruikutus keskittynee nyt nolon "asiantuntijan" säälittäviin yrityksiin pelastaa kasvojaan ja kuvitelmiin typeryyksistänsä venyttämisen edes hiukan, toivoen sen palauttavan uskottavuuttaan.

        Toivotan onnea tulevaisuudellesi, tulet todennäköisesti tarvitsevasi juuri sitä.

        Ei edelleenkään linkkiä eli höpöttelet jotakin ihan omiasi tuosta 3cm perustuen johonkin väärinymmärrykseesi. Se ei onneksi ole minun ongelmani eli minä en joudu selvittämään ajatusvirhettäsi eikä minun tarvitse korjata keskusteluissa esitettyjä virheellisiä väitteitä vaikka niitä toisteltaisiinkin.

        Kun jatkossa jäät vastauksia vaille niin se ei ole hiljaista hyväksyntää vaan uuden trollin jättämistä vaille huomiota. Epäkohteliaille vastailuun elämä on liian lyhyt.


      • Anonyymi
        Anonyymi kirjoitti:

        Ei edelleenkään linkkiä eli höpöttelet jotakin ihan omiasi tuosta 3cm perustuen johonkin väärinymmärrykseesi. Se ei onneksi ole minun ongelmani eli minä en joudu selvittämään ajatusvirhettäsi eikä minun tarvitse korjata keskusteluissa esitettyjä virheellisiä väitteitä vaikka niitä toisteltaisiinkin.

        Kun jatkossa jäät vastauksia vaille niin se ei ole hiljaista hyväksyntää vaan uuden trollin jättämistä vaille huomiota. Epäkohteliaille vastailuun elämä on liian lyhyt.

        Hävettääkö yhtään ?


      • Anonyymi
        Anonyymi kirjoitti:

        'Kelpaako linkki omaan edelliseen paskanjauhantaan' itsesi tavoin, vai minkä tyylistä linkkiä kaipaat.
        Tuo 3 sm oli vain laskettu matka 0,1 ns keskimääräisestä oletetusta ajasta, jolla itsellään ei ole mitään merkitystä millekään muulle kuin typerälle inttämisellesi.

        Kyseinen arvio ei liity mitenkään mihinkään tapahtumaan, keskeinen ruikutus keskittynee nyt nolon "asiantuntijan" säälittäviin yrityksiin pelastaa kasvojaan ja kuvitelmiin typeryyksistänsä venyttämisen edes hiukan, toivoen sen palauttavan uskottavuuttaan.

        Toivotan onnea tulevaisuudellesi, tulet todennäköisesti tarvitsevasi juuri sitä.

        Mikä on äidinkielesi, suomi ei näytä oikein sujuvan?


      • Anonyymi
        Anonyymi kirjoitti:

        Mikä on äidinkielesi, suomi ei näytä oikein sujuvan?

        Voi aikoja, ...

        Olen kuullut että nykyiset "niinku-jonnet" ei enää ymmärrä kahta sanaa pidempiä lauseita, ja ihmetyksesi näyttää tukevan kuulemaani.


      • Anonyymi
        Anonyymi kirjoitti:

        Voi aikoja, ...

        Olen kuullut että nykyiset "niinku-jonnet" ei enää ymmärrä kahta sanaa pidempiä lauseita, ja ihmetyksesi näyttää tukevan kuulemaani.

        Olet oikeassa, esim. näissä lauseissa tipahdan viimeistään kahden sanan jälkeen:
        "kuvitelmiin typeryyksistänsä venyttämisen edes hiukan"
        "tulet todennäköisesti tarvitsevasi juuri sitä"

        Käännöskone vai pelkästään hoono soomi?


      • Anonyymi
        Anonyymi kirjoitti:

        Olet oikeassa, esim. näissä lauseissa tipahdan viimeistään kahden sanan jälkeen:
        "kuvitelmiin typeryyksistänsä venyttämisen edes hiukan"
        "tulet todennäköisesti tarvitsevasi juuri sitä"

        Käännöskone vai pelkästään hoono soomi?

        Jos nuo lauseet tuottavat vaikeuksia ymmärryksellesi, niin suosittelen hakeutumista jonkinlaiseen terapiaan, tai hakemaan muuta yhteiskunnan tukitoimia haitallisen vajavuutesi auttamiseksi.


      • Anonyymi
        Anonyymi kirjoitti:

        Jos nuo lauseet tuottavat vaikeuksia ymmärryksellesi, niin suosittelen hakeutumista jonkinlaiseen terapiaan, tai hakemaan muuta yhteiskunnan tukitoimia haitallisen vajavuutesi auttamiseksi.

        " hakemaan muuta yhteiskunnan tukitoimia"
        Veikkaan huonoa kielitaitoa, Google-kääntäjä ei tee noin noloja virheitä.


    • Anonyymi

      Kumpula suljettiin liiallisten päästöjen vuoksi etukäteen.

    • Anonyymi
    • Anonyymi
    • Anonyymi

      Lämpösäteilyn vaikutusta Maapallon energiabudjettiin käsittelee tämä:

      https://en.wikipedia.org/wiki/Radiative_forcing

      Ihmisen aiheuttamien ilmakehän koostumuksen muutosten vuoksi säteilytasapaino on muuttunut siten, että tällä hetkellä Maapallolle tulee luokkaa 2.72 W/m^2 tehoa enemmän kuin esiteollisena aikana (säteilypakote, radiative forcing). Tässä luvussa on siis kasvihuonekaasujen ja aerosolien vaikutukset mukana. Wikipedian sivun tiedon lähteenä säteilypakotteen muutokselle on IPCC AR6 yhteenveto, jonka tiedonlähteet on IPCC_AR6_WGI_FullReport.pdf tiedostossa lueteltu (2409 sivua, koko 430 megatavua). Luku löytyy sivulta 926 (luku 7) ja tuon luvun lopussa on siinä tietolähteenä käytettyjen noin 980 julkaisun viitetiedot niille, jotka haluavat asioita itse lähemmin tarkastella.

      Climate Change 2021
      The Physical Science Basis
      Working Group I Contribution to the Sixth Assessment Report of the Intergovernmental Panel on Climate Change
      https://www.ipcc.ch/report/ar6/wg1/

      Keskimääräinen Auringosta Maan ilmakehän yläosaan tuleva säteilyteho on 340 wattia per neliömetri laskettuna keskiarvona vuoden ajalta yli koko pallon pinnan.

      Tuosta luvusta 2.72 W/m^2 on noin 0.8%

      Auringon säteilyteho kasvaa tällä hetkellä ajan nopeudella 8% miljardissa vuodessa:

      https://environmental-geology-dev.pressbooks.tru.ca/chapter/changes-in-solar-output-and-in-the-earths-atmosphere/

      Ihmisen aiheuttama säteilypakotteen nousu vastaa siten tilannetta, jossa Auringon säteilyteho olisi jo nyt äkillisesti kasvanut sen määrän, minkä se tulee kasvamaan seuraavan sadan miljoonan vuoden kuluessa.

    Ketjusta on poistettu 8 sääntöjenvastaista viestiä.

    Luetuimmat keskustelut

    1. Heikki Silvennoinen petti vaimoaan vuosien ajan

      Viiden lapsen isä Heikki kehuu kirjassaan kuinka paljon on pettänyt vaimoaan vuosien varrella.
      Kotimaiset julkkisjuorut
      279
      4619
    2. Miksi ihmeessä nainen seurustelit kanssani joskus

      Olin ruma silloin ja nykyisin vielä rumempi En voi kuin miettiä että miksi Olitko vain rikki edellisestä suhteesta ja ha
      Ikävä
      28
      2498
    3. Taasko se show alkaa

      Koo osottaa taas mieltään
      Ikävä
      24
      2181
    4. Persut nimittivät kummeli-hahmon valtiosihteeriksi!

      Persujen riveistä löytyi taas uusi törkyturpa valtiosihteeriksi! Jutun perusteella järjenjuoksu on kuin sketsihahmolla.
      Perussuomalaiset
      93
      2166
    5. Onko ministeri Juuso epäkelpo ministerin tehtäviensä hoitamiseen?

      Eikö hänellä ole kompetenttia hoitaa sosiaali- ja terveysministetin toimialalle kuuluvia ministerin tehtäviä?
      Perussuomalaiset
      129
      1817
    6. Sakarjan kirjan 6. luku

      Jolla korva on, se kuulkoon. Sain profetian 22.4.2023. Sen sisältö oli seuraava: Suomeen tulee nälänhätä niin, että se
      Profetiat
      26
      1462
    7. Söpö lutunen oot

      Kaipaan aina vaan, vaikkakin sitten yksipuolisesti.
      Ikävä
      16
      1442
    8. Kenen etua Stubb ajaa Euroopassa ilmoittaessaan olevansa enemmän Ruotsalainen

      Tasavallan presidentti Alexander Stubb kertoi ensimmäisellä valtiovierailullaan Ruotsissa, että hän ei ole koskaan tunte
      Maailman menoa
      343
      1418
    9. Avaa sydämesi mulle

      ❤ ❤❤ Tahdon pelkkää hyvää sulle Sillä ilmeisesti puhumalla Avoimesti välillämme Kaikki taas selviää Kerro kaikki, tahdo
      Ikävä
      36
      1357
    10. Elia tulee vielä

      Johannes Kastaja oli Elia, mutta Jeesus sanoi, että Elia tulee vielä. Malakian kirjan profetia Eliasta toteutuu kokonaan
      Helluntailaisuus
      35
      1247
    Aihe